Sexual Reproduction in Flowering Plants Class 12 Important Extra Questions Biology Chapter 2

Here we are providing Class 12 Biology Important Extra Questions and Answers Chapter 2 Sexual Reproduction in Flowering Plants. Important Questions for Class 12 Biology are the best resource for students which helps in Class 12 board exams.

Class 12 Biology Chapter 2 Important Extra Questions Sexual Reproduction in Flowering Plants

Sexual Reproduction in Flowering Plants Important Extra Questions Very Short Answer Type

Question 1.
Name the component cells of the egg apparatus in an embryo sac?
Answer:
One egg or oosphere and two synergids.

Question 2.
Name the part of gynoecium that determines the compatible nature of pollen grain?
Answer:
Stigma

Question 3.
Name the common function that cotyledons and nucellus perform?
Answer:
Nourishment (embryo by cotyledons and embryo sac by nucellus).

Question 4.
Complete the following flow chart. Pollen grain?
Answer:
Class 12 Biology Important Questions Chapter 2 Sexual Reproduction in Flowering Plants 1
A = Generative cell.

Question 5.
Name the structure formed the development of microsporangia?
Answer:
Pollen sac.

Question 6.
Why pollen grains can be preserved as fossils?
Answer:
It is due to the presence of sporopollenin present in exine.

Question 7.
Indicate the stage where meiosis occurs (1, 2 or 3) in the flow chart.
Class 12 Biology Important Questions Chapter 2 Sexual Reproduction in Flowering Plants 2
Answer:
1 = Meiosis

Question 8.
State the reason why pollen grains lose their viability when the tapetum in the anther is malfunctioning?
Answer:
As tapetum provides nutrition, the pollen loses its viability due to non-availability of nutrition.

Question 9.
In a case of polyembryony, if an embryo develops from the synergid and another from the nucellus, which is haploid and which is diploid?
Answer:
Synergid embryo is haploid (n) and nucellus embryo is diploid (2n).

Question 10.
Can an unfertilised, apomictic embryo sac give rise to a diploid embryo? If yes, then how?
Answer:
Yes, if the embryo develops from the cells of nucellus or integument, it will be diploid.

Question 11.
Why is an embryo sac diploid in agamospermy?
Answer:
The embryo sac is diploid because it is formed without reductional division.

Question 12.
How do drones develop in honeybees? Name the process.
Answer:
In honeybees, unfertilised eggs develop the drones. They are sterile male. The process is called parthenogenesis.

Question 13.
What is chalaza?
Answer:
Chalaza is the part of nucellus from where integuments arise.

Question 14.
What do you mean by agamospermy?
Answer:
Agamospermy is the formation of embryos by asexual reproductive processes.

Question 15.
What types of pollution occur in Vallisneria? (CBSE Delhi 2019)
Answer:
By water (Hydrophily)

Question 16.
Name a few plants which show viviparous germination?
Answer:
Rhizophora and Sonneratia.

Question 17.
A bilobed, dithecous anther has 100 microspore mother cells per microsporangium. How many male gametophytes this anther can produce? (CBSE 2010)
Answer:
100 × 4 = 400.

Question 18.
An anther with malfunctioning tapetum often fails to produce viable male gametophyte? Give one reason? (CBSE 2010)
Answer:
Tapetum provides nutrition. In the case of malfunctioning tapetum, due to absence of nutrition, it fails to produce viable gametophyte.

Question 19.
How many pollen grains and ovules are likely to be formed in the anther and the ovary of an angiosperm bearing 25 microspore mother cells and 25 megaspore mother cells respectively? (CBSE Sample Paper 2018-19)
Answer:
25 Microspore mother cells will form 100 pollen grains, whereas 25 megaspore mother cells will form 25 ovules respectively.

Question 20.
In the case of polyembryony, an embryo A develops from the synergids and the embryo B develops from the nucellus. State the ploidy of embryo A and B? (CBSE Sample Paper 2018-19)
Answer:
A-Haploid
B-Diploid

Question 21.
Write one advantage and one disadvantage of cleistogamy to flowering plants? (CBSE Delhi 2018C)
Answer:
Advantage:
It maintains pure lines.

Disadvantage:
Only parental characters are preserved.

Question 22.
Why does the zygote begin to divide only after the division of the primary endosperm cell?
Answer:
Division and storage of food in primary endosperm cell forms endosperm which is pre-requirement for developing embryo, which is formed due to division of the zygote.

Sexual Reproduction in Flowering Plants Important Extra Questions Short Answer Type

Question 1.
What is inflorescence?
Answer:
The arrangement and distribution of flowers on a peduncle are called an inflorescence. The axis of the inflorescence is termed peduncle. The flattened peduncle is called the receptacle.

Question 2.
Differentiate between flower and inflorescence?
Answer:
Differences between flower and inflorescence:

Flowers Inflorescence
1. It is a modified shoot. 1. It is an arrangement of flowers on the floral axis.
2. The stalk of flowers is called Pedicet. 2. The stalk of an inflorescence is called a peduncle.

Question 3.
List the functions of a flower?
Answer:
Functions of a flower:

  1. Production of pollen and ovules.
  2. Aids in pollination.
  3. Fertilisation.
  4. Development of seed and fruit.
  5. Helps in the dispersal of seeds and fruits.

Question 4.
Name the various methods by which anther dehisces?
Answer:

  1. By Longitudinal intros.
  2. By Longitudinal dextrose.
  3. By Apical pores.
  4. Transverse dehiscence.

Question 5.
Geitonogamy flowering plants are genetically autogamous but functionally cross-pollinated? Justify? (CBSE 2013)
Answer:

  1. Geitonogamy. It is a transfer of pollen grains from the anther to the stigma of another flower of the same plant.
  2. Geitonogamy is functionally cross-pollination because it involves a pollinating agent, however genetically it is similar to autogamy since the pollen grains come from the same plant.

Question 6.
List the advantages of self-pollination?
Answer:
Advantages of self-pollination:

  1. Parental characters can be preserved.
  2. Self-pollination helps in maintaining pure lines.
  3. No extra energy required, thus it is an economical process.
  4. Even if smaller quantities of pollen grains are produced, there is a high chance of pollination.
  5. Self-pollination avoids wastage of pollen grains.

Question 7.
What are the disadvantages of self¬pollination?
Answer:
Disadvantages of self-pollination:

  1. There is a gradual decrease in the vigour of the plants produced through self-pollination.
  2. The yield is decreased.
  3. The size and number of seeds also go on decreasing generation after generation.
  4. The plants become susceptible to diseases.

Question 8.
Give the characters of wind-pollinated flowers?
Answer:
Characters of wind-pollinated flowers:

  1. The size of the flower is very small.
  2. Bright coloured corolla, nectar glands and fragrance are absent.
  3. Pollen grains are produced in large quantities.
  4. Versatile fixation of anthers aids in shaking off pollen grains in the air.
  5. Pollen grains are dry, smooth and very light, e.g. coconut palm, date palm, Cannabis (Bhang), maize and grasses.

Question 9.
Define ornithophily, chiropterophily and malacophilous?
Answer:

  • Ornithophily: Pollination carried out by small birds is called ornithophily, e.g. Australian plants.
  • Chiropterophily: It is pollination by bats, e.g. Sausage tree.
  • Malacophily: Snails also bring pollination; it is termed malacophilous.

Question 10.
Give an account of the significance of double fertilisation? (CBSE 2010)
Answer:
Significance of double fertilisation:

  1. After double fertilisation, the development of the endosperm starts when it is required by developing embryo.
  2. It ensures that nutritive tissue is formed.
  3. It is an advanced character over gymnosperms where endosperm develops before fertilisation. If fertilisation fails, the energy spent on endosperm development gets wasted; while in angiosperms it develops after double fertilisation.
  4. Double fertilisation provides the characteristics of male plants as well as the nutritive tissue.

Question 11.
Describe the structure of the fruit?
Answer:
Structure of Fruit: True fruit is enclosed by a wall called the pericarp. When well developed and fleshy, it is differentiated (in most fruits) into three distinct parts-an outer epicarps, middle mesocarp and inner endocarp. In the ripe mango the outer, thin and leathery part (peel), which is usually discarded, is the epicarp.

The sweet, fleshy part that is edible is the mesocarp, and the innermost hard and woody part that encloses the seed is the endocarp. The nature of these three parts, however, varies in different fruits.

In dry fruits, the pericarp is dry, usually papery or woody, and is not distinguishable into the three parts.

Question 12.
(i) Suppose the haploid number of chromosomes in a flowering plant is 12. What will be the ploidy in the cells of integuments, nucellus, antipodals, endosperm and embryo of that plant?
Answer:

Name of the part No. of Chromosomes
(a) Integument (2n) 12 × 2 = 24
(b) NuceLtus (2n) 12 × 2 = 24
(c) Antipodats (in) 12 × 1 = 12
(d) Endosperm (3n) 12 × 3 = 36
(e) Embryo (2n) 12 × 2 = 24

(ii) How many haploid cells are present in a mature female gametophyte of a flowering plant? Name them? (CBSE 2010)
Answer:
(a) 6 haploid cells
(b) 3 antipodals, 1 egg cell and 2 synergids.

Question 13.
What do you mean by monosporic development of female gametophyte?
Answer:
In monosporic development as in (Polygonum) the development of embryo sac, only one megaspore situated towards chalazal end remains functional. The remaining three megaspores gradually degenerate and finally disappear. The functional haploid megaspore enlarges in size and, by means of three successive mitotic divisions, gives rise to an eight- nucleate embryo sac.

Question 14.
(i) How are parthenocarpic fruit produced by some plants and apomictic seeds by some? Explain.
Answer:
Parthenocarpy is the production and development of seedless fruits without pollination and fertilisation. Parthenocarpic fruits are normal. Example: Banana, Pineapple, Guava, Grapes, etc.

Apomixis is the mode of reproduction which does not involve the formation of zygote through gametic fusion. Example: Some species of grass.

(ii) When do farmers prefer using apomictic seeds? (CBSE Outside Delhi 2016)
Answer:
Farmers prefer apomictic seeds because they are free from infection and avoid using hybrid seeds which are costly.

Question 15.
Write any two ways by which apomictic seeds may be developed in angiosperms? (CBSE Delhi 2019 C) Answer:
Parthenocarpy. In certain plants, the fruits are formed without the act of fertilisation. Such fruits are called parthenocarpic fruits and the phenomenon is known as parthenocarpy. Parthenocarpic fruits are either seedless or contain empty or non-viable seeds.

In these fruits, the stimulus (seed factor) for fruit growth is provided by the tissue of the ovary wall itself. Seedless varieties of grapes and oranges contain those of the seeded varieties. Most commonly cultivated varieties of banana and pineapple are parthenocarpic. Polyembryony: Formation of more than one embryo in the seed. Onions, groundnuts, mangoes lemons and oranges are some of the examples. Polyembryony is very common among conifers.

Question 16.
You are conducting artificial hybridisation on papaya and potato. Which one of them would require the step of emasculation and why? However, for both, you will use the process of bagging. Justify giving one reason? (CBSE Delhi 2019)
Answer:

  1. Potato: Potato plant has a bisexual flower, which means it has both male and female reproductive parts in the same flower. And the plant is Monoecious. Hence it needs emasculation to prevent autogamy.
  2. Papaya: In case of papaya plant, male and female flowers are present on separate plants, that is each plant is either male or female (dioecious). This condition prevents both autogamy and geitonogamy.

So for artificial hybridisation, both will require bagging to prevent contamination of stigma.

Question 17.
Normally one embryo develops in one seed but when an orange seed is squeezed many embryos of different shapes and sizes are seen. Mention how it has happened? (CBSE 2011)
Answer:
In many citrus varieties, some of the nucellar cells surrounding the embryo sac start dividing, protrude into embryo sac and develop into embryos. In such species, each ovule contains many embryos. The occurrence of more than one embryo in a seed is referred to as polyembryony.

Question 18.
A pollen grain in angiosperm at the time of dehiscence from an anther could be 2-celled or 3-celled. Explain. How are the cells placed within the pollen grain when shed at a 2-celled stage? (CBSE 2017)
Answer:
In the majority of angiosperms, pollens are released in a 2-celled stage. However, in other species, the generative cell divides into two male gametes and forms a 3-celled stage. When the pollen grain is shed at the 2-celled stage, it has two unequal cells, a bigger vegetative cell and a smaller generative cell.

Question 19.
Do you think apomixis can be compared with asexual reproduction? Support your answer, giving one reason. How is apomixis beneficial to farmers? Explain. (CBSE Delhi 2018C)
Answer:
A few flowering plants such as some species of Asteraceae and grasses have evolved a special mechanism to produce seeds without fertilisation. This is called apomixis. So apomixis is a form of asexual reproduction.

Production of hybrid seeds is costly; if hybrids with desirable characteristics can be made into apomicts, there is no segregation of characters in the hybrid progeny. The farmer can continue using hybrid seeds year after year and need not to buy new seeds.

Sexual Reproduction in Flowering Plants Important Extra Questions Long Answer Type

Question 1.
Give an account of a structure of typical anther?
Answer:
Structure of anther. A typical angiosperm anther is bilobed with each lobe having two cells or theca, i.e. they are dithecous. Often a longitudinal groove runs lengthwise separating the theca. The bilobed nature of an anther is very distinct in the transverse section of the anther. The anther is a four-sided (tetragonal) structure consisting of four microsporangia located at the corners, two in each lobe.

The microsporangia develop further and become pollen sacs. They extend longitudinally all through the length of an anther and are packed with pollen grains.

Question 2.
Write about the structure of microsporangium? (CBSE 2014)
Answer:
A microsporangium is more often circular in outline and is surrounded by a four-layered wall:

  1. The outermost layer is the single layer of the epidermis.
  2. The second layer is endothecium, where cells develop thickenings.
  3. The middle layer is of 2-4 cells.
  4. Tapetum, the innermost layer, is of large diploid/polyploid and binucleate or multinucleate cells.
  5. There are four pollen sacs.
  6. Each pollen sac has sporogenous tissue.

Question 3.
Describe the development of microspore in angiosperms?
Answer:
Microsporogenesis and formation of pollen grains:

  1. Every cell of the sporogenous tissue is a potential pollen mother cell (PMC) and can give rise to microspore tetrad/ pollen grains.
  2. But some of them forego this potential and differentiate into the pollen mother cells of microspore mother cells.
  3. Each microspore mother cell undergoes meiosis to form a cluster of four haploid cells, called microspore tetrad.
    Class 12 Biology Important Questions Chapter 2 Sexual Reproduction in Flowering Plants 3
  4. As the anther matures, the microspores dissociate from the tetrad and develop into pollen grains.
  5. The tetrad may be tetrahedral, isobilateral, T-shaped, L-shaped and linear.
  6. The nucleus of the microspore undergoes mitosis to form large vegetative cell and a small spindle¬shaped generative cell that floats in the cytoplasm of the vegetative cell.

Question 4.
What is pollination? Name different types of pollination?
Answer:
Pollination: It is the transfer of pollen grains from the anther to stigma.Class 12 Biology Important Questions Chapter 2 Sexual Reproduction in Flowering Plants 4

Question 5.
With a neat diagram, explain the 7-celled, 8-nucleate mature female gametophyte? (CBSE Delhi 2014, Outside Delhi 2019)
Answer:
Structure of mature female gametophyte. The female gametophyte is formed from the contents of megaspore. However, it always remains embedded within the membrane of the megaspore. The mature female gametophyte is 7-celled (an egg, two synergids, three antipodals and a central cell).

Out of the seven cells of mature female gametophyte, the egg cell fuses with male gamete to form a zygote, one synergid degenerates and the other synergid directs the pollen tube entry. The central cell fuses with the male gamete to form triploid primary endosperm nucleus and the three antipodals degenerate before or after fertilisation.

They sometimes act as antipodal haustoria:
Class 12 Biology Important Questions Chapter 2 Sexual Reproduction in Flowering Plants 5

Question 6.
Tabulate the differences between self-pollination and cross-pollination?
Answer:
Differences between self-pollination and cross-pollination:

Self-pollination Cross-pollination
1. It is the transference of pollen grains from the anther to the stigma of the same flower. 1. It is the transference of pollen grains from the anther to the stigma of another flower of another plant.
2. Self-pollination always occurs through touch or wind. 2. Cross-pollination always occurs through an external agent.
3. Both anthers and stigmas mature simultaneously. 3. The anthers and stigmas mature at different times.
4. It can occur in closed flowers. 4. It occurs only when the flowers are open.
5. It gives rise to pure lines. 5. It gives rise to offspring having variations.

Question 7.
Briefly explain contrivances for self-pollination?
Answer:
Class 12 Biology Important Questions Chapter 2 Sexual Reproduction in Flowering Plants 6

Question 8.
What is bagging technique? How is it useful in plant breeding programmes?
Answer:
Bagging technique: It is a technique used during artificial hybridisation of plant breeding to ensure cross-pollination.

For this removal of anthers from the flower bud before the anther dehisces using a pair of forceps is necessary. This step is referred to as emasculation. Emasculated flowers have to be covered with a bag of suitable size, generally made up of butter paper, to prevent contamination of its stigma with unwanted pollen.

This process is called bagging. When the stigma of bagged flower attains receptivity, mature pollen grains collected from anthers of the male parent are dusted on the stigma, and the flowers are rebagged, and the fruits are allowed to develop.

If the female parent produces unisexual flowers, there is no need for emasculation. The female flower buds are bagged before the flowers open. When the stigma becomes receptive, pollination is carried out using the desired pollen and the flower rebagged.

Advantages of bagging technique:

  1. It is necessary for cross-breeding for the crop improvement programme.
  2. Commercially superior varieties can be obtained.

Question 9.
What are zoophily, zoophilous flowers and entomophilous flowers? List the characters of entomophilous flowers?
Answer:

  • Zoophily: Pollination by animals is called Zoophily.
  • Zoophilous flowers: The flowers which get pollinated with the aid of animals are called zoophilous flowers.
  • Entomophilous flowers: Insect pollinated flowers are called entomophilous flowers.

Characters of Entomophilous flowers:

  • The flowers are of large size.
  • Corolla of flowers is brightly coloured.
  • Some flowers are very fragrant.
  • They bear nectar-secreting glands.
  • The pollen grains are heavy with the rough and spiny surface.
  • The stigmatic surface is sticky.

Question 10.
What are the advantages of cross-pollination?
Answer:
Advantages of cross-pollination:

  1. It increases variability and certain new and useful characters appear in the offsprings.
  2. Sometimes harmful characters are eliminated.
  3. It leads to the production of new varieties.
  4. Several disease-resistant varieties of important crop plants have been produced as a result of cross-pollination.
  5. It increases the power of adaptation among the offsprings.
  6. The number of seed production can be increased among useful cereal crops.

Question 11.
Differentiate between anemophilous and entomophilous flower? (CBSE Delhi 2008 (S))
Answer:
Differences between anemophilous and entomophilous flowers:

Anemophilous flower Entomophilous flower
1. The flowers are small. 1. The flowers are either Large or if small the are grouped to form a mass.
2. Flowers are neither brightly coloured nor emit any odour. 2. The flowers are brightLy coLoured and emit odour.
3. The flowers are devoid of any nectar and edible pollen. 3. The flowers usually possess nectar or edible pollen.

Question 12.
How does the floral pattern of Mediterranean orchid Ophrys guarantee cross-pollination? (CBSE 2010, 2019)
Answer:
In many species of Mediterranean orchid Ophrys, the shape, colour marking and odour of flower are like the female moth Colpa.

The Ophrys employs sexual deceit to get pollination done by the Colpa. The male moth matures earlier than the female. It mistakes the Ophrys flower for female and tries to copulate. Thus it pollinates the flower.

Question 13.
Explain the events in the embryo sac during the process of fertilisation?
Answer:
Fertilisation. The fusion of male and female gametes is called fertilisation. The process was first discovered by Strasburger (1884).

Pollen tube enters through the micropyle and releases male gametes. This is called Porogamy. If pollen tube enters through chalaza, it is called chalazogamy and if pollen tube enters through integuments, it is called misogamy. One male gamete (1n) fuses with egg or oosphere to form oospore (2n) and second male gamete fuses with secondary nucleus (2n) to form the primary endosperm nucleus (3n).

In angiosperms where both male gametes are functional and bring about fertilisation of oosphere and secondary nucleus to form oospore and primary endosperm nucleus, is called double fertilisation.

Question 14.
What do you understand by double fertilisation? (CBSE Sample Paper) (CBSE Delhi (C) 2008)
Or
Where does triple fusion take place in a flowering plant? (CBSE 2010)
Answer:
In angiosperms both the male gametes are functional. Double fertilisation is a process in which the egg nucleus and secondary nucleus are fertilized at one time by the male gametic nuclei. The fusion of one male gamete nucleus (1n) with egg or oosphere (1n) is called syngamy and results in the formation of oospore (2n).

The second male gamete fuses with the secondary nucleus; it is called triple fusion which results in the formation of the primary endosperm nucleus (3n). Thus it takes place in the embryo sac. It was first discovered by Nawaschin (1898).

Question 15.
What is the fate of floral parts after fertilisation?
Answer:
After fertilisation, the transformation of parts of the flower is as follows:

Before fertilization After fertilization
1. Calyx, Corolla, Androecium, Style, Stigma Wither off
2. Ovary Fruit
3. Ovary wall Pericarp
4. Ovule Seed
5. Integuments Seed coats
6. Outer integument Testa
7. Inner integument Teg men
8. Micropyle Macrophyte
9. Funicte Stalk of seed
10. Nucellus (persistent) Pensperm
11. Egg cell Zygote (oospore)
12. Synergids Disintegrate and disappear

Question 16.
Write an account of various types of fruits?
Answer:
Types of fruits:
Class 12 Biology Important Questions Chapter 2 Sexual Reproduction in Flowering Plants 7

  • Simple fruit: Fruits develop from a single ovary.
  • Aggregate fruit: Develops from the polycarpellary, apocarpous ovary. Composite fruit. Which develops from the inflorescence.
  • Dry fruit: When mesocarp, epicarp and endocarp all are hard.
  • Succulent fruit: When mesocarp is pulpy.

Question 17.
Mention two strategies evolved by flowers to prevent self-pollination? (CBSE 2014)
Answer:
Contrivances favouring cross-pollination In many plants there are devices which completely or partially prevent self-pollination and encourage cross¬pollination.

  1. Unisexuality: The flowers are unisexual, i. e. stamens and carpels occur in different flowers. They may be present on the same plant, e.g. in maize or on different plants, e.g. in Papaya. The first condition is called monoecious and second is called dioecious.
  2. Dichogamy: When stamens and carpels of bisexual flowers mature at different times, it is called dichogamy. If stamens mature first it is called a protandrous condition, or if carpels mature first it is called protogynous condition.

Question 18.
Draw a monocot endospermic seed? (CBSE 2012, 2019)
Answer:
Seeds of angiosperm:
Class 12 Biology Important Questions Chapter 2 Sexual Reproduction in Flowering Plants 8
Seeds of angiosperm.

Question 19.
Give a brief account of post-pollination events? (CBSE Delhi 2016)
Answer:
Post-pollination events:

  1. The fusion of male and female gametes is called fertilisation. The process was first discovered by Strasburger (1884).
  2. Pollen tube enters through the micropyle and releases male gametes. This is called Porogamy. If pollen tube enters through chalaza, it is called chalazogamy and if pollen tube enters through integuments, it is called misogamy.
  3. One male gamete fuses with egg or oosphere to form oospore (2n) and second male gamete fuses with secondary nucleus (2n) to form the primary endosperm nucleus (3n).
  4. In angiosperms where both male gametes are functional and bring about fertilisation of oosphere and secondary nucleus to form oospore and primary endosperm nucleus, is called double fertilisation.
  5. Development of endosperm from the primary endosperm nucleus.
  6. Development of embryo from a zygote,
  7. Development of seed from the ovule.

Question 20.
(i) Can a plant flowering in Mumbai be pollinated by pollen grains of the same species growing in New Delhi? Provide explanations for your answer.
Answer:
Yes, it is possible by artificial hybridisation in which pollen grains of one flower is introduced artificially on the stigma of another flower. But there should not be self-incompatibility.

However, it is important to make sure that only the desired pollen grains are used for pollination and the stigma is protected from contamination (from unwanted pollen). This is achieved by emasculation and bagging techniques.

(ii) Draw the diagram of a pistil where pollination has successfully occurred. Label the parts involved in reaching the male gametes to its desired destination. (CBSE 2017)
Answer:
Class 12 Biology Important Questions Chapter 2 Sexual Reproduction in Flowering Plants 9

Question 21.
Differentiate between parthenocarpy and parthenogenesis. Give one example of each? (CBSE2018)
Answer:
Parthenocarpy is the process which produces fruits from unfertilised ovules in plants. Unfertilised ovules develop into fruits prior to fertilisation. Such fruits do not contain seeds.

However, parthenogenesis can be described as a process in which unfertilised ovum develops into an individual (virgin birth) without fertilisation. Therefore, it can be considered a method of asexual reproduction.

Such a type of reproduction is commonly shown by some invertebrates and lower plants, e.%. rotters, honeybees and some lizards and birds (turkey), etc.

Another key difference between parthenogenesis and parthenocarpy is parthenogenesis is shown by animals and plants while parthenocarpy is shown by plants only.

Question 22.
The embryo sac in female gametophyte is seven celled and eight nucleated structure. Justify the statement with the help of a labelled diagram? (CBSE Sample Paper 2018-19)
Answer:
A typical embryo sac has three cells that are grouped together at the micropylar end and constitute the egg apparatus. The egg apparatus, in turn, consists of two synergids and one egg cell.

Three cells are at the chalazal end and are cast the antipodals. The Large central cell has two polar nuclei.

Thus, a typical angiosperm embryo sac at maturity is 7-celled and 8-nucLeated.
Class 12 Biology Important Questions Chapter 2 Sexual Reproduction in Flowering Plants 10

Question 23.
List the changes that occur when an ovule matures into a seed? (CBSE Sample Paper 2018-19)
Answer:

  1. Integuments of ovules harden and become tough protective seed coats.
  2. The micropyle remains as a small pore in the seed coat.
  3. As the seed matures, its water content is reduced.
  4. Seeds become relatively dry. It has 10-15 percent moisture by mass.
  5. The general metabolic activity of the embryo slows down.
  6. The embryo may enter a state of inactivity called dormancy.

Question 24.
What are the various adaptations or contrivances met within angiosperms flowers which favour cross-pollination?
Answer:
Contrivances favouring cross-pollination: In many plants, there are devices which completely or partially prevent self-pollination and encourage cross-pollination.
1. Unisexuality: The flowers are unisexual, i.e. stamens and carpels occur in different flowers. They may be present on the same plant, e.g. in Three cells are at the chalazal end and are called the antipodals. The large central cell has two polar nuclei.

Thus, a typical angiosperm embryo sac at maturity is 7-celled and 8-nucleated.

Structure of typical embryo sac.

maize or on different plants e.g. in Papaya. The first condition is called monoecious and second is called dioecious. In monoecious plants, both self-and cross-pollination can occur but mostly there is cross-pollination as sex organs mature at different times. In dioecious only cross-pollination occurs.

2. Dichogamy: When stamens and carpels of bisexual flowers mature at different times it is called dichogamy.

If stamens mature first, it is called a protandrous condition, or if carpels mature first, it is called protogynous condition. Dichogamy prevents self-pollination and ensures cross-pollination.

3. Self-sterility: In orchids, pollen grains of one flower does not germinate on the stigma of the same flower. This is called self-sterility but pollen grain germinates on the stigma of some other plant and thus ensures cross pollination.

4. Heterostyly: In primrose, there are different lengths of styles which ensure cross-pollination.

5. Herkogamy (Herko-barrier): In bisexual flowers, e.g. in Salvia and pea, peculiarities of petals ensure insect pollination, thus aids in cross-pollination.

Question 25.
With a neat labelled diagram, describe the parts of a typical angiosperm ovule? (CBSE Delhi 2014)
Answer:
Structure of ovule. Each ovule consists of nucellus surrounded by two integuments and a stalk or funiculus.

  1. Funiculus is a stalk-like structure by which ovule is attached to the placenta.
    Class 12 Biology Important Questions Chapter 2 Sexual Reproduction in Flowering Plants 11
    v.s. of the mature ovule to show various parts.
  2. Hilum: It is the point of attachment of the body of the ovule with the funiculus.
  3. Raphe: It is the longitudinal ridge formed by the lengthwise fusion of funiculus with the body of ovule in a typical anatropous ovule.
  4. Nucellus: It is a mass of diploid cell called megasporangium. It provides nourishment in the development of embryo sac.
  5. Embryo sac: It is the female gametophyte which contains the egg apparatus.
  6. Integuments from seed coats, i.e. testa and tegmen.
  7. Micropyle: It is a small opening which is left by the integument in the ovule for the passage of pollen tube into the ovule.
  8. Chalaza: The basal region of the ovule where the funiculus is attached is termed chalaza.

Question 26.
Describe the structure of typical embryo sac and the functions performed by its various constituents?
Or
Draw a schematic labelled diagram of fertilized embryo sac of angiosperm? (CBSE 2008, 2012, 2013)
Answer:
In the mature ovule, within the nucellus towards the micropylar end is a large cell called embryo sac. It is also called a female gametophyte. It contains cytoplasm, two free nuclei and six small cells. Two free nuclei fuse to form a secondary nucleus.

Out of these, three cells constitute egg apparatus (two synergids and one oosphere or egg cell). The other three cells lie at the chalazal end of embryo sac and are called antipodal cells. They are small and enclosed in thin cell walls.

In egg apparatus, egg fuses with male gamete to form zygote or oospore which later gives rise to the embryo. Synergids help in fertilisation. Secondary nucleus fuses with the second male gamete to form primary endosperm nucleus which later on forms endosperm.

The three antipodal cells degenerate:
A typical embryo sac has three cells that are grouped together at the micropylar end and constitute the egg apparatus. The egg apparatus, in turn, consists of two synergids and one egg cell.

Three ceLLs are at the chaLazaL end and are cast the antipodals. The Large centraL ceLL has two poLar nucLei.

Thus, a typical angiosperm embryo sac at maturity is 7-celLed and 8-nucLeated.
Class 12 Biology Important Questions Chapter 2 Sexual Reproduction in Flowering Plants 10

Question 27.
Describe the structure of typical microspore (pollen grain)?
Or
Name the organic material of exine and intine of an angiospermous pollen grain. Mention their role. (CBSE Delhi 2014) Answer:
Structure of pollen grains:

  1. Pollen grains are normally spherical in outline.
  2. Each pollen grain has a prominent two-layered wall.
  3. The outer layer is called exine and the inner layer is called intine.
  4. The exine is hard and made of sporopollenin. It shows a variety of architecture as per the characteristic of each species.
    Class 12 Biology Important Questions Chapter 2 Sexual Reproduction in Flowering Plants 12
  5. Intine is thin and it is made of cellulose and pectin.
  6. At certain places, exine is either absent or very thin and such places are called germ pores. In monocots, there are germinal furrows.
  7. A mature pollen grain has two cells-a vegetative cells and a generative cell.
  8. The vegetative cell is larger, has abundant reserve food and a large irregular-shaped nucleus.
  9. The generative cell is small, spindle¬shaped and floats in the cytoplasm of the vegetative cell.

Question 28.
Draw a well-labelled diagram of T.S. of the anther? (CBSE Sample paper 2009)
Or
Draw a labelled schematic diagram of the transverse section of a mature anther of an angiosperm plant. (CBSE Delhi 2013)

Answer:
T. S. of anther
Class 12 Biology Important Questions Chapter 2 Sexual Reproduction in Flowering Plants 13
(A) Transverse section of a tetra eusporangiate anther to show its various tissue;
(B) Enlarged view of one microsporangium showing wall layers.

Question 29.
Explain the development of the female gametophyte. Illustrate the answer with suitable diagrams? (CBSE Delhi 2019, Outside Delhi 2019)
Answer:
Development of female gametophyte:

  1. The functional megaspore is the first cell of the female gametophyte of angiosperms.
  2. It enlarges to form the female gametophyte, also called embryo sac.
  3. Its nucleus undergoes a mitotic division and the two nuclei move to the opposite poles, forming the 2-nucleate embryo sac.
  4. Two successive mitotic divisions in each of these two nuclei result in the formation of an 8-nucleate embryo sac.
    Class 12 Biology Important Questions Chapter 2 Sexual Reproduction in Flowering Plants 14
    Fig. Development of embryo sac.
  5. Cell wall formation starts at the eight-nucleate stage, resulting in the formation of a typical female gametophyte.
  6. Three cells are grouped together at the micropylar end to form the egg apparatus, consisting of two synergids and a female gamete/egg cell.
  7. Three cells are grouped together at the chalazal end. They are called antipodal cells.
  8. The remaining two nuclei are called polar nuclei. They move to the centre of the embryo sac (now called a central cell) and fuse to form a diploid secondary nucleus.
  9. Thus, a typical angiosperm embryo sac is 8-nucleate and 7-celled.

Question 30.
Show the germination of pollen grain with diagrams only? (CBSE 2011, 2019 C)
Answer:
Germination of Pollen Grain:
Class 12 Biology Important Questions Chapter 2 Sexual Reproduction in Flowering Plants 15
Germination of pollen grain and formation of male gametophyte in an angiosperm.

Question 31.
What do you mean by the development of the embryo? Support the answer with a diagram? (CBSE Delhi 2014)
Answer:
Development of embryo.
1. The zygote or oospore divides by a transverse wall into an upper suspensor cell and lowers embryonal cell.

2. The suspensor cell which lies towards the micropylar end divides by transverse divisions to constitute 7-10 cells called suspensor.

3. The upper cell of the suspensor filament towards the micropylar end is called a haustorial cell, whereas the cell lying above the embryo cell is called hypophysis.

4. The haustorial cell enlarges in size and attaches the suspensor to the tip of embryo sac.

5. The embryonal cell divides by second longitudinal division at a right angle to the first and then by transverse division to form an octant or eight- celled embryo.

6. Out of these eight cells, the lower four cells of octant away from the suspensor give rise to the plumule and the two cotyledons, while the above four cells of octant near the suspensor from the hypocotyl and stele of the radicle.

7. Then this octant divides by a periclinal division to form outer single-layered dermatogen from which arises the epidermis layer. The inner cells further divide to form periblem below the dermatogen and the central plerome.

8. The periblem forms the cortex while the plerome gives stele of the embryo.

9. The lowermost cell of suspensor, which is lying just above the octant cells is known as hypophysis. The hypophysis divides to give rise to the dermatogen and periblem of the radicle.

10. A fully developed embryo of dicotyledons has an embryonal axis differentiated into plumule, two cotyledons and radicle.

11. In the beginning, the embryo is globular. When two cotyledons differentiate from the sides with faint plumule in the centre, the embryo becomes heart¬shaped. Then the embryo undergoes rest and ovule is transformed into the seed.

Monocot embryo: Unlike the dicots where the embryonal mass is formed of eight cells, the anterior cells forming the plumule and the cotyledons, and the posterior forming hypocotyl, the development is much variable in monocots, i.e. grass family. In some cases, suspensor does not develop at all. Only one cotyledon appears in monocots as a terminal structure. The plumule always appears laterally from it.

Question 32.
Draw the stages of development of the embryo in a dicot angiosperm?
Answer:
Stages of embryo development in a dicot angiosperm:
Class 12 Biology Important Questions Chapter 2 Sexual Reproduction in Flowering Plants 16
Stages of embryo development in a dicot angiosperm.

Question 33.
Explain the role of tapetum in the pollen grain wall formation?
Answer:
Role of tapetum in pollen grain wall formation. During microsporogenesis, the cells of tapetum provide various enzymes, hormones, amino acids and other nutritive materials to the dividing microsporocytes.

The main functions of tapetum are:

  1. Transportation of nutrients into anther locule at the time of meiosis in spore mother cells.
  2. Secretion of enzymes and hormones.
  3. Production of Ubisch bodies which are coated with sporopollenin to cause thickening of the exine.
  4. Secretion of any oily material (Pollenkitt) outside of mature pollen.
  5. Secretion of special proteins for pollen to recognise compatibility.

Question 34.
Describe in brief the various special modes of reproduction? Mention their economic significance?
Answer:
Special modes of reproduction:
1. Apomixis (Agamospermy): It is a modified form of reproduction in which seeds are formed without fusion of gametes. It includes the process whereby a diploid cell of nucellus develops into an embryo, giving a diploid seed with a genetic constitution identical to the parent. The organism that reproduces by apomixis is called apomict.

2. Parthenocarpy: In certain plants, the fruits are formed without the act of fertilisation. Such fruits are called parthenocarpic fruits and the phenomenon is known as parthenocarpy. Parthenocarpic fruits are either seedless or contain empty or non-viable seeds. In these fruits, the stimulus (“seed factor”) for fruit growth is provided by the tissue of the ovary wall itself. Seedless varieties of grapes and oranges contain those of the seeded varieties. Most commonly cultivated varieties of banana and pineapple are parthenocarpic.

3. Polyembryony: It is the formation of more than one embryo in the seed. Onion, groundnut, mango, lemon and oranges are some of the examples. Polyembryony is very common among conifers.

4. Sporophytic budding: It may occur in the nucellus and integuments, resulting in the development of an embryo. Examples: Orange, mango, onion, etc.

5. Micropropagation: In this method, tissue and organ culture is utilised. Tissue or organs grow on suitable medium containing hormones. Tissue proliferates to form a callus. From this callus, arise new plantlets. Each plantlet, when transferred to pot or soil, produces a new plant. Thus by this method, an indefinite number of plants can be raised from a small mass of partial tissue.

6. Plant tissue culture. It also helps in propagating plants to a great extent.

Question 35.
Fertilisation is essential for the production of seed, but in some angiosperms, seeds develop without fertilisation?
(i) Give an example of an angiosperm that produces seeds without fertilisation. Name the process.
Answer:
(a) Species of Asteraceae and grass.
(b) Process of formation of seeds without fertilization is called apomixis.

(if) Explain the two ways by which seeds develop without fertilisation. (CBSE 2009)
Answer:
Apomixis is a form of asexual reproduction that mimics sexual reproduction:
(a) The diploid egg cell is formed without reduction division and develop into an embryo without fertilisation.
(b) In some varieties of citrus and mango, some of the nucellar cells surrounding the embryo sac start dividing, produce embryo sac and develop into embryos. In such species, each ovule contains many embryones. Such a process is called polyembryony.

Question 36.
Write a note on the development of endosperm. Mention the types with examples?
Answer:
Development of endosperm: Endosperm develops after double fertilisation in angiosperms. It is triploid and develops from the primary endosperm nucleus.

In angiosperms three types of endosperm development are observed:
1. Nuclear endosperm. In nuclear endosperm development, the nucleus divides by free nuclear division, there is the appearance of the central vacuole, all the nuclei are pushed towards periphery then starts cytokinesis and results in the formation of cells but in coconut, vacuole persists which is filled with fluid.

2. Cellular endosperm. In cellular endosperm, each and every division is followed by cytokinesis, so endosperm is cellular from the very beginning, e.g. Datura, Petunia.

3. Helobial endosperm. It is intermediate between cellular and nuclear endosperm. The first division is followed by cytokinesis to form two unequal cells, micropylar cell and chalazal cell. Further divisions in each cell are free nuclear divisions. This type of endosperm is found in order Helobiales.
Class 12 Biology Important Questions Chapter 2 Sexual Reproduction in Flowering Plants 17
Types of endosperm development.

Question 37.
What are the advantages of seeds to plants and mankind? (CBSE Delhi 2014)
Answer:
1. Advantages of seeds to plants:
(a) Seeds have better adaptive strategies for dispersal to new habitats for better survival.
(b) Seeds have reserve food materials to nourish the seedlings during seed germination.
(c) The hard seed coat provides protection to the young embryo.
(d) Since seeds are formed after sexual reproduction, they show genetic recombinations leading to variations.
(e) Since seed formation does not depend on water (for fertilisation), their formation is more certain and they are the units of multiplication and continuity of the species.

2. Advantages of seeds to mankind:
(a) Seeds are stored and used as food throughout the year.
(b) Seeds are also used to raise the crop in the favourable seasons depending on their viability.

Question 38.
(i) Draw a diagrammatic sketch of a transverse section of an anther of an angiosperm. Label its different walls and the tissue forming microspore mother cells?
Answer:
T. S. of anther
Class 12 Biology Important Questions Chapter 2 Sexual Reproduction in Flowering Plants 13
(A) Transverse section of a tetra eusporangiate anther to show its various tissue;
(B) Enlarged view of one microsporangium showing wall layers.

(ii) Describe the process of microsporogenesis up to the formation of a microspore?
Answer:
Microsporogenesis and formation of pollen grains:

  1. Every cell of the sporogenous tissue is a potential pollen mother cell (PMC) and can give rise to microspore tetrad/ pollen grains.
  2. But some of them forego this potential and differentiate into the pollen mother cells of microspore mother cells.
  3. Each microspore mother cell undergoes meiosis to form a cluster of four haploid cells, called microspore tetrad.
    Class 12 Biology Important Questions Chapter 2 Sexual Reproduction in Flowering Plants 3
  4. As the anther matures, the microspores dissociate from the tetrad and develop into pollen grains.
  5. The tetrad may be tetrahedral, isobilateral, T-shaped, L-shaped and linear.
  6. The nucleus of the microspore undergoes mitosis to form large vegetative cell and a small spindle¬shaped generative cell that floats in the cytoplasm of the vegetative cell.

(iii) Write the function of ‘germ pore’ in a pollen grain of an angiosperm? (CBSE Delhi 2018C)
Answer:
Germ pores allow the germinating of pollen tube with contents of the male gametes and vegetative cell to come out of the pollen grains.

Question 39.
(i) When a seed of orange is squeezed, many embryos instead of one are observed. Explain how it is possible?
Answer:
When a seed of orange is squeezed, many embryos instead of one are observed. It is due to polyembryony. Polyembryony is the occurrence of more than one embryo in a seed.

The nucellar cells, synergid or integument cells develop into a number of embryos of different sizes in orange.
Sometimes the formation of more than one egg in an embryo sac can lead to polyembryony.

(ii) Are these embryos genetically similar or different? Comment.
Answer:
These embryos are genetically similar because in such embryos parental characters are maintained. In this process, there is no segregation of characters in the offspring.

Question 40.
(i) Describe any two devices in a flowering plant which prevent both autogamy and geitonogamy?
(ii) Explain the events up to double fertilisation after the pollen tube enters one of the synergids in an ovule of an angiosperm? (CBSE 2018)
Or
Explain the post-pollination events up to double fertilization that occurs in angiosperm. (CBSE 2019 (c))
Answer:
(i) Autogamy: It is the transfer of pollen grains from the anther to the stigma of the same flower.

Geitonogamy: It is the transfer of pollen grains from the anther to the stigma of another flower of the same plant.

Two devices that prevent both autogamy and geitonogamy are:
(a) Self-incompatibility: This is a genetic mechanism which prevents self pollen from fertilising the ovules by inhibiting pollen germination or growth of pollen tube in the pistil.
(b) Dioecious plants: Male and female flowers are present on different plants.

(ii) The events seen after the pollen tube enters one of the synergids in an ovule is as follows:
(a) After reaching the ovary, pollen tube enters the ovule through the micropyle and thus enters one of the synergids through filiform apparatus.

(b) After entering one of the synergids, the pollen tube releases the two male gametes into the cytoplasm of the synergid.

(c) One of the male gametes moves towards the egg cell and fuses with its nucleus and results in the formation of the zygote (diploid cell). This is called syngamy.

(d) The other male gamete moves towards the two polar nuclei present in the central cell and fuses to form triploid primary endosperm nucleus (PEN) i.e., there is a fusion of three haploid nuclei and hence termed as triple fusion.

(e) Two types of fusions-syngamy and triple fusion-take place in an embryo sac. Hence the phenomenon is termed double fertilisation.

(f) After fertilisation, the primary endosperm nucleus becomes the primary endosperm cell (PEC) and develops into endosperm while zygote develops into an embryo.
Class 12 Biology Important Questions Chapter 2 Sexual Reproduction in Flowering Plants 18

Fig. (A) EnLarged view of an egg apparatus showing entry of poLLen tube into a synergy, (B) Discharge of male gametes into a synergid and the movements of the sperms, one Into the egg and the other Into the centraL celL.

Very Important Figures

Class 12 Biology Important Questions Chapter 2 Sexual Reproduction in Flowering Plants 19

Evolution Class 12 Important Extra Questions Biology Chapter 7

Here we are providing Class 12 Biology Important Extra Questions and Answers Chapter 7 Evolution. Important Questions for Class 12 Biology are the best resource for students which helps in Class 12 board exams.

Class 12 Biology Chapter 7 Important Extra Questions Evolution

Evolution Important Extra Questions Very Short Answer Type

Question 1.
When did life appear on earth?
Answer:
Life originated sometimes 3600 million years ago.

Question 2.
Arrange the following substances in a proper sequence with regard to the formation of chemical constituents at the time of origin of life: Sugar, methane, nucleic acid, and amino acid.
Answer:
Methane-sugar-amino acid-nucleic acid.

Question 3.
Mention the type of nutrition in the cells that originated first during the origin of life.
Answer:
Heterotrophic nutrition.

Question 4.
Which group of organisms were responsible for the appearance of free oxygen in the atmosphere of the primitive earth?
Answer:
Oxygenic photosynthetic bacteria resembling the present-day blue-green algae or cyanobacteria.

Question 5.
Give the three key factors of the modern concept of evolution.
Answer:
Genetic variations, natural selection, and isolation.

Question 6.
What is the ultimate source of organic variation?
Answer:
Genetic variations due to mutations, recombination, and polyploidy.

Question 7.
What is proved by the phenomenon of resistance to DDT in mosquitoes?
Answer:
Natural selection.

Question 8.
Attempt giving a clear definition of the term species.
Answer:
The members of a species that resemble structurally and functionally are able to interbreed freely and produce fertile offspring of their own kind and share a common gene pool.

Question 9.
What causes speciation according to Hugo de Vries? (CBSE Delhi 2008)
Answer:
Mutations.

Question 10.
How did Charles Darwin express ‘fitness’? (CBSE Delhi 2019)
Answer:
Charles Darwin expressed ‘fitness’ as reproductive fitness. The adaptive ability of an organism is inherited. The best-adapted species increase their population by reproduction.

Question 11.
Name the scientist who disproved spontaneous generation theory. (CBSE 2010)
Answer:
Louis Pasteur.

Question 12.
Mention the type of evolution that has brought the similarity as seen in potato tuber and sweet potato. (CBSE 2009)
Answer:
Convergent evolution.

Question 13.
Briefly explain the theory of biogenesis. (CBSE 2012)
Answer:
According to this theory, life originated from pre-existing life forms.

Question 14.
Write the probable difference in eating habits of Homo habilis and Homo erectus. (CBSE Outside Delhi 2016)
Answer:
Homo habilis was carnivorous. Homo erectus was omnivorous.

Question 15.
How do we compute the age of a living tree?
Answer:
By determining the number of annual rings in the lower part of its trunk, called dendrochronology.

Evolution Important Extra Questions Short Answer Type

Question 1.
What are we referring to when we say “simpler organisms” or “complex organisms”?
Answer:
By “simpler organisms”, we mean those organisms which are primitive with simple organization and simple metabolic pathways, structural and functional. “By complex organisms”, we mean those organisms which are more evolved, have a complex level of structural and functional organization and complex metabolic pathways.

Question 2.
Louis Pasteur’s experiments, if you recall, proved that life can arise from only pre-existing life. Can we correct this as life evolves from pre-existent life or otherwise we will never answer the question as to how the first forms of life arose? Comment.
Or
State the two principal outcomes of the experiments conducted by Louis Pasteur on the origin of life. (CBSE Delhi 2019)
Answer:

  1. Dismissed theory of spontaneous generation of life.
  2. Yes, we can correct this as life evolves from pre-existent life.

The first life that appeared on the earth was apparently the result of chemical evolution, i.e. the life originated from inorganic molecules that formed organic molecules, further forming complex compounds.

This finally resulted in simple cells and then simple organisms, where-in complexity developed with time. However, once life originated, abiogenesis could not follow, and hence, life evolved further only through biogenesis, i.e. pre-existent life gave rise to new life.

Question 3.
What is convergent evolution?
Answer:
Convergent evolution. When adaptation for survival in similar habitats is similar, then this form of evolution is called convergent evolution or parallel evolution. Examples: Some of the marsupials of Australia resemble placental mammals that live in similar habitats to other continents. Australia separated about 50 million years ago. Marsupials arrived here before the separation from Antarctica and evolved in isolation earlier than placental mammals.

Question 4.
What are analogous organs? Give examples. (CBSE 2016)
Answer:
Analogous organs: The organs which are similar in appearance and perform the same function but differ in their fundamental structure and origin are called analogous organs.

Examples:

  1. Wings of birds and insects.
  2. Leaves of a plant and cladodes of Ruscus are also analogous organs.

Question 5.
What are vestigial organs? Give examples.
Answer:
Vestigial organs are non-functional organs in an organism that are functional in related animals and were functional in the ancestors. There are 90 vestigial organs in the human body and mainly include coccyx (tail bone); nictitating membrane (3rd eyelid); caecum and vermiform appendix; canines; wisdom teeth; body hair; auricular muscles; mammary glands in male; etc. Vestigial organs are also present in some other animals, e.g. splint bones in the horse; hind-limbs and pelvic girdles in python; wings and feathers in flightless birds; etc.

Question 6.
Write the significance of vestigial organs.
Answer:
Significance of vestigial organs. Organic evolution states that these vestigial organs were functional in the ancestral forms but have become non-functional due to changes in their function and may finally disappear. So the presence of vestigial organs is convincing evidence of organic evolution and is supported by Lamarck’s theory of use and disuse of organs.

Question 7.
What are fossils?
Answer:
Fossils. The remnants or impressions of living organisms from the remote past may be regarded as fossils. Fossil formation involves the burial of dead organisms. Layer after layer of sediment is laid above the dead organisms. It is only on this account that the oldest fossils are found in the deepest layers of the earth’s surface, while the fossils of recent origin are found in the upper layers. Thus by examining the various layers of the earth from the deepest to the most superficial strata for the fossil record, the story of life in the correct historical sequence can be known.

Question 8.
Why is Archaeopteryx called a connecting link between reptiles and birds?
Or
What is the significance of Archaeopteryx in the study of organic evolution?
Answer:
Importance of Archaeopteryx as connecting link. The fossil forms representing the characters of the two distinct groups of living animals are known as missing links. The best example of connecting link is afforded by a fossil bird, Archaeopteryx. It was the size of a crow. Archaeopteryx is decidedly a bird as it has feathers and a beak. But like reptiles, it has a long tail, jaws full of teeth, claws on forefingers, and keelless sternum. Thus it represents a stage between reptiles and birds through Archaeopteryx-like intermediate form.

Question 9.
What is adaptive radiation? (CBSE Delhi 2016)
Or
Describe one example of adaptive radiations. (CBSE Delhi 2008 S, 2010, 2015, 2019 C, Outside Delhi 2014)
Or
How did Darwin explain the existence of different varieties of finches on the Galapagos Islands? (CBSE Outside Delhi 2009, Delhi 2010)
Answer:
Adaptive radiation: It is the process of divergent evolution in which members of the same ancestral species of a large taxonomic group are evolved along different lines in different habitats of the same geographical area.

Example: Darwin’s Finches are an example of adaptive radiation.

There were many varieties of small blackbirds in the Galapagos Islands. Darwin reasoned that after originating from a common seed-eating stock, the finches must have radiated to different geographical locations in the same island and undergone adaptive changes, especially in the type of beak. Living in isolation for long, the new kinds of finches emerged that could survive and function in the new habitats.

Question 10.
List the main points of Lamarck’s theory.
Answer:
Lamarck’s theory of evolution:

  1. Effect of environment.
  2. Effect of use and disuse of organs.
  3. The inheritance of acquired characters.
  4. The origin of new species.

Question 11.
Give the main points of Darwin’s Theory of Evolution. (CBSE Outside Delhi 2019)
Answer:
Darwin’s Theory of Evolution may be summed up as follows:

  1. Rapid multiplication/overproduction
  2. Struggle for existence.
  3. Variations.
  4. Natural selection or survival of the fittest.
  5. Inheritance of useful variations.
  6. Origin of new species.

Alfred Wallace also arrived at the same conclusion as that of Charles Darwin.

Question 12.
Name the following:
(i) Who conceived the idea of the chemosynthetic hypothesis of the origin of life on earth?
Answer:
Oparin and Haldane

(ii) Who proved that spontaneous generation does not occur?
Answer:
Francesco Redi, Spallanzani, and Louis Pasteur

(iii) Who experimentally proved that life develops from pre-existing life only?
Answer:
Pasteur

(iv) Who gave the theory of organic evolution? (CBSE 2010)
Answer:
Charles Darwin.

Question 13.
Describe De Vries Mutation theory. (CBSE Delhi 2011)
Answer:
De Vries Mutation Theory: Hugo de Vries (1848-1935) was a Dutch Botanist. He performed experiments on the Evening Primrose (Oenothera Lamarckian). According to this theory, new species arise suddenly showing abrupt deviations in characters from the normal forms. These sudden deviations are because of mutation. Thus evolution is not a slow and gradual process but a sudden discontinuous and jerky process.

Question 14.
State a reason for the increased population of dark-colored moths coinciding with the loss of lichens (on tree barks/during industrialization period in England). (CBSE Delhi 2015)
Answer:
Soot evolved from coal-based industries deposited on tree bark of oak plants and darkened it, which is called industrial melanism. Dark-colored moth (Bistort Carbonaria) had more chances of survival so increased in number. Before industrialization, white-colored thick lichens grew on tree barks. In that background, white-winged moths survived but dark moths were picked out by predators.

But post-industrialization, lichens disappeared and tree trunks became dark due to the deposition of soot evolved from coal-based industries. Now white moths became easy prey compared to dark ones. Thus dark moths survived and increased their population.

Question 15.
Select the homologous structures from the combinations given below:
1. Fore-limbs of whale and bats
2. A tuber of potato and sweet potato
3. Eyes of Octopus and Mammals
4. Thorn of Bougainvillea and Tendril of Cucurbita. (CBSE Outside Delhi 2015)
Answer:

  1. Because both of them share similarities in the pattern of bones of forelimbs, though they perform different functions like forelimb of whale helps in swimming, while that in bats it helps in flying.
  2. Because both are modified stem branches (axillary buds) but are differently modified to perform different functions, e.g. thorns for protection from grazing animals and tendrils for climbing.

Question 16.
According to Hardy-Weinberg’s principle, the allele frequency of a population remains constant. How do you interpret the change of frequency of alleles in a population? (CBSE 2009)
Answer:
Disturbances in genetic equilibrium or Hardy-Weinberg equilibrium lead to a change of frequency of alleles in a population which results in evolution.

Question 17.
With the help of two suitable examples, explain the effect of anthropogenic actions on organic evolution. (CBSE Delhi 2013)
Answer:
Effect of anthropogenic actions on organic evolution:

  1. Creation of breeds by artificial or selective breeding programs.
  2. (a) Excess use of herbicides, pesticides, etc. has resulted in the selection of resistant varieties in a much lesser time.
    (b) Selection of drug-resistant microbes.
  3. Survival of dark-winged (melanized) moths after industrialization in England.

Question 18.
Can we call human evolution adaptive radiation?
Answer:
No, we cannot call human evolution adaptive radiation. It is the case of descent with modification in which more advanced types are evolved from the simple forms. In the case of human evolution, there appears to be parallel evolution of the human brain and language.

Question 19.
State two postulates of Oparin and Haldane with reference to the origin of life. (CBSE Delhi 2017)
Answer:
Oparin and Haldane proposed that:

  1. life originated from pre-existing non¬living organic molecules.
  2. the diverse organic molecules were formed from inorganic constituents by chemical evolution.

Question 20.
Write the names of the following:
(i) A 15 mya primate that was ape-like
Answer:
Dryopithecus

(ii) A 2 mya primate that lives in East African grasslands (CBSE Delhi 2018)
Answer:
Australopithecus

Question 21.
(i) Write two differences between Homo erectus and Homo habilis.
Answer:
Differences between Homo erectus and Homo habilis:

Homo erectus Homo habilis
1. Brain capacity is 900 cc. 1. Brain capacity is 900 cc.
2. Probably ate meat. 2. Probably ate meat.

(ii) Rearrange the following from early to late geologic periods: Carboniferous, Silurian, Jurassic. (CBSE Delhi 2019)
Answer:
Silurian, Carboniferous, Jurassic.

Evolution Important Extra Questions Long Answer Type

Question 1.
What are homologous organs? Give similar or different functions are catted examples. (CBSE 2016) homologous organs.
Answer:
Homologous organs: Organs that have a common origin, embryonic development, and the same fundamental structure but perform similar or different functions are catted homologous organs.

Examples of homologous organs:
1. The wings of bird and bat, flipper embryonic development, and same (fin) of whale and human forearm are fundamental structures but perform differently in forms because these have to perform different functions. Studies of the bones forming the skeleton of these organs would reveaL similarity in construction. In fact, these are the forms of forearms that have originated from pentadactyl forms and due to the different functions they are performing, they transformed into different forms.

2. In plants, the homologous organs may be a thorn of Bougainvillea or a tendriL of Cucurbita both arising in axillary position. Both have different forms depending on their function to perform.

Question 2.
How has the study of fossils helped in convincing scientists that organisms have come into existence through evolution? (CBSE Outside Delhi 2019)
Answer:
Fossils are important for man because of many reasons:

  1. They provide evidence of past life.
  2. They furnish direct and most convincing proofs in favor of organic evolution.
  3. They afford some information of ancient environment and climate.
  4. The most primitive forms of life are in the oldest rocks.
  5. Ancient forms were simpler than those found today.
  6. None of the plants and animals of the past were exactly similar to those found today.
  7. A complete fossil record has been found in the evolution of horses.

Question 3.
Explain antibiotic resistance observed in light of Darwinian selection theory.
Answer:
Antibiotics were considered to be very effective against diseases caused by bacteria. But within two or three years of the introduction of antibiotics, new antibiotic-resistant bacteria appeared in the population. Sometimes a bacterial population happens to contain one or a few bacteria having mutations that make them resistant to the antibiotic. Such resistant bacteria survive and multiply quickly as the competing bacteria have died.

Soon the resistance-providing genes become widespread and the entire bacterial population becomes resistant. Some hospitals harbor antibiotic-resistant bacteria due to the extensive use of antibiotics.

Question 4.
How does natural selection operate according to Darwin’s theory of natural selection? (CBSE Delhi 2016, 2019 C)
Answer:
Natural selection operates in the following ways according to Darwin’s theory:

  1. All plants and animals reproduce in a geometrical pattern. The number of organisms produced is much more than that can survive due to limited space and food.
  2. Due to the same basic requirement, competition between organisms takes place and those who are better adapted to the environment survive while the rest die.
  3. The individuals having useful variation overpower those without such variations. These variations are transmitted to future generations.

Question 5.
Distinguish between microevolution and macroevolution. Narrate the significance of population genetics in evolution.
Answer:
Evolution on the grand scale of geological time is called macroevolution, while evolution at the genetic level is microevolution. Microevolution is actually operative at genetic level change within a population. Significance of population genetics.

The gene frequency of a population is called population genetics. Evolution occurs within populations as the relative frequencies of different variations of DNA change over time. If genes change, then enzymes automatically change and represent two different forms of individuals and definitely result in evolution.

Question 6.
Compare and contrast the theories of evolution proposed by Darwin and Hugo De Vries. (CBSE Sample Paper 2018-19, Outside Delhi 2019)
Answer:

Darwin’s Theory of Natural Selection

De Vries Theory of Mutation

1. Minor variations cause evolution. 1. Mutation causes evolution.
2. Darwinian variations are small and directional. 2. Mutations are random and directionless.
3. Evolution is gradual. 3. Sudden mutations cause evolution.

Question 7.
How would the gene flow or genetic drift affect the population in which either of them happens to take place? (CBSE Delhi 2019)
Answer:
Gene flow is the transfer of genetic variations from one population to another. As a result of gene flow, the gene frequencies change in the original as well as in the new population. New genes or alleles are added to the new population and lost from the old population. If such a change in genes/alleles happens by chance, it is called genetic drift. In this case, the allelic frequency of the population will be affected. Such changes in allelic or gene frequencies lead to evolution, speciation, or founder effect.

Question 8.
Using the Internet and discussing with your teacher, trace the evolutionary stages of any one animal, say, a horse.
Answer:
The major evolutionary trend of horses:

  1. General increase (with occasional decrease) in size.
  2. The progressive loss of toes.
  3. Lengthening of toes that are retained.
  4. Lengthening of limbs in general.
  5. Enlargement of the brain (especially cerebral hemisphere).
  6. Increase in height.
  7. Increase in the complexity of molar teeth and an enlargement of the last two and, eventually, the last three premolars until they came to resemble molars.

Question 9.
Summarise Milter’s simulation experiment for organic synthesis. Comment on its efficacy. (CBSE Delhi 2012)
Answer:
Miller’s experiment. Milter (1953) made the first successful simulation experiment to assess the validity of the claim for the origin of organic molecules. Miller sealed in a spark chamber a mixture of water, methane, ammonia, and hydrogen gas. He made arrangements for boiling water.

The trap in turn was connected with the flask for boiling water. After 18 days, a significant amount of simple major organic compounds, such as amino acids, such as glycine, alanine, and aspartic acid, and peptide chains, began to appear. Simple sugars, urea, and short-chain fatty acids were also formed. In the atmosphere, this spark is provided by U.V. light or other energy sources.
Class 12 Biology Important Questions Chapter 7 Evolution 1
Stanley Miller’s Experiment in the artificial production of organic compounds.

Question 10.
With the help of an algebraic equation, how did Hardy-Weinberg explain that in a given population the frequency of occurrence of alleles of a gene is supposed to remain the same through generations? (CBSE Delhi 2018)
Or
Explain Hardy-Weinberg’s principle. (CBSE Delhi 2019 C)
Answer:
In a given population, one can find out the frequency of occurrence of alleles of a gene or a locus. This frequency is supposed to remain fixed and even remain the same through generations. Hardy-Weinberg’s principle stated it using algebraic equations. According to this principle, allele frequencies in a population are stable and are constant from generation to generation. The gene pool (total genes and their alleles in a population) remains constant. This is called genetic equilibrium.

Sum total of all the allelic frequencies is 1. Individual frequencies, for example, can be named as p, q, etc. In a diploid, p and q represent the frequency of allele A and allele a, respectively. The frequency of AA individuals in a population is simply p2. This is simply stated in another way, i.e. the probability that an allele A with a frequency of p appears on both the chromosomes of a diploid individual is simply the product of the probabilities, i.e. p2. Similarly of aa is q2, of Aa 2pq. Hence, p2 + 2pq + q2 = 1. This is a binomial expansion of (p + q)2. When the frequency measured is different from expected values, the difference (direction) indicates the extent of evolutionary change.

Disturbance in genetic equilibrium, or Hardy-Weinberg equilibrium, i.e. change of frequency of alleles in a population, would then be interpreted as resulting in evolution.

Question 11.
(i) Differentiate between analogous and homologous structures.
Answer:

Analogous organs Homologous organs
(i) Organs that are structurally dissimilar but functionally similar are called analogous organs.
Example: wings of birds and insects.
(i) Organs that are structurally similar but functional dissimilar are called homologous organs.
Example: forelimbs of frog, lizard, bird, bat, horse, man, etc.
(ii) They lead to convergent evolution. (ii) They lead to divergent evolution.

(ii) Select and write analogous structures from the list given below:
(o) Wings of butterfly and birds
(b) Vertebrate hearts
(c) Tendrils of Cucurbita and thorns of Bougainvillea
(d) Tubers of sweet potato and potato (CBSE Delhi 2018)
Answer:
(a) Wings of butterflies and birds.
(b) Tubers of sweet potato and potato.

Question 12.
Write thecharacteristicsofRamapithecus, Dryopithecus, and Neanderthal man. (CBSE Delhi 2017)
Answer:
Characteristics of Ramapithecus:

  • They evolved around 15 mya.
  • They were more man-like, walked more erect, and had teeth like modern men.

Characteristics of Dryopithecus:

  • They evolved around 5 mya.
  • They were ape-like, having hairy arms and legs of the same length, large brains. They used to eat soft fruits and leaves and walked like gorillas and chimpanzees.

Characteristics of Neanderthal Man:

  • They evolved around 1,00,000-40,000 years ago.
  • Fossil found in east and central Asia had brain size 1400 cc. They used hides to protect their body. They buried their dead.

Question 13.
How does the process of natural selection affect Hardy-Weinberg equilibrium? List the other four factors that disturb the equilibrium. (CBSE Outside Delhi 2013)
Or
Write Hardy-Weinberg principle.
Or
How can Hardy-Weinberg equilibrium be affected? Explain giving three reasons. (CBSE Delhi 2018C)
Answer:
Hardy-Weinberg Principle states that the sum of allelic frequencies in a population is stable and is constant from generation to generation, i.e. the gene pool (total genes and their alleles) in a population remains constant. This is called genetic equilibrium. The sum total of all the allelic frequencies is

Hardy-Weinberg’s Equilibrium p2+ q2 + 2pq =

Five factors that influence these values are:
The five factors which affect Hardy- Weinberg’s equilibrium is as follows:

  1. Gene migration: When some individuals of a population migrate to other populations or when certain individuals come into a population (i.e. migration and immigration), some genes are lost in the first case and added in the second.
  2. Genetic drift: Random changes in the allele frequencies of a population occurring only by chance constitute genetic drift. The change in allele frequency may become so drastically different that they form a new species.
  3. Mutations: The mutations are random and directionless. They are sufficient to create a considerable genetic variation for speciation to occur.
  4. Recombination: New combinations of genes occur due to crossing over in meiosis during gametic formation.
  5. Natural selection: It is the most critical evolutionary process that leads to changes in allele frequencies
    and favors adaptation as a product of evolution.

Question 14.
Define genetic drift. How does it produce the founder effect and genetic bottleneck?
Or
How does the original drifted population become a founder? (CBSE 2019 C)
Answer:
Genetic drift: Random change occurring in the allele frequency by chance alone is called genetic drift. It is due to habitat fragmentation, isolation, natural calamities, or any epidemics.

Founder effect: When a section of the population gets separated from the original population, then this section becomes genetically different from the original population due to a change in alleles frequency. The original population becomes the founder of the new population. This is called the founder effect which is the result of genetic drift, i.e. by chance. Genetic bottleneck.

When in a season one population died leaving few individuals of the population which become the founder of the new population, then it will produce only a few genes by selection only, i.e. by chance new population is emerged and it is similar to a bottle in which only certain population is allowed to flow as in the neck of a bottle.

Class 12 Biology Important Questions Chapter 7 Evolution 2
Bottleneck effect

Question 15.
How does Darwin’s theory of natural selection explain new forms of life on earth? (CBSE 2008, 2016)
Answer:
Darwin’s Theory of evolution may be summed up as follows:
Darwin’s Theory of natural selection. Charles Darwin (1809 – 1882), a naturalist, proposed a theory to explain the process of evolution. His theory was published in his famous book “Origin of Species” published in 1858.

His theory of natural selection is termed Darwinism:

  • Rapid multiplication
  • Struggle for existence
  • Variations
  • Natural selection or survival of the fittest
  • Inheritance of useful variations
  • Origin of new species.

Evidence in favor of Darwin’s theory: Darwin’s theory is supported by natural selection, phenomena of mimicry and protective coloration, and the correlation between nectaries of flowers and proboscis of pollinating insects.

Darwin’s theory fails to explain the perpetuation of vestigial organs and over-specialization of organs.
Darwin’s theory has since been modified in the light of progress in genetics.

Question 16.
Describe the present-day concept of evolution.
Answer:
1. Modern concept of evolution: The modern concept of evolution is a modified form of Darwin’s theory of natural selection and is often called Neo-Darwinism. It comprises genetic variation, natural selection, and isolation.
(a) Mutations: These have been recognized as the ultimate source of biological changes and hence the raw material of evolution. The mutation in chromosomes may be due to changes in structure, number, or gene.

(b) Gene Recombination takes place during crossing over in meiosis. New combinations of genes produce new phenotypes.

(c) Hybridisation is the intermingling of the genes of the members of closely related species.

(d) Genetic drift is the elimination of the genes of some original characteristics of a species by extreme reduction due to different reasons.

In Monoparental reproduction, only chromosomal and gene mutation are sources of genetic variation,

2. Natural Selection: If differential reproduction (i.e. some individuals produce abundant offspring, some only a few, and some organisms none) continues for many generations, genes of the individuals which produce more offspring will become predominant in the gene pool of the population. Thus natural selection occurs through differential reproduction in successive generations. The migration of individuals from one to another population is an accessory factor for speciation (origin of new species).

3. Isolation: By selecting the most suitable genotypes, natural selection guides different populations into different adaptive channels. The reproductive isolation between the populations due to certain physical barriers or others leads to the formation of new species. Isolation plays a significant role in evolution.

Question 17.
(i) Name the primates that lived about 15 million years ago. List their characteristic features.
Answer:
Primates Dryopithecus and Ramapithecus lived about 15 mya.

Features:
(a) Hairy and walked like gorillas and chimpanzees. Height up to 4 feet but walked upright.

(ii) (a) Where was the first man-like animal found?
Answer:
Ethiopia and Tanzania

(b) Write the order in which Neanderthals, Homo habilis, and Homo erectus appeared on the earth. State the brain capacity of each one of them.
Answer:

  • Homo habilis – 700 cc
  • Homo erectus – 900 cc
  • Neanderthals man – 1300-1600 cc

(c) When did modern Homo sapiens appear on this planet ? (CBSE Delhi 2011)
Answer:
Homo sapiens appeared about 34000 years ago.

Very Importance Figures:

Class 12 Biology Important Questions Chapter 7 Evolution 3
(A) Foretimbs of vertebrates as homologous organs.
(B) AnaLogous organs. Wings of insect and bird.

Class 12 Biology Important Questions Chapter 7 Evolution 4
Darwin finches

Class 12 Biology Important Questions Chapter 7 Evolution 5
Adaptive radiations of Australian marsupials

Class 12 Biology Important Questions Chapter 7 Evolution 6
Kinds of selection

1 Mark Questions for Economics Class 12

1 Mark Questions for Economics Class 12 Pdf: Here we are providing CBSE 1 Mark Questions for Economics Class 12 in Hindi and English Chapter Wise Pdf. Students can get Class 12 Economics Important Questions, Economics Class 12 One Mark Questions and Answers designed by subject expert teachers.

One Mark Questions for Class 12 Economics Chapter Wise Pdf

Class 12 Economics 1 Mark Questions Part A Introductory Macroeconomics

  1. Introduction to Macroeconomics 1 Mark Questions
  2. National Income Accounting 1 Mark Questions
  3. Money and Banking 1 Mark Questions
  4. Determination of Income and Employment 1 Mark Questions
  5. Government Budget and the Economy 1 Mark Questions
  6. Open Economy Macroeconomics 1 Mark Questions

One Mark Questions for Class 12 Economics Part B Indian Economic Development

  1. Indian Economy on the Eve of Independence 1 Mark Questions
  2. Indian Economy 1950-1990 1 Mark Questions
  3. Liberalisation, Privatisation and Globalisation: An Appraisal 1 Mark Questions
  4. Poverty 1 Mark Questions
  5. Human Capital Formation In India 1 Mark Questions
  6. Rural Development 1 Mark Questions
  7. Employment: Growth, Informalisation and Other Issues 1 Mark Questions
  8. Infrastructure 1 Mark Questions
  9. Environment and Sustainable Development 1 Mark Questions
  10. Comparative Development Experiences of India and its Neighbours 1 Mark Questions

We hope the given NCERT Economics Class 12 One Mark Questions in Hindi and English Chapter Wise Pdf will help you. If you have any queries regarding Class 12 Economics 1 Mark Questions and Answers, drop a comment below and we will get back to you at the earliest.

FAQ’s on 1 Mark Questions for Economics Class 12

Question 1.
Which website caters to 1 mark questions for Economics Class 12?

Answer:
The best site to get the Economics Class 12 1 mark questions is Learninsta.com This is the best online portal to get the 1 mark questions chapterwise.

Question 2.
Is it possible to score top by preparing the 1 Mark Questions for Economics Class 12?

Answer:
Yes you can score the best grades by preparing the Economics Class 12 1 Mark Questions. These Economics 12th Std 1 Mark Questions are really helpful for you during the exams to have a good practice.

Question 3.
Can I get Economics 12th Class 1 Mark Questions for all the chapters?

Answer:
Yes the students of Class 12 can get the 1 mark questions chapterwise. Download 1 Mark Questions for Economics Class 12 Pdf for free of cost and prepare well for the exams.

Question 4.
Does CBSE repeat the 1 mark questions for Economics Class 12 in board exams?

Answer:
Yes, you can find CBSE repeat the 1 mark questions for Economics Class 12 in board exams. So, it is necessary for the 12th class students to concentrate more on repeated 1 mark questions for Economics.

Important Questions for Class 12 Biology Chapter Wise with Answers

Important Questions for Class 12 Biology Chapter Wise Pdf Download 2020-21: Here we are providing CBSE Important Extra Questions for Class 12 Biology Chapter Wise Pdf download in Hindi and English Medium. Students can get Class 12 Biology NCERT Solutions, Biology Class 12 Important Extra Questions and Answers designed by subject expert teachers.

CBSE Class 12th Biology Important Extra Questions and Answers Chapter Wise Pdf

  1. Class 12 Biology Chapter 1 Important Questions Reproduction in Organisms
  2. Class 12 Biology Chapter 2 Important Questions Sexual Reproduction in Flowering Plants
  3. Class 12 Biology Chapter 3 Important Questions Human Reproduction
  4. Class 12 Biology Chapter 4 Important Questions Reproductive Health
  5. Class 12 Biology Chapter 5 Important Questions Principles of Inheritance and Variation
  6. Class 12 Biology Chapter 6 Important Questions Molecular Basis of Inheritance
  7. Evolution Class 12 Important Questions
  8. Human Health and Disease Class 12 Biology Extra Questions
  9. Strategies for Enhancement in Food Production Class 12 Important Questions
  10. Microbes in Human Welfare Class 12 Important Questions
  11. Biotechnology: Principles and Processes Class 12 Important Questions
  12. Biotechnology and its Applications Class 12 Important Questions
  13. Organisms and Populations Class 12 Bio Important Questions
  14. Ecosystem 1 Mark Questions for Biology Class 12
  15. Biodiversity and Conservation Imp Questions of Biology Class 12
  16. Environmental Issues Bio Important Questions Class 12

We hope the given CBSE Important Questions of Biology Class 12 Chapter Wise Pdf download in Hindi and English Medium will help you. If you have any queries regarding NCERT Class 12 Biology Extra Important Questions and Answers, drop a comment below and we will get back to you at the earliest.

FAQ’s on Important Questions for Class 12 Biology

Question 1.
What are the important questions in biology?

Answer:
The list of Class 12 Biology Important questions is provided on this page. Students are advised to check out the entire article and collect the list of important 12th biology questions for better understanding & preparation of the concepts.

Question 2.
Which chapters include more 12th Class Biology Important Questions?

Answer:
According to us, all chapters of CBSE Class 12 biology are equally important to learn the subject and perform well in the exams. But some of the chapters that you should concentrate on are Reproduction, Genetics, Human welfare, Biotechnology, Ecology, etc. From these units, you can find more Important Questions for the Grade 12 Biology Exam.

Question 3.
Where can I get the list of Grade XII biology Important Questions?

Answer:
You can get the list of class 12 important questions for the biology board exam from this page which are arranged in a unit-wise manner.

Question 4.
Which website will help in offering 12th class biology important questions?

Answer:
Learninsta.com is a reliable and trustworthy web portal that helps students in offering board exam study resources like previous papers, important questions, etc. for all classes. So, you can also find 12th class biology important questions from the learninsta.com website.

Issue and Redemption of Debentures Class 12 Important Questions Accountancy Chapter 7

Here we are providing Class 12 Accountancy Important Extra Questions and Answers Chapter 7 Issue and Redemption of Debentures. Accountancy Class 12 Important Questions and Answers are the best resource for students which helps in class 12 board exams.

Class 12 Accountancy Chapter 7 Important Extra Questions Issue and Redemption of Debentures

Issue and Redemption of Debentures Important Extra Questions Very Short Answer Type

Question 1.
What is meant by ‘Issue of Debentures as Collateral Security’ ? (CBSE Outside Delhi 2019)
Answer:
Debenture issued as secondary security/additional security over and above the primary security is known as Issue of Debentures as Collateral Security.

Question 2.
State the provision of the Companies Act, 2013 for the creation of Debenture Redemption Reserve. (CBSE Outside Delhi 2019)
Answer:
Where a company has issued Debentures, it shall create a DRR equivalent to at least 25% of the nominal value of debentures outstanding for the redemption of such debentures.

Question 3.
Profit arisen on account of buying an existing business at profit is transferred to which account?
Answer:
Capital Reserve.

Question 4.
Name the debentures which continue till the continuity of the company.
Answer:
Irredeemable.

Question 5.
Name the debenture which may be converted into equity shares at specified time.
Answer:
Convertible debentures.

Question 6.
Name the debentures which have charge on the company’s assets.
Answer:
Secured debentures (also known as mortgaged debentures).

Question 7.
When a debenture is issued at a price less than its face value or nominal value, what does such difference represent?
Answer:
Discount.

Question 8.
When debentures are redeemed more than the face value of debenture, What does the difference between face value of debenture and redeemed value of debenture is called?
Answer:
Premium on redemption of debentures.

Question 9.
Name the head under which ‘discount on issue of debentures’ appears in the Balance Sheet of a company.
Answer:
Head ‘Current Assets’ and sub-head ‘Other Current Assets’.

Question 10.
What does the repayment or discharge of liability on account of debentures is called?
Answer:
Redemption of debentures.

Question 11.
Under which head is the ‘Debenture Redemption Reserve’ shown in the Balance Sheet?
Answer:
‘Reserve & Surplus’.

Question 12.
When the company issues debentures to the lenders as an additional/secondary security, in addition to other assets already pledged/ some primary security. What does such issue of debentures is called? (CBSE 2018)
Answer:
Issue of dedentures as collateral security.

Question 13.
It is a written instrument acknowledging a debt under the common seal of the company, name the term.
Answer:
Debenture.

Question 14.
State an exception to the creation of Debenture Redemption Reserve as per Companies (Share Capital and Debentures) Rules 18(7). (CBSE Sample Paper 2014 Modified)
Answer:
Banking Companies

Question 15.
Mention the type of debentures whose ownership passes on mere delivery of debenture certificates.
Answer:
Bearer debentures.

Question 16.
Can ‘Securities Premium’ be used as working capital?
Answer:
No.

Question 17.
A company purchased net assets of another company worth ₹ 20,00,000 and issued debentures worth ₹ 19,00,000. What type of profit has the buying company made?
Answer:
Capital Profit.

Question 18.
Vikas Infrastructure Ltd. has issued 50,000, 10% debentures of ₹ 100 each at par redeemable after the end of 7th year. Mention the amount by which the company should create Debenture Redemption Reserve as per Companies (Share Capital and Debentures) Rules 2014 before starting redemption of debenture. Answer with giving reason.
Answer:
₹ 12,50,000.

Question 19.
Axis Ltd. has issued 8,000, 10% debentures of₹ 100 at a premium of ₹ 5 per debenture redeemable at the end of 5 years. The company has created Debenture Redemption Reserve with ₹ 4,00,000. After 5 years, the company redeemed all the debentures ₹ Where should the company transfer the amount of Debenture Redemption Reserve?
Answer:
General Reserve.

Issue and Redemption of Debentures Important Extra Questions Short Answer Type

Question 1.
Garvit Ltd. invited applications for issuing 3,000, 11% Debentures of₹ 100 each at a discount of 6%. The full amount was payable on application. Applications were received for 3,600 debentures. Applications for 600 debentures were rejected and the application money was refunded. Debentures were allotted to the remaining applicants.
Pass the necessary journal entries for the above transactions in the books of Garvit Ltd. (CBSE Delhi 2019)
Answer:
Class 12 Accountancy Important Questions Chapter 7 Issue and Redemption of Debentures 1

Question 2.
On 1st April 2015, P Ltd. Issued 6,000 12% Debentures of ₹ 100 each at par redeemable at a premium of 7%. The Debentures were to be redeemed at the end of third year. Prepare Loss on issue of 12% Debentures Account. (CBSE Delhi 2019)
Answer:
Class 12 Accountancy Important Questions Chapter 7 Issue and Redemption of Debentures 2

Question 3.
Unilink Ltd. (An unlisted company) had outstanding ₹ 12,00,000, 9% debentures on 1st April, 2020 redeemable at a premium of 8% in two equal annual instalments starting from 31st March, 2022. The company had a balance of₹ 1,20,000 in Debenture Redemption-Reserve on 31st March, 2020. Pass the necessary journal entries for redemption of debentures in the books of Unilink Ltd. for the year ended 31st March, 2022. . (CBSE Delhi 2019, Modified)
Answer:
Class 12 Accountancy Important Questions Chapter 7 Issue and Redemption of Debentures 3
Class 12 Accountancy Important Questions Chapter 7 Issue and Redemption of Debentures 4
Note: This question has been updated as per Companies (Share Capital and Debentures) Amendment Rules, 2019.

Question 4.
Krishna Ltd. (An unlisted company) had outstanding 20,000,9% debentures of₹ 100 each on 1st April, 2014. These debentures were redeemable at a premium of 10% in two equal instalments starting from 31 st March, 2021. The company had a balance of ₹2,00,000 in Debenture Redemption Reserve on 31 st March, 2020.
Pass necessary journal entries for redemption of debentures in the books of Krishna Ltd. for the year ended 31st March, 2018. (CBSE Delhi 2019, Modified)
Answer:
Class 12 Accountancy Important Questions Chapter 7 Issue and Redemption of Debentures 5
Note: This question has been updated as per Companies (Share Capital and Debentures) Amendment Rules, 2019.

Question 5.
On 1st April, 2013 Anushka Ltd. (An unlisted companies) issued ₹70,00,000, 9% debentures of ₹ 100 each at par, redeemable at a premium of 5% on 31 st march, 2021. The company created the necessary, minimum amount of debenture redemption reserve and purchased debenture redemption reserve investments. The debentures were redeemed on 31 st March, 20121. Pass necessary journal entries for the redemption of debentures, in the books of the company.
(CBSE Delhi 2019, Modified)
Answer:
Class 12 Accountancy Important Questions Chapter 7 Issue and Redemption of Debentures 6
Note: This question has been updated as per companies (Share capital and debentures) Amendment Rules, 2019.

Question 6.
Pass necessary journal entries and prepare 9% Debentures Account for the issue of 7,500,9% Debentures of ₹ 50 each at a discount of 6%, redeemable at a premium of 10%. (CBSE Delhi 2019, Modified)
Answer:
Class 12 Accountancy Important Questions Chapter 7 Issue and Redemption of Debentures 7
Class 12 Accountancy Important Questions Chapter 7 Issue and Redemption of Debentures 8

Question 7.
Krishna Ltd. had (an unlisted company) outstanding 20,000, 9% debentures of₹ 100 each on 1st April, 2014.. These debentures were redeemable at a premium of 10% in two equal instalments starting from 31st March, 2021.
The company had a balance of₹ 1,80,000 in Debenture Redemption Reserve on 31st March, 2020.
Pass necessary journal entries for redemption of debentures in the books of Krishna Ltd. for the year ended 31 st March, 2021. (CBSE Outside Delhi 2019, Modified)
Answer:
Class 12 Accountancy Important Questions Chapter 7 Issue and Redemption of Debentures 9
Class 12 Accountancy Important Questions Chapter 7 Issue and Redemption of Debentures 10
Note: This question has been updated as per companies (Share capital and debentures) Amendment Rules, 2019.

Question 8.
On 1 st April, 2013 Anushka Ltd. (unlisted company) issued ₹70,00,000, 9% debentures of ₹ 100 each at par, redeemable at a premium of 5% on 31 st march, 2021. The company created the necessary, minimum amount of debenture redemption reserve and purchased debenture redemption reserve investments. The debentures were redeemed on 31st March, 2021.
Pass necessary journal entries for the redemption of debentures, in the books of the company.
Answer:
Class 12 Accountancy Important Questions Chapter 7 Issue and Redemption of Debentures 11
Note: This question has been updated as per companies (Share capital and debentures) Amendment Rules, 2019.

Question 9.
Complete the following Journal Entries
Class 12 Accountancy Important Questions Chapter 7 Issue and Redemption of Debentures 12
Answer:
Class 12 Accountancy Important Questions Chapter 7 Issue and Redemption of Debentures 13

Question 10.
V K Limited purchased machinery from Modem Equipment Manufacturers Limited. The company paid the vendors by issue of some equity shares and debentures and the balance through an acceptance in then- favour payable after three months. The accountant of the company, while Journalising the above mentioned transactions, left some items blank. You are required to fill in the blanks.
Class 12 Accountancy Important Questions Chapter 7 Issue and Redemption of Debentures 14
Answer:
Class 12 Accountancy Important Questions Chapter 7 Issue and Redemption of Debentures 15

Question 11.
‘sangam Wooilens Ltd’, Ludhiana, are the and exporters of garments. The company decided to distribute free of cost oUes. garments to 10 villages of lahual and spiti district of Himachal Pradesh. The company also decided to employ 50 young persons from these villages in its newly established factory. The company issued 40,000 equity shares of’ 10 each and 1,000 9% debentures of’ 100 each to the vendors for the purchase of machinery of’ 5,00,000.
Pass necessary Journal Entries. (Dehli 2015, Modified)
Answer:
Class 12 Accountancy Important Questions Chapter 7 Issue and Redemption of Debentures 16

Question 12.
Anirudh Limited (Listed company) has 4,000, 8% debentures of₹ 100 each due for redemption on March 31, 2022. The company has a DRR of ₹ 20,000 on that date. Assuming that no interest is due. Record the necessary journal entries at the time of redemption of debentures.
Answer:
Class 12 Accountancy Important Questions Chapter 7 Issue and Redemption of Debentures 17
Note: This question has been updated as per companies (Share capital and debentures) Amendment Rules, 2019.

Question 13.
R. Ltd. offered 20,00,000,10% Debenture of₹ 200 each at a discount of redeemable at premium of 8% after 9 years. Record necessary entries in the book of R. Ltd.
Answer:
Class 12 Accountancy Important Questions Chapter 7 Issue and Redemption of Debentures 18

Question 14.
Journalise the following:
(i) A debenture issued at ₹ 95, repayable at ₹ 100;
(ii) A debenture issued at ₹ 95, repayable at ₹ 105; and
(iii) A debenture issued at ₹ 100 repayable at ₹ 105;
The face value of debenture in each of the above cases is ₹ 100.
Answer:
Class 12 Accountancy Important Questions Chapter 7 Issue and Redemption of Debentures 19
Class 12 Accountancy Important Questions Chapter 7 Issue and Redemption of Debentures 20

Question 15.
A Company issues the following debentures:
(i) 10,000,12% debentures of₹ 100 each at par but redeemable at premium of 5% after 5 years;
(ii) 10,000,12% debentures of₹ 100 each at a discount of 10% but redeemable at par after 5 years;
(iii) 5,000,12% debentures of₹ 1,000 each at a premium of 5% but redeemable at par after 5 years;
Answer:
Class 12 Accountancy Important Questions Chapter 7 Issue and Redemption of Debentures 21
Class 12 Accountancy Important Questions Chapter 7 Issue and Redemption of Debentures 22
Class 12 Accountancy Important Questions Chapter 7 Issue and Redemption of Debentures 23

Question 16.
X Ltd. invited applications for issuing 1000,9% debentures of₹ 100 each at a discount of 6%. Applications for 1,200 debentures were received. Pro-rata allotment was made to all the applicants. Pass necessary Journal Entries for the issue of debentures assuming that the whole amount was payable with applications. [Delhi 2017]
Answer:
Class 12 Accountancy Important Questions Chapter 7 Issue and Redemption of Debentures 24

Question 17.
Z Ltd. purchased machinery from K Ltd. Z Ltd paid K Ltd as follows:
(i) By issuing 5,000 equity shares of ₹ 10 each at a premium of 30%.
(ii) By issuing 1000, 8% Debentures of₹ 100 each at a discount of 10%.
(iii) Balance by giving a promissory note of ₹ 48,000 payable after two months.
Pass necessary journal entries for the purchase of machinery and payment to K Ltd. in the books of Z Ltd. [Delhi 2017]
Answer:
Class 12 Accountancy Important Questions Chapter 7 Issue and Redemption of Debentures 25
Class 12 Accountancy Important Questions Chapter 7 Issue and Redemption of Debentures 26

Question 18.
K K Limited obtained a loan of₹ 10,00,000 from State Bank of India @ 9 % interest. The company issued ₹ 15,00,000, 9 % debentures of₹ 100/- each, in favour of State Bank of India as collateral security. Pass necessary Journal entries for the above transactions:
(i) When company decided not to record the issue of 9 % Debentures as collateral security.
(ii) When company decided to record the issue of 9 % Debentures as collateral security. (CBSE Sample Paper 2018-19, 2017-18)
Answer:
Class 12 Accountancy Important Questions Chapter 7 Issue and Redemption of Debentures 27

Question 19.
Explain with an imaginary example how issue of debenture as collateral security is shown in the balance sheet of a company when it is recorded in the books of accounts. (CBSE Sample Paper 2016, 2017)
Answer:
Alfa Ltd. obtained Loan of 1,00,000 from Indian Bank and issued 1200, 10% Debentures of 100 each as Collateral security.
Treatment: An extract of Balance sheet of Alfa Ltd.
Class 12 Accountancy Important Questions Chapter 7 Issue and Redemption of Debentures 28

Question 20.
KTR Ltd., issued 365, 9% Debenture of 7’1,000 each on 4.3.2016. Pass necessary journal entries for the issue of debenture in the following situations : (CBSE Outside Delhi 2016)
(i) When debentures were issued at per redeemable at a premium of 10%.
(ii) When debentures were issued at 6% discount redemable at 5% premium.
Answer:
Class 12 Accountancy Important Questions Chapter 7 Issue and Redemption of Debentures 29
Class 12 Accountancy Important Questions Chapter 7 Issue and Redemption of Debentures 30

Question 21.
VKR Ltd. issued 975; 9% Debentures of 7 500 each on 4.3.2016. Pass necessary journal entries for the issue of debentures under the following situations:
(i) When debentures were issued at a premium of 10% redeemable at a premium of 6%.
(ii) When debentures were issued at a par redeemable at 9% premium. (CBSE Outside Delhi 2016)
Answer:
Class 12 Accountancy Important Questions Chapter 7 Issue and Redemption of Debentures 31
Class 12 Accountancy Important Questions Chapter 7 Issue and Redemption of Debentures 32

Question 22.
BG. Ltd. issued 2,000, 12% debentures of (100 each on 1st April 2012. The issue was fully subscribed. According to the terms of issue, interest on the debentures is payable half-yearly on 30s1 September and 31st March and the tax deducted at source is 10%. Pass necessary journal entries related to the debenture interest for the half-yearly ending 3151 March, 2013 and transfer of interest on debentures of the year to the Statement of Profit & Loss. . (CBSE Delhi 2014, Set I, II)
Answer:
Class 12 Accountancy Important Questions Chapter 7 Issue and Redemption of Debentures 33
Class 12 Accountancy Important Questions Chapter 7 Issue and Redemption of Debentures 34
Question 23.
Jan Dhan Bank, an All India Financial Institution, had 10,000,12% debentures of₹ 100 each, outstanding as at 31st March, 2022. These debentures were due for redemption on 30th June, 2023. Pass necessary Journal Entries for redemption of debentures. Also, state the amount of Debenture Redemption Reserve to be created for the purpose of redemption. (CBSE Sample Paper 2018-19, Modified)
Answer:
Class 12 Accountancy Important Questions Chapter 7 Issue and Redemption of Debentures 35
Note:
According to Section 71(4) of the Companies Act, 2013 and Companies (Share Capital & Debentures) Amendment Rules, 2019, an All India Financial Institution is not required to create Debenture Redemption Reserve.

Question 24.
Raghuveer Limited created 10,00,000, 8% debentures stock which was issued as follows to:
1. Sundry subscribers for cash at 90%
2. Creditors for ₹ 2,00,000 capital expenditure in satisfaction of his claim
3. Bankers as collateral securities for a bank loan
worth ₹ 20,00,000 for which principal security is business premises worth
The issue (1) and (2) are redeemable at the end of 10 years at par. State how the debenture stock be dealt with while preparing the balance sheet of a company.
Answer:
Class 12 Accountancy Important Questions Chapter 7 Issue and Redemption of Debentures 36
Class 12 Accountancy Important Questions Chapter 7 Issue and Redemption of Debentures 37

Question 25.
Hassan Limited took a loan of ₹ 30,00,000 from a bank against primary security worth ₹ 40,00,000 and . issued 4,000, 6% debentures of₹ 100 each as a collateral security. The company again after one year took a loan of ₹ 50,00,000 from bank against plant as primary security and deposited 6,000, 6% debentures of₹ 100 each as collateral security. Record necessary journal entries and prepare balance sheet of the company.
Class 12 Accountancy Important Questions Chapter 7 Issue and Redemption of Debentures 38
Answer:
Class 12 Accountancy Important Questions Chapter 7 Issue and Redemption of Debentures 39

Question 26.
Meghnath Limited took a loan of ₹ 1,20,000 from a bank and deposited 1,400, 8% debentures of ₹ 100 each as collateral security along with primary security worth ₹ 2 Lakhs. Company again took a loan of ₹ 80,000 after two months from a bank and deposited 1,000, 8% debentures of ₹ 100 each as collateral security. Record necessary journal entries and prepare a balance sheet of a company.
Answer:
Class 12 Accountancy Important Questions Chapter 7 Issue and Redemption of Debentures 40
Class 12 Accountancy Important Questions Chapter 7 Issue and Redemption of Debentures 41

Question 27.
Diwakar Enterprises Ltd. issued 10,00,000, 6% debentures on April 1, 2008. Interest is paid on September 30, 2012 and March 31, 2013.
Record necessary journal entries assuming that income tax is deducted @ 30% of the amount of interest.
Answer:
Class 12 Accountancy Important Questions Chapter 7 Issue and Redemption of Debentures 42
Class 12 Accountancy Important Questions Chapter 7 Issue and Redemption of Debentures 43

Question 28.
Laser India Ltd. issued 7,00,000, 8%. debentures of₹ 100 each at par. Company deducts income tax from the interest of these debentures at source. Interest is to be paid on these debentures half yearly on September 30 and March 31, every year. Amount of income tax deducted half yearly is ₹ 2,80,000.
Answer:
Class 12 Accountancy Important Questions Chapter 7 Issue and Redemption of Debentures 44
Class 12 Accountancy Important Questions Chapter 7 Issue and Redemption of Debentures 45

Question 29.
B. Ltd. purchased assets of the book value of ₹ 4,00,000 and took over the liability of ₹ 50,000 from Mohan Bros. It was agreed that the purchase consideration settled at ₹ 3,80,000 be paid by issuing debentures of₹ 100 each.
What journal entries will be made in the following three cases, if debentures are issued: (a) at par; (b) at a discount of 10%, (c) at a premium of 10%₹ It was agreed that a fraction of debentures be paid in cash.
Answer:
Class 12 Accountancy Important Questions Chapter 7 Issue and Redemption of Debentures 46
Class 12 Accountancy Important Questions Chapter 7 Issue and Redemption of Debentures 47

Question 30.
B. Ltd. issued 1,000,12% debentures of₹ 100 each on January 01,2008 at a discount of 5% redeemable at a premium of 10%.
Give journal entries relating to the issue of debentures and debentures interest for the period ending December 31,2008 assuming that interest is paid half yearly on June 30 and December 31 and tax deducted at sources is 10%. B. Ltd. follows calendar year as its accounting year.
Answer:
Class 12 Accountancy Important Questions Chapter 7 Issue and Redemption of Debentures 48
Class 12 Accountancy Important Questions Chapter 7 Issue and Redemption of Debentures 49

Issue and Redemption of Debentures Important Extra Questions Long Answer Type

Question 1.
Journalise the following transactions
(a) Mehar Ltd. issued ₹ 1,00,000, 12% Debentures of ₹ 100 each at a premium of 5% redeemable at a premium of 2%
(b) 12% Debentures • were issued at a discount of 10% to a vendor of machinery for payment of ₹ 9,00,000
(c) Issue of 10,000 11% debentures of₹ 100 each as collateral in favour of State Bank of India. Company
opted to pass necessary entry for issue of debentures. (CBSE Sample Paper 2019-20)
Answer:
Class 12 Accountancy Important Questions Chapter 7 Issue and Redemption of Debentures 50

Question 2.
Faith and Belief Ltd has total redeemable debentures of₹ 5,00,000. It decides to redeem these debentures in two instalments of₹ 3,00,000 and ₹ 2,00,000 on December 31st 2021 and March 31st 2023 respectively. Assuming that the Company has sufficient funds in Debenture Redemption Reserve Account, pass necessary journal entries for the year ending March 31st 2020. (CBSE Sample Paper 2019-20, Modified)
Answer:
Class 12 Accountancy Important Questions Chapter 7 Issue and Redemption of Debentures 51
Class 12 Accountancy Important Questions Chapter 7 Issue and Redemption of Debentures 52
Note:
This question has been updated as per companies (Share capital and debentures) Amendment Rules, 2019.

Question 3.
On 1st April, 2016, Ganesh Limited (An Unlisted Company) acquired assets of₹ 6,00,000 and took over liabilities of₹ 70,000 of Sohan Ltd. at an agreed value of₹ 6,60,000 Ganesh Ltd. issued 12% Debentures of ₹ 100 each at a premium of 10% in Ml satisfaction of purchase consideration. The debentures were redeemable after three years at a premium of 5%. The company decided to transfer the minimum required amount to Debenture Redemption Reserve of 31st March, 2022. It also made the required investment in Government securities earning interest @ 10% p.a. on IstApril, 2022. Tax was deducted on interest earned @ 10%.
Ignoring entries relating to writing off loss on issue of debentures and interest paid on debentures, pass the necessary journal entries to record the issue and redemption of debentures. (CBSE Compt. 2019, Modified)
Answer:
Class 12 Accountancy Important Questions Chapter 7 Issue and Redemption of Debentures 53
Class 12 Accountancy Important Questions Chapter 7 Issue and Redemption of Debentures 54

Question 4.
On 1st April, 2016 X Ltd. (An unlisted company) issued 1000; 9% debentures of₹ 100 each at a premium of ₹ 5 per debenture and redeemable on 31 st March, 2022 at a premium of ₹ 8 per debenture. The company created the minimum amount of debenture redemption reserve as per the amended provisions of the Companies Act, 2013 on 31 st March, 2021 and made investments in 8% p.a. fixed deposits in State Bank of India on 1 st April, 2021.
Excluding the entries for writing off loss on issue of debentures and interest on debentures, pass necessary journal entries for the above transactions in the books of X Ltd. (CBSE Compt. 2019, Modified)
Answer:
Class 12 Accountancy Important Questions Chapter 7 Issue and Redemption of Debentures 55
Class 12 Accountancy Important Questions Chapter 7 Issue and Redemption of Debentures 56
————-

Question 5.
Nena Limited issued 50,000, 10% debentures of₹ 100 each on the basis of the following conditions:
(a) Debentures issued at par and redeemable at par.
(b) Debentures issued at discount @ 5% & redeemable at par.
(c) Debentures issued at a premium @ 10% & redeemable at par.
(d) Debentures issued at par & redeemable at premium @ 10%.
(e) Debentures issued at discount of 5% and redeemable at a premium of 10%.
(f) Debentures issued at premium of 6% and redeemable at a premium of 4%.
Record necessary journal entries in the above mentioned cases at the time of issue and redemption of debentures.
Answer:
Class 12 Accountancy Important Questions Chapter 7 Issue and Redemption of Debentures 57
Class 12 Accountancy Important Questions Chapter 7 Issue and Redemption of Debentures 58
Class 12 Accountancy Important Questions Chapter 7 Issue and Redemption of Debentures 59
Class 12 Accountancy Important Questions Chapter 7 Issue and Redemption of Debentures 60
Class 12 Accountancy Important Questions Chapter 7 Issue and Redemption of Debentures 61
Class 12 Accountancy Important Questions Chapter 7 Issue and Redemption of Debentures 62

Question 6.
Z. Limited (An unlisted company) issued, 2,000, 14% debentures of₹ 100 each on January 01, 2020 at a discount of 10%, redeemable at a premium of 10% in equal annual Drawings in 4 years out of profits. Give journal entries both at the time of issue and redemption of debentures.
(Ignore the treatment of loss on issue of debentures and interest.)
Answer:
Class 12 Accountancy Important Questions Chapter 7 Issue and Redemption of Debentures 63
Class 12 Accountancy Important Questions Chapter 7 Issue and Redemption of Debentures 64
Class 12 Accountancy Important Questions Chapter 7 Issue and Redemption of Debentures 65
Class 12 Accountancy Important Questions Chapter 7 Issue and Redemption of Debentures 66

Question 7.
On 1-4-2015 K.K. Ltd. issued 500, 9% Debentures of ₹ 500 each at a discount of 4%, redeemable at a premium of 5% after three years.
Pass necessary Journal Entries for the issue of debentures and debenture interest for the year ended 31 -3-2016 assuming that interest is payable on 30th September and 31 st March and the rate of tax deducted at source is 10%. The company closes its books on 31st March every year. [Delhi 2017]
Answer:
Class 12 Accountancy Important Questions Chapter 7 Issue and Redemption of Debentures 67
Class 12 Accountancy Important Questions Chapter 7 Issue and Redemption of Debentures 68

Question 8.
X. Ltd. issued 15,000, 10% debentures of₹ 100 each. Give journal entries and the Balance Sheet in each of the following cases:
(i) The debentures are issued at a premium of 10%.
(ii) The debentures are issued at a discount of 5%.
(iii) The debentures are issued as a collateral security to bank against a loan ₹ 12,00,000.
(iv) The debentures are issued to a supplier of machinery costing ₹ 13,50,000.
Answer:
Class 12 Accountancy Important Questions Chapter 7 Issue and Redemption of Debentures 69
Class 12 Accountancy Important Questions Chapter 7 Issue and Redemption of Debentures 70
Class 12 Accountancy Important Questions Chapter 7 Issue and Redemption of Debentures 71
Class 12 Accountancy Important Questions Chapter 7 Issue and Redemption of Debentures 72
Class 12 Accountancy Important Questions Chapter 7 Issue and Redemption of Debentures 73
Class 12 Accountancy Important Questions Chapter 7 Issue and Redemption of Debentures 74
Class 12 Accountancy Important Questions Chapter 7 Issue and Redemption of Debentures 75
Class 12 Accountancy Important Questions Chapter 7 Issue and Redemption of Debentures 76

Question 9.
On 1st April, 2018, KK Ltd. (An unlisted company) invited applications for issuing 5,000 10% debentures of₹ 1,000 each at a discount of 6%. These debentures were payable at the end of 3rd year at a premium of 10%. Applications for 6,000 debentures were received and the debentures were allotted on pro-rata basis to all the applicants. Excess money received with applications was refunded. The directors decided to transfer the minimum amount to Debenture Redemption Reserve on 31.3.2020. On 1.4.2020, the company invested the necessary amount in 9% bank fixed deposit as per the amended provisions of the Companies Act, 2013. Tax was deducted at source by bank on interest @ 10% p.a. Pass the necessary journal entries for issue and redemption of debentures. Ignore entries relating to writing off loss on issue of debentures and interest paid on debentures. (CBSE 2018, Modified)
Answer:
Class 12 Accountancy Important Questions Chapter 7 Issue and Redemption of Debentures 77
Class 12 Accountancy Important Questions Chapter 7 Issue and Redemption of Debentures 78

Question 10.
Ruchi Ltd (An unlisted company) issued 42,000,7% Debentures of 100 each on 1st April, 2015, redeemable at a premium of 8% on 31st March 2021. The Company decided to create required Debenture Redemption Reserve on 31st March 2020. The company invested the funds as required by law in a fixed deposit with State Bank of India on 1st April, 2021 earning interest @ 10% per annum. Tax was deducted at source by the bank on interest @ 10% per annum. Pass necessary Journal Entries regarding issue and redemption of debentures.
(CBSE Sample Paper 2016, 2017, Modified)
Answer:
Class 12 Accountancy Important Questions Chapter 7 Issue and Redemption of Debentures 79
Class 12 Accountancy Important Questions Chapter 7 Issue and Redemption of Debentures 80

Question 11.
(Lump-sum Method): Alibaba Ltd. (An unlisted company) issued 20,000, 9% debentures of ₹ 50 each on April 1, 2022 redeemable at par on March 31, 2022. All the debentures were subscribed and allotted. Investment of the required amount in securities is subject to deduction of 10% tax at source. Pass journal entries for issue and redemption of debentures assuming required investments were made in 6% specified securities.
Answer:
Class 12 Accountancy Important Questions Chapter 7 Issue and Redemption of Debentures 81
Class 12 Accountancy Important Questions Chapter 7 Issue and Redemption of Debentures 82

Question 12.
(Lump-sum Method): On April 1, 2020, Nelson and Tubro Ltd. (An unlisted company) issued 10,000, 10% debentures of₹ 100 each redeemable at 5% premium on March 31, 2022. Debentures were fully subscribed and allotted. Pass necessary journal entries for issue and redemption of debentures assuming necessary amount was invested in 7 % specified securities.
Answer:
Class 12 Accountancy Important Questions Chapter 7 Issue and Redemption of Debentures 83
Class 12 Accountancy Important Questions Chapter 7 Issue and Redemption of Debentures 84
Note: This question has been updated as per companies (Share capital and debentures) Amendment Rules, 2019.

Question 13.
(Instalments or Draw a Lot Method): Neelgiri Limited issued 6,000, 10% debentures of ₹ 100 each on April 01, 2018 redeemable in 3 three equal instalments commeucing with March 31, 2021. The board of directors decided to transfer the required amount to Debenture Redemption Reserve in 2 equal instalments. Company also complied with Companies (Share Capital and Debentures) Amendment Rules, 2019.
Answer:
Class 12 Accountancy Important Questions Chapter 7 Issue and Redemption of Debentures 85
Class 12 Accountancy Important Questions Chapter 7 Issue and Redemption of Debentures 86
Class 12 Accountancy Important Questions Chapter 7 Issue and Redemption of Debentures 87

Financial Management Class 12 Important Extra Questions Business Studies Chapter 9

Here we are providing Class 12 Business Studies Important Extra Questions and Answers Chapter 9 Financial Management. Business Studies Class 12 Important Questions are the best resource for students which helps in class 12 board exams.

Class 12 Business Studies Chapter 9 Important Extra Questions Financial Management

Financial Management Important Extra Questions Short Answer Type

Question 1.
What are the various factors affecting Financial Planning?
Answer:
A financial plan should be prepared very carefully because it has a long-term impact on the working of an enterprise. A financial plan is affected by a number of factors. All these factors should be’ taken into consideration while preparing a financial plan.

1. Nature of Business: The nature of business plays a decisive role in formulating a financial plan. A manufacturing business requires more amount of long-term funds than a trading business. In addition to it, the factors such as stability and regularity of income, future prospects of growth, seasonal fluctuations, assets structure, etc. affect the financial requirements as well as sources of finance.

2. Degree of Risk: The risk involved in the business also plays an important role while planning the sources of finance. A firm whose sales and earnings are subject to wide fluctuations runs the risk of not being able to meet the required payments in respect of interest and repayment of loans. Clearly, such firms should use more amount of their own funds and rely less on debt. On the other hand, the enterprises with stable sales and earnings can employ more amount of debts and hence can take the advantage of trading on equity.

3. Standing of the concern: Credit standing of concern among investors affects financial planning to a great extent. The credit standing of concern is determined by a number of factors such as the age of the firm, its past performance, size, market area, the reputation of management, etc.

4. Plans for future Growth: The plans for growth and expansion of the firm in near future are considered while formulating a financial plan. The financial plan should be developed in such a way as to facilitate required funds without much difficultly.

5. Alternative Sources of Finance: Since finance can be procured from a number of sources, the pros and cons of all the sources should be properly considered while choosing the proper sources of finance. The sources should be able to provide adequate funds to meet the requirements of the business.

6. Attitude of Management: The attitude of management towards risk and control of the business affects financial planning to a great extent. If the management is of risk-taking nature, it would employ more amount of borrowed funds. On the contrary, if it is of conservative nature it will employ more amount of equity capital. From the control point of view, if the management desires to keep full control of the enterprise, it will not issue fresh equity shares so that the new shareholders may not control the enterprises.

7. Government Policies and Control: The financial plan of a company is affected by the rules and regulations framed by the Government stock exchanges and financial institutions from time to time. The terms of issue of shares and debentures, interest rates, dividend payments, etc. are governed by the rules framed by the government periodically. Permission of the Securities and Stock Exchange Board of India (SEBI) is also required for the issue of shares and debentures.

8. Changes in Technology, Consumer Tastes, and Competitive Factors: Rapid innovations are taking place in every field nowadays. A financial plan is adequately affected by changes in technology, consumer preferences, degree of competition, and general economic conditions.

Question 2.
Explain in brief the various steps in financial planning.
Answer:
Following steps should be taken for preparing a financial plan:
1. Determination of Financial Objectives: For the purpose of preparing an effective financial plan first of all the financial objectives of a firm should be clearly determined. The financial objectives should be divided into short-term objectives as well as long-term objectives. The short-term objectives may include maintaining the liquidity of funds, maintaining the market standing of the firm and proper maintenance of sales, etc.

On the other hand, the long-term objectives may include the achievement of maximum efficiency of factors of production at minimum cost and the maximization of shareholder’s wealth. The objectives should be clearly defined so that they can be used as guidelines for determining the 1 policies and procedures.

2. Formulation of Financial Policies: The second step in financial planning is the formulation of financial policies. Financial policies act as guidelines for the f procurement, allocation, and effective utilization of funds of the organization. Financial policies are framed by the top management with the advice of the financial manager. The policies may be regarding capitalization, capital structure, trading on equity, fixed assets management, working capital management, dividend distribution, etc.

3. Formulation of Procedures: The policies laid down must be clarified in the form of detailed procedures. Each subordinate must know what he is required to do. Procedures are essential to ensure the consistency of actions. In financial procedures, financial executives decide about the control system, establish the standards of performance and compare the actual performance with the standards to ascertain the deviations and their causes. Thereafter, necessary steps are taken to control the deviations.

4. Provision of Flexibility: The objectives, policies, and procedures laid down as above constitute the financial plan of a business. Financial planning is a continuous process and hence there should be proper flexibility in the financial objectives, policies, and procedures so that these may be revised or thoroughly overhauled according to the changing circumstances.

Question 3.
Explain the major characteristics or Principles of a sound financial plan.
Answer:
An ideal financial plan must be based on the principles or qualities mentioned below:
1. Simplicity: Financial plan should be so simple that it may be easily understood by everyone. It should have a simple capital structure capable of being managed easily. The type of securities issued should be kept at a minimum because various types of securities will create unnecessary suspicion in the minds of investors.

2. Foresight: The financial plan should be prepared to keep in view the future needs of the business. It should take into consideration the future demand of the company’s products, the future scale of operations, technological innovations, and various other changes. A financial plan should be able to meet the future requirements of fixed as well as working capital.

3. Optimum use of Funds: An ideal financial plan should always aim at the best possible and intensive use of all available resources of finance. The business should neither be starved of funds nor it should have a surplus or idle funds. Unnecessary idle funds are as bad as inadequate funds. A proper balance should also be kept between the short-term and long-term funds of the business.

4. Flexibility: A financial plan should be sufficiently flexible. It should be possible for a company to change its financial plan with minimum cost and delay if warranted by changed circumstances. The company should be able to substitute one form of financing for another to economize the use of funds. The financial plan should allow a scope for adjustments as and when a new situation arises like recession, boom, etc. A rigid financial plan can easily become a burden rather than a 1 technique of financial management.

5. Liquidity: Liquidity is the ability of the enterprise to pay off its day-to-day expenses and other short-term liabilities on time. The financial plan should provide sufficient liquidity of funds as it will ensure the creditworthiness and goodwill of the enterprise. Adequate liquidity in the \ financial plan increases its flexibility also.

6. Economical: Financial plan must be prepared in such a way that the cost of capital is minimum. The average cost of capital will be minimum when a fair 1 balance is maintained between debt funds and owned capital. Also, the financial plan should involve minimum expenses on the issue of capital such as underwriting Commission, brokerage, etc.

7. Contingencies: A financial plan should keep-in view the requirement of funds for contingencies. Contingencies mean the requirement of funds for unseen: events.

8. Adequate system of Control: A financial plan should establish and maintain a proper system of financial control.

9. Suitable to the Organisation structure: A financial plan should be in accordance with the size and organizational structure of the firm.

Question 4.
Define the nature and types of working capital.
Answer:
Along with the fixed capital, almost every business requires working capital though the extent of working capital requirement differs in different businesses. Working capital is needed for running the day-to-day business activities. When a business is started, working capital is needed for purchasing raw materials. The raw’ material is then converted into finished goods by incurring some additional costs on it.

Now goods are sold. Sales do not convert into cash instantly because there is invariably some credit sales. Thus, there exists a time, lag between sales of goods and receipt of cash. During this period, expenses are to be incurred for continuing the* business operations.

For this purpose working capital is needed, Therefore, sufficient working capital is needed which shall be involved from the purchase of raw materials to the realization of cash.

The time period which is required to convert raw materials into finished goods and then into cash is known as the operation cycle or cash cycle. The need for working capital can also be explained with the help of the operating cycle.

The operating cycle of a manufacturing concern involves five phases:

  1. Conversion of cash into raw material
  2. Conversion of raw material into work-in-progress
  3. Conversion of work-in-progress into finished goods
  4. Conversion of finished goods into debtors by credit sales
  5. Conversion of debtors into cash by realizing cash from them.

Thus the operating cycle starts from cash, finishes at cash, and then again restarts from cash. The need for working capital depends upon the period of the operation cycle. Greater the period, more will be the need for working capital. The period of operation cycle in a manufacturing concern is greater than a period of operating cycle in a trading concern because in trading units cash is directly converted into finished goods.
Class 12 Business Studies Important Questions Chapter 9 Financial Management 1
Diagram: Operating Cycle (Nature of Working Capital)

Because of the time involved in an operating cycle, there is a need, for working capital in the form of current assets. Firms have to keep I adequate stock of raw materials to avoid the risk of non-availability of raw materials. Similarly, the concern must have adequate stock of finished goods to meet the demand in the market on a continuous basis and to avoid being out of stock. Concern also has to sell finished goods on credit due to competition which necessitates the money tied up in debtors. Y and bills receivables. In addition to all these, concerns have to necessarily keep cash to pay the manufacturing expenses, etc., and to meet the contingencies.

Permanent and Temporary Working Capital Working Capital in a business is needed because of the operating cycle.

But the need for working capital does not .come to an end after the cycle is completed. Since the operating cycle is a continuous process, there remains a need for a continuous supply of working capital. However, the amount of working capital required is not constant. throughout the year, but keeps fluctuating.

On the basis of this concept, working capital is classified into two types:
(a) Permanent Working Capital: The need for working capital or current assets fluctuates from time to time. However, to carry on day-to-day operations of the business without any obstacles, a certain minimum level of raw- materials, work-in-progress, finished goods and cash must be maintained on a continuous basis. The amount needed to maintain current assets on this minimum level is called permanent or regular working capital. The amount involved as permanent working capital has to be met from long-term sources of finance, e.g., capital, debentures, long-terms loans, etc.

(b) Temporary or Variable Working Capital: Any amount over and above the permanent level of working capital is called temporary, fluctuating, or variable working capital. Due to seasonal changes, the level of business activities higher than normal during some months of the year, and therefore, additional working capital will be required along with the permanent working capital. It is so because during peak season, demand rises and more stock is to be maintained to meet the demand.

Similarly, the amount of debtors increases due to excessive sales. Additional working capital thus needed is known as temporary working capital because once the season is over, the additional demand will be no more.’The need for temporary working capital should be met from short-term sources of finance, e.g. short-terms loans, etc. So that it can be refunded when it is not required.

Both types of working capital are necessary to run the business smoothly. The distinction between permanent and temporary working capital is illustrated in the following diagram:
Class 12 Business Studies Important Questions Chapter 9 Financial Management 2
Diagram: Showing Permanent and Temporary Working Capital.

The above diagram shows that permanent working capital remains the same throughout the year, while temporary working capital is fluctuating in accordance with seasonal demand.

However, in case of an expanding concern, the need for permanent working capital may not be constant and it would be increasing.

Therefore, the permanent working capital line also may not be horizontal and it will go on rising as illustrated in the following diagram:
Class 12 Business Studies Important Questions Chapter 9 Financial Management 3
Diagram: Showing Permanent and Temporary Working Capital in a Growing Concern

Question 5.
Define the term ‘Cost of Capital’. Also, explain the Significance of the cost of capital.
Answer:
The cost of capital of a firm is the minimum rate of return expected by its investors. The capital used by a firm may be in the form of equity shares, preference shares, debts, and retained earnings. The cost of capital is the weighted average cost of these sources of finance used by the firm. The concept of cost of capital occupies a very important role in financial management because investment decisions are based on it. If a firm is not able to achieve its cost of capital the market value of its shares will fall.

Definition:
Cost of capital for a firm may be defined as the cost of obtaining the funds, i.e., the average rate of return that the investors in a firm expect, for investing funds in the firm.

It is also referred to as cut-off rate,-target rate, hurdle rate, the minimum required rate of return, etc.

Some of the important definitions of cost of capital are stated below:

  • “The cost of capital is the minimum required rate of earnings or the cut-off rate of capital expenditures.” – Ezra Salomon
  • “The cost of capital is the minimum rate of return which a firm requires as a condition for undertaking as an investment.” – Milton H. Spencer
  • “Cost of Capital represents a cut-off rate for the allocation of capital to investments of projects. It is the rate of return on a project that will leave unchanged the market price of its securities.” – James C. Van. Horne
  • “The Cost of Capital is the rate of return a company must earn on an investment to maintain the value of the company.” – M. J. Fordon
  • “A firm’s so-called cost of capital – commonly expressed as an annual percentage figure – is simply that rate of return which its assets must produce in order to justify raising the funds to acquire them.” – W. G. Lawpllen

Thus, on the basis of the above definitions, we can say that cost of capital is the minimum rate of return that a firm, must and, is expected to earn on its investments so as to maintain the market value of its shares.

Significance of the Cost of Capital: The concept of cost of capital is very important in making all the financial decisions of the firm. No financial decision is possible without the use of the cost of capital. Some important uses of cost of ’ capital are:
1. Helpful in Designing the Capital Structure: The concept of cost of capital plays a vital role in designing the capital structure of a company. The capital structure of a company consists of different sources of capital such as equity capital, retained earnings. Preference capital and debt capital. These sources differ from each other in terms of their respective costs. As such a company will have k to design such a capital structure that minimizes the cost of capital. Hence, the calculation of the cost of capital of different sources of capital is very essential to design an optimum capital structure.

2. Helpful in taking Capital Budgeting Decisions Capital budgeting is the process of decision making regarding the investment of funds in long-term projects of the company. The concept of cost of capital is very useful in making capital budgeting decisions $ because the cost of capital is the minimum required rate of return on an investment project. Also, a Finn must not invest in those projects which generate a return less than the cost of capital incurred for its financing.

Net Present Value (NPV) and Internal Rate of Return (IRR) are two important methods used in capital budgeting. Both of these methods are dependent upon the use of the cost of capital. In the NPV method, a project is accepted if its NPV is positive. The project’s NPV is calculated by discounting its cash flows at the cost of capital rate. Under the IRR method, the cost of capital is used as a minimum required rate of return. Hence, the cost of capital serves as a decision criterion for taking capital budgeting decisions.

3. Helpful in Evaluation of Financial Efficiency of Top Management: The concept of cost of capital can be used to evaluate the financial efficiency of top management. Such an evaluation will involve a comparison of the projected overall cost of capital with the actual cost of capital incurred by the management. Lower the actual cost of capital better is the financial performance of the management of the firm.

4. Helpful in Comparative Analysis of Various Sources of Finance: Cost of capital to be raised from various sources goes on changing from time to time. Calculation of cost of capital is helpful in the analysis of the usefulness of various sources of finance. A particular source of finance may be encouraged or discouraged on the basis of its changed cost.

5. Helpful in taking other Financial Decisions: The cost of a capital concept is also useful in making other financial decisions such as dividend policy, rights issue, working capital decisions, and capitalization of profits.

Question 6.
Explain the various factors affecting working capital requirements.
Answer:
Factors affecting working capital requirements:
1. Nature of Business: The basic nature of business influences the amount of working capital required. A trading organization usually needs a lower amount of working capital compared to a manufacturing organization. This is sales can be effected immediately upon the receipt of materials, sometimes even before that. In a manufacturing business, however, raw material needs to, be converted into furnished goods before any sales become possible. Other factors remaining the same, trading business requires less working capital. Similarly, service industries that usually don’t have to maintain inventory require less working capital.

2. Scale of operations: For an organization that operates on a higher scale of operations, the quantum of inventory, debtors that are required is generally high. Such organizations, therefore require a large amount of working capital as compared to the organizations which operate on a lower scale.

3. Business Cycle: Different phases of business cycles affect the requirement of working capita! by a firm. In case of a boom, the sales, as well as production, are likely to be higher, and therefore higher amount of working capital is required. As against this, the requirement for working capital will be lower during the period of depression as the sales as well as production will below.

4. Seasonal factor: Most businesses have some seasonality in their operations. In peak season, because of a higher level of activity, a higher amount of working capital is required. As against this, the level of activity, as well as the requirement for working capital, will be lower during the lean season.

5. Production Cycle: The production cycle is the time span between the receipt of raw materials and their conversion into finished goods. Some businesses have a longer production cycle while some have a shorter one. Duration and the length of the production cycle affect the number of funds required for raw materials and expenses. Consequently working capital requirement is higher in firms with longer processing cycles and lower in firms with shorter processing cycles.

6. Credit Allowed: Different firms allow different credit terms to their customers. These depend upon the level of competition that a firm faces as well as the creditworthiness of its clientele.

A liberal credit policy results in a higher amount of debtors, increasing the requirement of working capital.

7. Credit Availed: Just as a firm allows credit to its customers it also may get credit from its suppliers. To the extent, it avails the credit on its purchases, the working capital requirement is reduced.

8. Operating Efficiency: Firms manage their operations with varying degrees of efficiency. For example, a firm managing its raw materials efficiently may be able to manage with a smaller balance. This is reflected in a higher inventory turnover ratio. Similarly, a better debtors turnover ratio may be achieved reducing the amount tied up in receivable. Better sales effort may reduce the average time for which finished goods inventory is held. Such efficiencies may reduce the level of raw materials, finished goods, and debtors resulting in the lower requirement of working capital.

9. Availability of raw material: If the raw materials and other required materials are available freely and continuously, lower stock levels may suffice. If however, raw materials do not have a record of uninterrupted availability, higher stock levels may be required. In addition, the time lag between the placement of the order and actual receipt of the materials (also called lead time) is also relevant. The higher the lead time, the higher the quantity of material to be stored and the higher is the amount of working capital requirement.

10. Growth Prospects: If the growth potential of concern is perceived to be higher, it will require a higher amount of working capital so that it is able to meet higher production and sales target whenever required.

11. Level of Competition: A higher level of competitiveness may necessitate higher stocks of finished goods to meet urgent orders from customers. This increases the working capital requirement. Competition may also force the firm to extend liberal credit terms.

12. Inflation: With rising prices, higher amounts are required even to maintain a constant volume of production and sales. The working capital requirement of a business thus becomes higher with a higher rate of inflation. It must, however, be noted that an inflation rate of 5%, does not mean that every component of working capital will change by the same percentage. The actual requirement shall depend upon the rates of price change of different components (e.g. raw materials,’ labor cost, finished goods.) as well as their proportion in the total requirement.

Financial Management Important Extra Questions Long Answer Type

Question 1.
Explain the various determinants of the financial needs of a business?
Answer:
Determination of Financial Needs of a Business
or
Assessing Funds Requirements
Answer:
Estimating or determining the financial requirements of the business is one of the main objectives of financial planning. Before raising funds, it is essential that the requirement of funds be correctly estimated. In the absence of correct estimates, the firm may suffer either from inadequate or surplus funds. If the funds are short of its requirements, the firm will not be able to meet its day-to-day expenses and pay the short-term and long-term liabilities on time.

On the other hand, if the funds are in excess of the requirements of the business, they will remain idle and will reduce the profitability of the business. Hence, the estimates should be made in a way that all financial requirements are properly satisfied.

Funds requirements of a business can broadly be classified into two main categories. They are:

  1. Fixed Capital Requirements, and
  2. Working Capital Requirements.

Assessment of Fixed Capital Requirements: Fixed capital is the capital that is meant for fulfilling the permanent or long-term needs of the business. In the words of Shubin, “Fixed capital is the funds required for the acquisition of those assets that are to be used over and over for a long period.”

Fixed capital is required for acquiring fixed assets. Fixed assets may include the following:

  1. Tangible assets such as land, buildings, plant and machinery, furniture, etc.
  2. Intangible assets such as goodwill, patents, copyrights, etc.

A certain amount of fixed capital is also required for meeting certain expenditures not leading to the creation of an asset like research expenses, promotional expenditure incurred for the establishment of business, share issue expenses, underwriting commission, etc. The requirement of funds for these expenditures is long-term and hence the funds required in respect thereof are also included under fixed capital.

Every business needs a fair amount of fixed capital to be invested in fixed assets so as to create production or business facilities. For a new business, the fixed capital is needed in the beginning because fixed assets are needed at the time of promoting or establishing the business. For an existing business fixed capital is required for the development and expansion of the business. Hence, it is essential to have an adequate amount of fixed capital in the business.
The assessment of fixed capital requirements for a new business can be made by preparing a list of fixed assets needed by the business.

The list is prepared by the promoters by studying similar units and by taking advice from technical experts. The estimation of cost of land can be made from property dealers, estimation regarding the cost of building can be made with the help of building contractors and the cost of machinery can be ascertained from the suppliers of the machinery. Similarly, the amount to be paid for goodwill, patents, trade-marks, etc. can also be estimated.

Factors Affecting the Estimation of Fixed Capital/Fixed Assets Requirements: Factors that affect the estimation of Fixed Capital or Fixed assets requirements can be studied under two heads
(a) Internal Factors and
(b) External Factors.

(a) Internal Factors:
1. Nature of Business: Certain types of businesses require heavy investment in fixed assets, while others do not. Usually, the manufacturing concerns require more fixed assets than trading concerns. Similarly, public utility undertakings like railway, electricity, water supply, etc. require huge funds to be invested in fixed assets.

2. Size of Business: Larger the size of a concern, the greater will be the requirement of fixed capital. Also, in larger concerns, most of the activities are performed with the help of automatic machines. As such, they require a huge investment in fixed assets.

3. Types of Products: A concern that manufactures simple consumer products such as soap, oil, etc. will need a lesser amount of fixed capital in comparison to a concern that manufactures complicated products such as motorcycles, cars, etc.

4. Activities Undertaken by the Enterprise: A concern that is engaged in the manufacturing of all parts of a product by itself will require a greater amount of fixed capital as compared to a concern that gets most of the parts manufactured from outside and merely assembles them. Similarly, if a concern itself manufactures and markets its products, it will require more amount of fixed capital as compared to a concern that is engaged only in the manufacturing or only in marketing activities.

5. Mode of Acquisition of Fixed Assets: If some of the fixed assets are available on the lease or on hire, a lesser amount of fixed capital will be required. On the contrary, if all the fixed assets are to be purchased on immediate cash payment, a larger amount of fixed capital will be needed.

6. Acquisition of Old Assets: In certain industries, old plant and machinery may be available at sufficiently reduced prices and which can be used ‘satisfactorily. It would reduce the requirement of fixed capital to a great capital to a great extent. But the old plant and machinery should be used in the industries where the technological changes are moderate or slow.

7. Availability of Fixed Assets of Concessional Rate: In some areas, the Government provides land and other equipment at concessional rates to promote balanced industrial growth. In such a case, the requirement of fixed capital is reduced.

(b) External Factors:
1. General Economic Outlook: If the economy is recovering from depression and the level of business activity is expected to rise, the requirement for fixed assets will also rise and hence the need for fixed capital will also rise.

2. Technological Changes: If rapid technological innovations are taking place in an industry, the need for fixed capital will be larger because the old and out-dated machinery will have to be replaced by new ones.

3. Degree of Competition: The degree of Competition also affects the Fixed Capital-requirements. If there is a lot of competition in some industries, the need for fixed capital will be more because if some firms go on adopting the new technology, the others have to follow them.

4. Shift in Consumer Preferences: If the consumer preferences go on changing in some industries, the need for fixed capital will be more because the firm will have to produce new varieties accordingly, which require more investment in fixed assets.

Assessment of Working Capital Requirements: After the assessment of fixed Capital, funds required for working capital are assessed. The term ‘Working Capital’ is used in two ways.

In one sense it denotes the ‘total current assets’ whereas in another sense it is regarded as the excess of current assets over current liabilities. Current assets include cash, receivables (i.e., debtors and bills receivables), stock, etc. The amount required to be invested in current assets differs from one business to another. The amount depends on various factors such as nature and size of the business, duration of the production cycle, rapidity of turnover, credit policy, the quantity of stock, seasonal fluctuations, rate of growth, etc.

Working capital may be fixed or fluctuating. Fixed working capital refers to the minimum amount which would always be invested in raw materials, work-in-progress, finished goods, receivables, and cash balance. This amount is absolutely essential throughout the year on a continuous basis to maintain a desirable level of business activity. The amount required for fixed working capital mainly depends on the duration of the production cycle.

The cycle starts from the purchase of raw material; then the raw material is converted into finished goods by incurring labor and other costs. On sale, these finished goods are converted into debtors and lastly, the firm will again have cash when the debtors pay. The length of the production cycle (i.e., the length of time between the purchase of raw material and receiving cash from debtors) will determine the quantum or requirements of fixed working capital. The longer the cycle, the higher will be the requirements of fixed working capital.

The requirement of working capital over and above the fixed working capital is known as fluctuating working capital. It keeps on fluctuating from time to time according to the change in the level of business activities. For instance, during peak season, due to intensive sales, more funds are blocked in stocks and debtors and thus more amount will be required for fluctuating working capital.

The total amount of working capital can be estimated by estimating the needs of working capital for the following:

  1. For maintaining adequate stock
  2. For receivables.
  3. For paying day-to-day expenses
  4. For contingencies

1. For maintaining adequate stock: Every industrial undertaking is required to maintain a minimum stock of raw materials, work in progress, and finished goods. The requirement of the stock is determined by various factors like volume of production, the length of the production cycle, and the period for which the finished goods have to remain in a warehouse before they are sold.

2. For receivables: Finished goods may be sold for cash or on credit. Credit sales take the form of receivables (i.e., debtors and bills receivables). The amount is tied up in receivables until cash is realized from them. The amount tied up in receivables depends upon a number of factors such as quantum of credit sales, credit period allowed, the efficiency of the debt collection system, etc. For example, if a firm changes its credit period from 30 days to 60 days, the amount tied up in debtors will go up, and consequently, the need for working capital will also increase by a similar amount.

3. For paying day-to-day expenses: A firm has to carry some minimum cash balance to make payment for wages, salaries, and other expenses throughout the year. A proper cash balance is also maintained to avail of the cash discounts facilities offered by proper cash balance is also maintained to avail of the cash discounts facilities offered by the suppliers.

4. For contingencies: A minimum cash balance is also maintained for meeting unseen contingencies so that the business successfully sails through the period of crisis.

Thus, the overall financial needs of a business can be determined, by assessing the needs for fixed capital and working capital separately and then by adding the two.

Question 2.
Define the term ‘Over-Capitalisation’ and ‘Under Capitalisation’ and their causes?
Answer:
Over Capitalisation: Quiet often, the term ‘Over-Capitalisation’ is misunderstood to mean the excess of capital. But in actual practice, over-capitalized concerns have been found short of funds.

In fact, over-capitalization refers to that state of affairs where a company earns less than what should have earned at a fair rate of return on the capital invested in it. In other words, if a company is continuously unable to earn a fair rate of return on its capital, it is termed an over-capitalized company.

In the words of Bonneville Dewey, ” When a business is unable to earn a fair rate of return on its outstanding securities, it is over¬capitalised.”

According to Gerstenberg, “A corporation is over-capitalized when its earnings are not large enough to yield a fair return on the number of stocks and bonds that have been issued or when the amount of securities outstanding exceeds the current value of assets.”

The same view has been expressed by Harold Gilbert in these words, “When a company has consistently been unable to earn the prevailing rate of return o.n its outstanding securities (considering the earning of similar companies in the same industry and the degree of risk involved) it is said to be over-capitalized.”

It is clear from the above definitions that the situation of over¬capitalisation arises due to a fall in the earning capacity of the business. On account of this, the earnings will not be sufficient to give a reasonable return on capital employed in it. For example, a company is earning a profit of Rs. 8,00,000 on a total capital investment of Rs. 80,0, 000. In case the normal, rate of return prevailing in the market is 10%, this company will be said to be fairly capitalized. However, if it earns only Rs. 2,20,000 while the normal rate is 10%, the company will be said to be over-capitalized because it will be able to give a return of only 6% on the total capital employed.

In order to ascertain whether a company is earning a fair return or not, the rate of return earned by the company should be compared with similar firms in that industry. If the company’s rate of return is t .substantially less than the average rate earned by other firms, will indicate that the company is unable to earn a fair return on the capital 1 employed in it. It may also be noteworthy that a company will be said to be over-capitalized only when it is continuously unable to earn fair income over a long period of time. If its earning is reduced temporarily, owing to the occurrence of abnormal events like strikes, lockouts, etc. the company will not be called over-capitalized.

Causes of Over-Capitalisation:
Following are some of the important causes of over-capitalization:-
1. Over-Issue of Capital: If a company raises more capital than it can profitably use, there will be a large number of idle funds will the company. Because of idle funds, the earning capacity of the company will be reduced. This leads – to the situation of over-capitalization because the company will have to pay dividends on idle capital too. Hence, the rate of dividend will fall which in turn leads to a fall in the market price of its shares.

2. Promotion of the Company with Inflated Assets: A company will fall prey to over-capitalization if it is promoted with assets purchased at excessive prices, the reason is that such prices of the assets do not fear any relation to their earning capacity. Such a situation arises particularly when a partnership firm or private company is converted into a public company and in that process, their assets may be transferred to the public company at price higher than their real values. Sometimes, the promoters also transfer their property to the new company at inflated prices.

3. Promotion or Expansion of the Company during Boom Period: If a company is formed or expanded during the boom period, it; may becomes a victim of over-capitalization. The reason is that the price paid for assets will be quite high. When the boom disappears the real value of such assets will decline to a great extent whereas they will be shown in the books at their original values. Such a company is over-capitalized because its earning will fall due to depression but the assets and capital will be shown in the books in previous figures.

4. High Promotion Expenses: A certain degree of over-capitalization may be caused due to the. fact that the promoters have incurred heavy expenses on the promotion. of the company, a huge amount may have been spent on issue and underwriting of shares and the promoters may have taken a fabulous. remuneration for the services rendered by them. A major part of the earnings of the company will be utilized to write off these expenses and consequently, the company will not be able to pay fair dividends on its shares.

5. Over-estimation of Earnings at the Time of promotion: In case of a new concern,-the amount of capitalization is determined on the basis of estimates of future earnings. However, if it is found that the actual earning is less than the estimated earning, it will lead to a situation of over-capitalization. For example, if a company’s annual earnings were estimated at Rs, 50,000 and its current rate of return (or N capitalization rate) is 10% its, capitalization will be fixed at Rs. 5,00,000. Subsequently, it was found that the company actually earned (Rs. 40,000. On this basis, the company’s capitalization should have been: fixed at Rs. 4,00,000. Thus, the company will be over-capitalized by 4 Rs. 1,00,000.

6. Under-estimation of Rate of Return at the Time of Promotion: A concern may have correctly estimated the number of its earnings, but it may have under-estimated its rate of return (i.e., capitalization, rate). For example, a company’s annual earnings were estimated at ‘ Rs. 50,000 and the rate of return were fixed at 10%. By applying this rate the company’s capitalization was worked out at Rs. 5,00,000. Subsequently, it was found that the actual rate of return was 12.5%, and hence the amount of capitalization should have been fixed at Rs. i.e., Rs. 50,000 × 100. 12.5 Obviously, there is over-capitalization to the extent of Rs.1,00,000.

7. Shortage of Capital: Sometimes, the shortage of capital may also lead to over-capitalization. It may happen when the promoters underestimate the requirements of capital and raise less capital in relation to the needs of the business. In such a case the company will be forced to borrow a large sum of money at an unreasonably high rate of interest. A major part of the earnings will be absorbed by the amount of interest, leaving little for the shareholders. This will bring down the value of shares leading to over-capitalization.

8. Inadequate Depreciation: If a company does not make sufficient provisions for depreciation and replacement of assets, it will find after some time that the earning capacity of the assets is diminished leading to a fall in its earnings. This is yet another case of over-capitalization.

Under-capitalization: The term ‘under-capitalization’ does not mean a shortage or inadequacy of capital. The term is just reverse to over-capitalization. In the words of Greenberg:

“A corporation may be under-capitalized when the rate of profits, it is making on total Capital, is exceptionally high in relation to the return enjoyed by similarly situated companies in the same industry, or when it has too little capital with which to conduct its business.”

In simple words, under-capitalization is a state of affairs when the capital or resources of the company are being utilized more efficiently. As a result, the company succeeds in continuously earning an abnormally high rate of return on the capital employed in it. Such a company declares a high rate of dividend in comparison to the prevailing rate and the market value of its shares exceeds their book value. Thus under-capitalization refers to the sound financial position and good management of the company.

Causes of Under-Capitalisation:
The following are the important causes of under-capitalization:
1. Under-Estimation of Capital Requirements: At the time of promotion, the promoters may under-estimate the capital requirements of the company. This results in a situation of under-capitalization at later stages when more capital is required.

2. Under-Estimation of Earnings: Sometimes at the time of promotion, the future earnings of the company are under-estimated and the company is capitalized accordingly. If afterward it is found that the actual earnings are far in excess of the estimates, the company may find itself in a situation of under-capitalization.

3. Over-Estimation of Rate of Return at the Time of Promotion: Sometimes a concern estimates its income correctly but it over-estimates its rate of return (i.e„ capitalization rate). For example if a company’s earnings were estimated at Rs. 60,000 and the rate of earnings were fixed at 15%. By applying this rate the capitalization was fixed at‘Rs. 4,00,000 (i.e., Rs. 60,000 × \(\frac{100}{15}\)). Subsequently, it was ascertained that the actual rate was 10% and hence the amount of capitalization should have been Rs, 6,00,000 (i.e., Rs. 60,000 × \(\frac{100}{10}\)). Thus, the company is under-capitalized by Rs. 2,00,000.

4. Promotion of Company During Deflation: Companies that are floated under recessionary conditions often experience under-capitalization after the recession is over. This is because of two reasons. Firstly, during recession assets are purchased at a price that is must lower in comparison to their earning capacity. Secondly, companies established during a recession are capitalized at a low figure anticipating low earnings but when the recession is over earnings increase and the company becomes under-capitalized.

5. Conservative Dividend Policy: Certain companies follow a policy of declaring low dividends and plowing back a major part of their earnings. They build up large funds for replacement, renovation, and expansion. The result of such a policy is reflected in high earnings which is a situation of under-capitalization.

6. High Level of Efficiency: In a company where the management is very efficient, the company may operate on a high efficiency even with a meager amount of capital. Over a period, earning the position of the company will improve and it will become under-capitalized.

Entrepreneurship Development Class 12 Important Extra Questions Business Studies Chapter 13

Here we are providing Class 12 Business Studies Important Extra Questions and Answers Chapter 13 Entrepreneurship Development. Business Studies Class 12 Important Questions are the best resource for students which helps in class 12 board exams.

Class 12 Business Studies Chapter 13 Important Extra Questions Entrepreneurship Development

Entrepreneurship Development Important Extra Questions Short Answer Type

Question 1.
What is the relationship between innovation and entrepreneurship?
Answer:
Creativity is the seed that inspires entrepreneurship. Innovation is the process of entrepreneurship. Drucker agrees and elaborates “Innovation” is the means by which the entrepreneur either creates new wealth-producing resources or endorse existing resources with enhanced potential for creating wealth.

It is important to recognize that innovation implies action, not just conceiving new ideas when people have passed through the illumination and verification stages of creativity, they may have become inventors, but they are not yet innovators. The diff. between invention and innovation is shown as under:
Class 12 Business Studies Important Questions Chapter 13 Entrepreneurship Development 1

Question 2.
Explain the fundamentals of a feasibility plan for the success of a business enterprise.
Answer:
A feasibility plan encompasses the full range of business planning activities, but it seldom requires the depth of research of detail expected for an established enterprise.

Every new business is unique. Each will have something that sets it apart from others even if it is no more than the personality of an entrepreneur. For that reason, no plan is going to provide an absolute prescription of success. A feasibility plan is an outline of potential issues to address and a set of guidelines to help an entrepreneur make better decisions.

Developing a Good Plan: Feasibility plans usually are written for investors and lenders, and being aware of this audience often leads to over-optimistic presentations by entrepreneurs who “hard sell” their business concepts. Occasionally this tactic may attract investors and help secure loans, but it will have little value as a management tool for the founder. Writing an honest plan with well-supported information will benefit everyone.

A well-written plan should be succinct, clearly identifying products, services, markets, and the founder. A feasibility plan does not have to be “stick”, but it does have to be prepared in a quality manner. The plan should be easy to read, complete and accurate.

There should be no misspelling, improper grammar, or mistakes ‘ in data. Effective plans avoid emotion-packed phrases like “This can’t miss!” or everybody needs this!” They also avoid abstract language. Entrepreneurs who know have to write a good plan will avoid saying they “think” there is a market or they “believe” a product will work. Instead, they will use facts to support their assertions.

Protecting the Business: Since business plans are used to attract investors and lenders, many copies are circulated. Wide circulation can be dangerous if the plan contains sensitive information consequently, It is wise to include a strong “nondisclosure statement” on the cover page that states information in the plan is proprietary and cannot be copied, disclosed, shared, or otherwise compromised.

Many entrepreneurs also assign an index number on each copy in addition to a signature line for each recipient. This constitutes an agreement on the nondisclosure terms and provides a reference number for documenting circulation. Although this procedure may not always protect entrepreneurs from having their ideas stolen, it can be a strong deterred.

Making the Plan Readable: A thorough business plan often has more than 50 pages, but many plans based on easily understood business concepts may be less than 20 pages long. Plans for complex enterprises requiring extensive documentation are much longer. If there is a choice keep it short potential investors and lenders receive many proposals, but they rarely read more than the first few pages.

If the concept is intriguing, they spend more time probing financial data. It can be quite disturbing to an entrepreneur who has spent months writing a good plan to watch a loan officer spend five minutes reading the front page and skimming projections. Therefore, it is even more important to be convincing in the opening pages.

For those few enterprises that capture an investor’s attention (or get past the junior loan officer), there is a more complete study. This means that an entrepreneur must be very careful to capture a reader’s attention early, yet provides thorough information for a detailed analysis that occurs later.

Question 3.
Explain the role of entrepreneurship and relations between Entrepreneurship and Management?
Answer:
Entrepreneurship is about business start-ups and renewals. That is, it appears at the time of starting a new business, disappears for some time in the course of stabilizing the venture as an on-going business, and reappears in case there is a need for introducing changes in product, market, technology, structure, and so on. In fact, it is said that everyone is an entrepreneur when he actually ‘carries out new combinations’, and loses the character as soon as he has built up his business when he settles down to running it as other people run their businesses.

In developed countries, the distinction between the entrepreneurial focus on start-ups and managerial focus on routine is so sharp that it is argued that once the project has reached the level of maturity, the entrepreneurs must move out and the managers must come in.

In developing countries, however, the concept of owner-manager seems more apt for entrepreneurship as the entrepreneur remains attached even to the day-to-day operations of the venture. In fact, they’re lacking managerial skills is often forwarded as the cause of business failures. Just as managers are expected to play entrepreneurial roles in times of need, likewise the entrepreneurs must also demonstrate managerial abilities for the success of their ventures. Irrespective of whether the entrepreneur’s power way for the managers or they themselves assume the managerial responsibilities, it is possible to distinguish between the terms of entrepreneurship and management.

Differences Between Entrepreneurship and Management:

No. Basis of Differentiation

Entrepreneurship

Management

1. Focus Business Start-up Ongoing Operations of an existing business.
2. Resources orientation The entrepreneur does not feel constrained by resources. Entrepreneur mobilizes the resources. A manager is constrained by the resources at his disposal
3. Approach to the task Informal Formal
4. Primary motivation Achievement Power
5. Status vis-a-vis the enterprise. Owner Employee
6. Primary economic reward Profit Salary
7. Innovation orientation Challenges the status quo, that is, the existing. Maintains the status quo.
8. Risk orientation Risk-taker Risk-averse
9. Approach to decision-making Driven by inductive logic and personal courage and determination. Driven by deductive logic and research.
10. Scale of operations Small business Large business
11. Primary skills requirement Opportunity spotting, initiative, resource negotiation. Organizing, systems design and operating procedures, people management.
12.. Specialisation orientation Generalist; has to know and do all the trades by himself. Specialist.

Question 4.
Explain the role and functions of an Entrepreneur in relation to the enterprise?
Answer:
Roles and functions of the Entrepreneur in relation to the enterprise: Developing Exchange Relations

  1. Perceiving market opportunities
  2. Gaining command over scarce resources
  3. Purchasing inputs
  4. Marketing of products and responding to competition. Political Administration
  5. Dealing with public bureaucracy Capprovals, concessions, taxes)
  6. Managing human relations within the firm
  7. Managing customer and supplier relations. Management Control
  8. Managing Finance
  9. Managing production Technology
  10. Acquiring and overseeing the assembly of the factory
  11. Industrial engineering (minimizing inputs with a given production process)
  12. Upgrading the production process and product quality.
  13. Introducing new production techniques and products.

Note: The scope of the entrepreneurial functions varies with the level of economy in which the entrepreneur operates; the scale of production/operations; and entrepreneurs’ comparative efficiency in utilizing managerial employees. In developed countries, entrepreneurship assumes upon itself the responsibility of introducing innovation and after some time, pave way for the managers.

In large-scale organizations, entrepreneurs provide leadership and there is a team of managers to look after specific aspects of the enterprise. Likewise, those entrepreneurs who have the ability and willingness to delegate may concentrate on the select few, strategic aspects of .enterprise.

Question 5.
Explain in brief the functions of an entrepreneur especially to the Economic Development of the enterprise.
Answer:
Functions of entrepreneurs in relation to Economic Development: You are aware that entrepreneurs “organize” the production process. In the absence of this function, all other resources, namely land, labor, and capital would remain idle. They may not be inventing/ discovering the products, their role in the commercial exploitation of the advancements in science and technology via the organization of the productive apparatus makes the other resources productive and useful. So much so that it is said that in the absence of entrepreneurial intervention, every plant would remain a weed and every mineral would remain a rock!

1. Contribution of GDP: Increase in the Gross Domestic Product or GDP is the most common definition of economic development. You are aware that income is generated in the process of production. So, entrepreneurs generate income via the organization of production be it agriculture, manufacturing, or services.

You are also aware that income generated is distributed among the factors of production where land gets rent, labor gets wages and salaries, capital gets interested and the residual income accrues to the entrepreneur in the form of profits. As rent and interest accrue to those few who have land and capital-respectively whereas large masses are destined to earn their incomes via wage employment, the biggest contribution of entrepreneurship lies in capital formation and generation of employment.

2. Capital Formation: The entrepreneurial decision, in effect, is an investment decision that arguments the productive capacity of the economy and hence results in capital formation. In fact, GDP and capital formation are related to each other via capital-output Ratio (COR); more precisely Incremental Capital Output Ratio (I.COR) measures the percentage increase in capital formation required to obtain a percentage increase in GDP. So, if a country desires to grow @ 10:0%. p.a. and its ICOR is 2.6%. p.a. Entrepreneurs, by investing their own savings and informally mobilizing the savings of their friends and relatives contribute to the process of capital formation. These informal funding supplements the funds made available by the formal means of raising resources from banks, financial institutions, and capital markets.

3. Generation of Employment: Every new business is a source of employment for people with different abilities, skills, and qualifications. As such entrepreneurship becomes a source of livelihood to those who do neither have the capital to learn interest in nor have the land to earn rent. In fact, what they earn is not only a livelihood or means of sustenance but also a lifestyle for themselves and their families as well as personal job satisfaction. As such entrepreneurs touch the lives of many, directly as well as indirectly.

4. Generation of Business opportunities for others: Every new business creates opportunities for the suppliers of n (this is referred to as backward linkages) and the marketers of the output (what is referred to as forwarding linkages). As a pen manufacturer you would create opportunities for refill manufacturers as well as wholesalers and retailers of stationery products. These immediate linkages induce further linkages.

For example, greater opportunities for refill manufacturers would mean the expansion of business for ink manufacturers. In general, there are greater opportunities for the transporter, advertisers, and, so on. So, via a chain-reaction, entrepreneurship provides a spur to the level of economic activity.

5. Improvement in Economic Efficiency: You are aware that efficiency means to have greater output from the same input. Entrepreneurs improve economic efficiency by:
(a) Improving processes, reducing wastes, increasing yield, and,
(b) Bringing about technical progress, that is, by altering labor-capital ratios. You are aware that if labor is provided with good implements (capital), its productivity increases.

Question 6.
What are the various success factors for entrepreneurs?
Answer:
Success factors for entrepreneurs: Several success factors are apparent from research on innovation and entrepreneurship. We now have fairly solid evidence of what it takes to succeed in a new venture, and although there will always be exceptions, most new ventures succeed because their founders are capable individuals.

1. The Entrepreneurial Team: At the top of the success factor list is the “entrepreneurial team”. The term team is used because, more often than, not entrepreneurs do not start businesses by themselves; they have teams, partners, close associate, or extensive Networks of advisers. In major studies of entrepreneurs in the United States, Canada, and Europe, between 60 and 70 percent of all technology-based ventures were started by the founder with at least one panner or cofounder. Those in nontechnical enterprises (e.g., personal services or merchandising) were less likely to have partners or cofounders; yet they were well networked with associates or expert advisers.

An entrepreneurial team is usually headed by an individual who provides the critical profile of success. This focal entrepreneur typically has an above-average education, with about 35 percent of technical entrepreneurs holding graduate degrees. Most entrepreneurs started their businesses when they were in their 30s, and they had solid job experience. Also, nearly two-thirds of those studied in the United States had attempted a new venture before, and slightly fewer Canadians had made an earlier attempt of some interest, far less than half of those from Europe had previously tried to start a business.

Most technical entrepreneurs tend to start businesses closely related to what they did in previous career positions. Those in non – technical areas often leverage their experience in marketing, merchandising, or a professional service area such as insurance or finance. We can infer that success is closely tied to a solid knowledge base and substantial experience in related fields of endeavor.

They will also have well-developed social and business relationships, and therefore have a strong foundation for building a team or support network. This finding was reinforced in studies of Silicon Valley firms where researchers found entrepreneurs to have good relationships with vendors, potential customers, financiers, bankers, attorneys, and their competitors.

2. Venture Products or Services: Nearly all successful ventures start small and grow incrementally; few “gear up” with the substantial organization for a big-bang start. Increment expansion of products and services also tends to stay within the bounds of positive cash flow. Products tend to have strong profit potential with high initial margins rather than small margins that require a substantial volume of sales to meet profit objectives. Service businesses retain good margins by effective cost controls and well- monitored overheads.

In each instance, products and services tend to display a distinctive competency in their industries. This is important because very few entrepreneurs start businesses in already competitive situations. This observation relates to an earlier point that we emphasized; Entrepreneurs must assure themselves of a niche for- their services. A corollary to this rule is that successful entrepreneurs should “stick to their knitting” by concentrating initially on one distinct product or service, making it successful before diversifying.

From an investor’s viewpoint, the product or service idea is secondary to the entrepreneur. A popular expression among investors is that they would rather “hack a first-rate entrepreneur with a second-rate product than the other way around”. This guideline does not mean the business concept can be weak, but it does suggest that investors must have considerable confidence in the entrepreneurial team before buying into the venture.

3. Market and Timing: Successful entrepreneurs tend to have a clear vision of both existing and potential customers. A crucial aspect of planning is to have a well-documented forecast of sales based on sensible projections at each stage of incremental growth. A charismatic entrepreneur loaded with talent and a great idea will not convince investors that a venture is Viable without valid market research. There are no shortcuts; innovation requires market demand, not simply a good idea.

The market evolves, and as noted earlier, there are windows of opportunity that can lead to exceptional success. Misjudging those windows can result in dismal failure. The market potential is critically influenced by the timing of new products or services. Timing pertains to when products or services are introduced, how they are priced, how they are distributed, and how they are promoted.

4. Business Ideology: From an entrepreneur’s perspective, every venture has an ideology, a philosophy, or rationale for existing. Although the ideology may be extremely difficult to quantify, it is nevertheless important. A business ideology is defined as a system of beliefs about how one conducts an enterprise.

These beliefs include a commitment to providing customers with value, the ability to take calculated risks, the determination to grow and to control the fate of the business, the propensity to elicit cooperation among team RH, embers, and the perspective of creating wealth, realistically. A business ideology may not be entirely defined by these notions, but failure is often blamed as one of them. For example, rarely do we hear that a business failed because the product was flowed, but more often because the firm lost track of its commitment to customers

Entrepreneurship Development Important Extra Questions Long Answer Type

Question 1.
What is Market Plan? Explain the main Elements of the Marketing Plan?
Answer:
Class 12 Business Studies Important Questions Chapter 13 Entrepreneurship Development 2

The Market Plan: The market plan describes an entrepreneur’s intended strategy, it builds on market research and distinct characteristics of the business to explain how the venture will succeed. Some issues addressed in the research section may be reserved for the market plan, such as describing a market niche. This section usually focuses on specific marketing activities. It describes pricing policies, quality image, warranty policies, promotional programs, distribution channels, and other issues such as service-after-sale and marketing responsibility. These are described as under:

Prices: Well-defined prices are obviously necessary to project sales volume and financial performance. As discussed earlier, prices also indicate quality and product image, and depending on the channels of distribution, prices will reflect the nature of the business. Pricing policies relate to bulk, wholesale, retail, and discount methods used to set prices. Such methods as cost-plus pricing or setting prices to match those of competitors indicate how entrepreneurs will make strategic pricing decisions.

Promotions: Advertising and promotional strategies must be consistent with the product or service image. For example, quality office furniture is not apt to be sold through discount newspaper ads. Choosing proper media for advertising is one aspect of the plan, but introductory strategies should relate to the start-up stage. For example, a new software program may be introduced at computer trade shows and be demonstrated at seminars offered to select clientele.

Software developers may also sponsor business contests, set up displays in book stores or computer retail outlets, or provide educational versions of programs to universities. The promotional mix is determined by a conscious decision, selecting various promotional tools from advertising, personal selling, public relations, point-of-purchase displays, sampling, and direct-mail solicitation, among others.

Distribution Channels: If distribution channels have not been identified earlier, they must be described here. For example, unusual gift items ranging from greeting cards to imported beef fillets are sold through catalogs, but Hallmark opened chain stores in shopping malls nationwide to market gifts and greeting cards. Liz Claiborne, Inc., reached $ 3 billion in sales by positioning fashionable women’s clothing in department stores through regional distribution centers, but recently the company opened a chain of exclusive stores supplemented with catalog sales.”

Service and warranty considerations: Most retail stores offer warranties and service-after-sale guarantees in the event a product requires repair or adjustment. Often the distinguishing characteristic of a car dealership is its service and – warranty policies. Appliance dealers may also base their strategies on follow-up services and warranties. Telemarketing companies invariably offer money-back guaranteed because customers cannot evaluate products before they buy. On the other hand, there are many cash-and-carry discount outlets that sell “seconds” or flowed merchandise, and customers rarely expect warranty service.

Service companies also compete on warranty and service-after-sale policies. Software firms, for example, typically have “hotlines” for answering customers’ inquiries. Because software programs are updated with how or enhanced versions, or trade-in allowance. In estate planning, a recent new service in which consultants help clients plan their investments, service after sale includes periodic reviews of clients portfolios, investment newsletters, and special reports on tax laws and legislative activities.”

Marketing Leadership: The market plan should address the way in which organizational members will be involved in the marketing effort. From a strategic perspective, investors want to know who is going to actually take the lead in making customer sales. If the venture requires a sales force, then issues such as sales training, commission structures, recruitment, and sales management become important.

Investors and lenders are accustomed to seeing too general patterns in poor business proposals that get rejected. First, there are technically competent entrepreneurs who have great ideas but who know every little about marketing. Their plans provide overkill on product attributes but ignore marketing strategies. Second, there are super salespeople with brilliant ideas who are overenthusiastic about projects.

Question 2.
Explain the various factors essential for the success of a business enterprise especially in a service sector/venture.
Answer:
Factors essential in service ventures: There are various critical factors that helped entrepreneurs succeed. The nature of a service venture is different from a Product- based company in that services require exceptional human resource skills. Services can usually be initiated with low-entry capital requirements, but having the right people is vital. A good service idea can also be easily copied; therefore, a competitor flock to a growth market, having committed people will often make the difference between success and failure.

Human-resource issues are far-reaching, but we will explore several focal points for new ventures. During the start-up planning phase, for example, an entrepreneur must ventures. During the start-up planning phase, for example, an entrepreneur must establish a vision that everyone will work to fulfill. This may relate to a distinct competency of quality service in a particular kind of business.

At the outset, entrepreneurs must firmly establish sound policies for customer service. Service firms rely on capable staff, and this need requires skills at hiring, training, and motivating employees. The leadership skills of an entrepreneur are vital to position the service firm for growth. Although we cannot begin to cover each topic in detail, we can describe its importance to new service ventures.

Creating the vision: A good way to fail quickly in a new business is to start without a clear vision. A vision encompasses the value that an entrepreneur will provide for his or her customers, and if that service is achieved, the encompasses the result the entrepreneur will achieve without this basic vision, start a business on a whim is tantamount to shooting dice; you are relying on chance to dictate fate.

Vision is vital but it must be orchestrated through effective planning and a strong commitment to the image of service one wants to project. The image is a result of conscientious planning to provide value to customers there has to be a clarity of direction that provides service goals for human endeavor.

Effective Hiring: Three things are generally needed to get a business started. New ventures need a good product or service based on a sound vision. Sufficient money to pursue that venture, and people-good people.

Research indicates that patterns of employment for most small businesses are relatively fixed at the moment of opening. This generalization is particularly true for personal service enterprises that start and remain small. Consequently, a beauty salon or clothing boutique will open with a few carefully selected employees, and although these employees may be replaced, their numbers and skills will remain fairly constant. Owners are, never the less, responsible for staffing the enterprise no matter how small.

The pattern of employment for smaller enterprises is that owners will initially hire one or two full-time persons and supplement busy seasons usually will be skilled or experienced in the trade. The rest often will have to be trained. Unfortunately, small business owners rarely follow good personnel practices in hiring, and even less often provide adequate training.

Many full-time employees are hired from among friends or family members. This practice provides no assurance of having employees with the required skills or commitment to make a business a success. Part-time employees may come from unemployed walking or students looking for supplemental income. Too many entrepreneurs tend to hire them as a matter of convenience, not as a conscious effort of staff their enterprises.

Perhaps a small firm can survive without systematic hiring and training practices but entrepreneurs will more often experience high turnover among employees who are poorly prepared to do a good job. Also, there is a fundamental problem with finding highly motivated long-term employees because smaller firms can seldom offer high wages, good benefits, or opportunities for advancement.

When the entrepreneur is not in a position to hire or train employees, and when the business is too small to support an organization, another interesting option is to lease personnel. This is a recent innovation in staffing that is, itself, an entrepreneurial service. Unlike temporary service agencies, leasing firms actually hire hundreds of employees, train them, and provide a full range of employee benefits. The leasing company places employees in a client’s firing, thereby relieving an owner of hiring, firing, training, and managing a complex system of compensation and benefits.

For growing companies, increased sales mean changes in human resources and substantial responsibility for attracting, hiring, training, and retaining employees during the early stage of planning, it is important to clearly understand that there is no way companies can grow if entrepreneurs try to do everything themselves. To resolve this difficulty, the first step is for entrepreneurs to purposely describe their roles and how those roles will change with growth.

The second step is to write employee job descriptions for the first stage of business. The third step is to write expanded job descriptions for employees whose jobs will change with growth. These descriptions will help identify how responsibilities will change and therefore opportunities for career development. They will not only attract better- applicants who want a challenge, but also will help clarify how the entrepreneur and the employee want a challenge, but also will help clarify how the entrepreneur and the employee will relate to one another over time.

High-growth enterprises are unlikely to follow the recruiting methods of smaller firms, for two reasons. First, professional and personal service firms that remain localized usually will not need highly skilled individuals They can therefore recruit through local labor markets. Second, small firms that do not intend to grow will not develop management positions for functional specialists in areas such as marketing operations, or finance.

On the other hand, growing firms will need functional specialists, and in high-tech fields, they will often need research scientists engineers, and other technical specialists. For these firms, recruiting through newspaper ads in the local labor market is pointless. Professional and managerial talent is found through national searches professional societies conferences, university placement services, and networking.

The last method, networking is particularly fruitful as entrepreneurs socialize with other entrepreneurs, do business with their counterparts in other firms, and develop contact with suppliers and customers who may become potential applicants. Net-working provides an inside track to key people, but the entrepreneur still has the responsibility to recruit them and help them mold their entrepreneurial careers.

Training: Personal service firms tend to be structured around the skills of the founder. For very small firms, such as independent beauticians or professional photographers, success hinges on the reputation of skilled individuals. When the expansion occurs, owners have two options: they can hire comparably skilled individuals or train apprentices. Either option can be extremely difficult to accomplish.

For example, if a professional photographer wants to hire someone who can reinforce his or her established reputation, it means getting someone as skilled as the founder, and hiring such a person translates to a rather high cost. The new employee will demand a substantial income, and the owner initially will have to share clients and income, if a less skilled, person is hired, then the owner must train the employee. In either case, initial costs can be high, and there will be inefficiencies until the employee becomes proficient transient employees enjoy wages well in excess of the minimum wage required by law. In fact, the minimum wage has become meaningless in these circumstances.

Unlike fast-food restaurants and discount retailers, computer service firms, telecommunication specialists, antique boutiques, health clinics, and many other enterprises must have competently trained employees. Moreover, service and technical ability may be comparably important so that, for example, an owner of an antique boutique may require employees who are at once experienced inexpensive antiques and able to work with knowledgeable customers.

A computer service firm may require employees who are skilled in technical aspects of hardware, who are capable of working with a substantial range of software, and who also have the human relation skills to “service and sell” to end-users that range from high school students to corporate executives. Finding employees with all these attributes can be extremely difficult.

How many skilled computer technicians with sales experience exist? How many experts are therein? antiques who also want a job? The same questions apply to hundreds of professionals in equally diverse fields. A safe assumption, therefore is that ‘ideal’ employees rarely can be found, and it falls to an entrepreneur to find those with potential who can be trained.

Training poses two problems: first learning how to seek out and hire people with potential, and second, establishing effective methods of employee training.

Question 3.
Explain in detail the entrepreneurial values and attitudes.
Answer:
Entrepreneurial values and attitudes: In any explanation of human behavior, the reference to values and attitudes is inevitable.

Values: Values are general ideas about all that is deemed desirable/ important by an individual or a collectivity. The term is used in the plural sense, that is there are various types of values. Because they are general they do not specify how one should act in a particular situation. When the value is shared across the majority of a collectivity be it the organization. Professional body or even a nation in the sense that these are upheld professed, included. Practiced and rewarded, they comprise “culture;” as they take roots. They become so deep-seated that they begin to govern individual behavior. In any discussion on economic development and social change, the role of value is inevitable. –
Class 12 Business Studies Important Questions Chapter 13 Entrepreneurship Development 3

These values underlie the human quest for knowledge, As such, these values imply whether and how must a society respects and rewards pursuits in education in general and science and technology in particular, you would appreciate that a society that value education, science, and technology is like to be more entrepreneurial than the one that does not. An important aspect of these values is avoidance or embracing of uncertainly. Any question science, technology, and even the outcomes of entrepreneurial endeavor are subject to a host of unseen circumstances. Daring and risk-taking become inevitable aspects of the quest for knowledge and all advances.

Aesthetics: It is a scientific fact that man cannot create matter. What he does is that through his ingenuity and creativity he find its want-satisfying properties and change its form to create good and services for consumption and need satisfaction. What, however, we, use is the whole ‘ like figure on the pipe like figure, as you grew up you learned to draw branches and leave to make it look more real; as you perfected the extent and advancement of entrepreneurial activity. It includes respect for nature and concern for maintaining ecological balance, sustainability, and harmony between economic progress and the environment.

Moral Values: Honesty, Fairplay, integrity, peace, truthfulness, commitment, concern for other, etc. are the value that provides us the critical a to judge what is wrong and what is right. One often doubts whether business and morality go together yet one knows that these values are desirable.

In today’s context when business performance is based on the combined efforts of not just the people within the organization but also on the collaborative efforts of the geographically dispersed (in many cases even internationally dispersed) network of suppliers and distributors it -is not the management by controls but management by values that will be effective. Thus nowadays, organizations make conscious efforts in defining, communicating, and encouraging the practice of some core values.

Political values: Democracy is a desirable value as it fasters individual freedom and the right to equality. That is why in the modern interpretation of development, political freedom is also considered as a factor, societies that value individual freedom is fertile ground for entrepreneurship, we have seen that entrepreneurship is a great leveler as it provides the opportunity to rise to those section of the society that does not have recourse to property and interest income.

The socialized face of power reflects the political value of decentralization and empowerment and concern for the upliftment of those who are at the bottom of the pyramid, organization that creates a culture of decentralization and people empowerment become highly dynamic, creative and entrepreneurship individuals can empower themselves by taking recourse to entrepreneurship. The societies that are hierarchically ordered are exhibit high power distance stifle personal growth as they demand obedience to authority and tradition rather than encouraging the habit of questioning suggestion seeking and giving and collaborative problem solving and teamwork.

Social Values: Concern for people in distress wherever they may be is the central value that drives help and rescue operations in case of natural disasters. These values make a man a truly social animal. These values include respect for the norms of collectivity and placing the society ahead of the self. These include being responsive to the needs and expectations of society and fulfilling one’s social responsibilities.

In dealings with people, these values include openness trust, compassion, empathy brotherhood. Solidarity and so on. Social value defined role – variations across family caste and sex. The societies that adhere to rigid stratification of the roles and occupations along these lines are not very suitable for the flowering of entrepreneurship, on the other hand, the societies that allow mobility auger well for the development of entrepreneurship. Entrepreneurship is not and cannot be an exclusive preserve of the privileged few on the basis of their birth in a particular caste family all as a particular sex.

Economic values: These values imply a desire for material well-being by engagement in productive activities and include wealth accumulation and capital formation. Consumption and saving are the two most important behaviors that are governed by these values. Economics value also governs the institution of property – Who owns the society’s resources? How would these resources be distributed? What uses these resources be put to-toward producing “guns” or toward producing “bread”? We have seen that capitalism or free-market economics are a very good ground for the flourishing of individuals’ Entrepreship.

These values then form the basis of exchange within and across economics. These values to an extent determine whether we would follow inward-looking growth or we would proactively integrate with the world economy to benefit from the opportunities arising from globalization.

Together these values comprise a social value system. Some values are interdependent such as democracy freedom and equality, others may be arranged in order of preference, for example in some societies, observing, trading may be considered more important than individual freedom. Different societies ‘ organizations/collectivities, as well as different individuals, may subscribe to different values but once their value system is known it becomes easier to understand and predict behavior.

Values tend to be stable yet the values do and can be changed in fact those societies that are hot able to bring about the mindset/ value system change in sync with the changing times stagnate and lag behind. Are there any values that typify entrepreneurial culture? Alternatively, what are values that an entrepreneurial society “Consistently demonstrates? We take the example of America, inarguably ‘ among the most entrepreneurial societies of the world.

Attitudes: Attitudes refers to tendencies to feel and behave in a particular way toward the object of the attitudes in a consistent manner. It may vary from like/dislike for a thing say, tea, to positive or negative views about a person or a circumstance and maybe a generalized enthusiasm or lack of it for life. (Remember” The glass is half empty “vs. “The glasses half full” “attitudes?)

Attitudes, therefore have three characteristics:
(a) Attitudes persist and are therefore difficult to change.
(b) They lie anywhere on either side of the scale very unfavorable to favorable; and,
(c) They are directed toward something or someone about whom we have strong feelings. Such a feeling may develop as a result of the information or knowledge we may have about the person or the things that do not really matter, whether the information itself was correct or real.

Values represent a sociological variable whereas attitudes have a lot to do with a person’s psyche. Value in as much as they represent what is virtuous or desirable, tends to be positively loaded whereas attitude can be either positive or negative. In terms of their influence on behavior, attitudes seem to be exerting a more proximate influence vis-a-vis values.

Controlling Class 12 Important Extra Questions Business Studies Chapter 8

Here we are providing Class 12 Business Studies Important Extra Questions and Answers Chapter 8 Controlling. Business Studies Class 12 Important Questions are the best resource for students which helps in class 12 board exams.

Class 12 Business Studies Chapter 8 Important Extra Questions Controlling

Controlling Important Extra Questions Short Answer Type

Question 1.
Mention the important features of the control function of management.
Answer:
Characteristics of Control: The Control function of management has the following features:
1. Controlling is a continuous Process: It involves continuous measurement and review of actual performance and results in corrective action based on this review. It may lead to changes in planning, staffing, organizing, etc.

2. Controlling is looking Back: Control leads to the appraisal of past activities. Thus, it is looking back. The shortcomings in the performance of various individuals and departments are revealed by the control process. This is known as feedback information. It will help in knowing the reasons for poor performance. Corrective action can be initiated on the basis of feedback information.

3. Control is forward-looking: Control is said to be forward-looking. It is related to the future as the past cannot be controlled. It is usually preventive as the preserve of the control system leads to minimizing wastages, losses, and deviations from standards. It should be noted that control does not curtail the rights of individuals. It simply keeps a check on the Performance of individuals.

4. Control is Action-Oriented: Control implies taking corrective measures, whenever required. Action is the essence of control. The purpose of control is achieved only when corrective action is taken on the basis of feedback information. If the control does not lead to any action, then a manager can’t ensure actual performance as per standards. A good system of control facilities timely action so that there is minimum wastage of resources, time, and energy.

5. Control is a Pervasive function: It is a basic function of every manger irrespective of his level of authority. It is a follow-up action to the other function of management. Every manager is responsible for controlling the activities of those working under him and taking necessary action whenever necessary.

Question 2.
Explain the term Management by Exception.
Answer:
Management by exception: Effective control can be achieved if critical or key points are identified and close attention is directed to adjustment at these points. This is known as ‘control by exception’. According to this principle, only significant deviations from the standards require management’s attention as they constitute exceptions. An attempt to go through all deviations tends to increase the workload of the Manager and decrease attention on important problems.

For instance, if postal expenses mqt&ase by twenty percent, the deviation is too insignificant to require managerial attention. On the other hand, if labor costs rise by twenty percent, they should receive immediate managerial attention.

The Principle of Management by exception implies that minor deviations from the standards may be ignored or given less attention. This would conserve managerial time, effort, and energy which could be utilized on important matters. But whenever deviations from standards are higher than the accepted level, management must take corrective measures to deal with the situation.

Question 3.
Explain in brief the term Responsibility Accounting.
Answer:
Responsibility Accounting is a system of control where responsibility is assigned for the control of costs. The persons are made responsible for the control of costs. Proper authority is given to the persons so that they are able to keep up their performance. In case the performance is not according to the predetermined standards then the persons who are assigned this duty will be personally responsible for it.

In responsibility accounting, the emphasis is on men rather than on systems e.g. if Mr. A, the manager of a department, prepares the cost budget of his department then he will be made responsible for keeping the budget under control. A will be supplied with full information of cost incurred by this department. In case the cost incurred is more than the budgeted costs, then A will try to find out reasons and take necessary corrective measures. A will be personally responsible for the performance of his department.

Question 4.
Explain in brief the term Zero base budgeting.
Answer:
Zero Based Budgeting Whenever a budget is prepared, past records and experience are taken into consideration. The new budget is prepared on the basis of budgets of the previous period. This method is known as ‘Based Budgeting’. On the contrary, the concept of zero-base budgeting considers the future not a mere projection of the past. As the environment is changing fast, the need to forecast systematically future events arise. For the growth and survival of any business organization, these changes need to be adopted.

Zero-base budgeting is the latest technique and it is meant for a particular period of time. It doesn’t prepare an altogether new budget. Normally, the previous year’s performance is taken as a base for the current year’s budget in budgeting techniques. But in zero-base budgeting, every year is taken as a new year, and the previous year is not taken as a base.

Zero is taken as a base and all budget proposals are considered in the light of present conditions. In zero-based budgeting, the amount to be spent on various activities will depend upon justification given by the manager.

Hence, zero-based budgeting provides flexibility and freedom in allocating the resources and the chances of repletion of weaknesses of the previous year are reduced.

Question 5.
Explain the Modern technique of control by the management information system (MIS) a computer-based technique.
Answer:
Management information system: A management information system (MIS) is a computer-based information system that provides information and support for effective managerial decision-making. A decision-maker requires up to date, accurate and timely information. MIS provides the required information to the managers by systematically Processing massive data generated in an organization. Thus, MIS is an important communication tool for managers.

MIS also serves as an important control technique. It provides data and information to the managers at the right time so that appropriate corrective action may be taken in case of deviations from standards.

MIS offers the following advantages to Managers:

  1. It facilitates the collection, management, and dissemination of information at different levels of Management and across different departments of the Organisation.
  2. It supports planning, decision making, and controlling at all levels.
  3. It improves the quality of information with which a manager works.
  4. It ensures cost-effectiveness in managing information.
  5. It reduces information overload on the managers as only relevant information is provided to them.

Question 6.
Distinguish between Planning and Control.
Answer:
A comparison of Planning and Control:

Planning Control
1. Planning emphasizes more on non-personal, abstract long-range problems. 1. Control emphasizes more and long-range problems.
2. Planning is based on estimates 2. Control is based on estimates.
3. Planning is done by top-level executives. 3. Control is done by top-level executives.
4. Under Planning, the evaluation of results is very difficult. 4. Under Control, the evaluation of results is very difficult.
5. Determination of results takes a very long time. 5. Determination of results takes a very long time.

Question 7.
What are the advantages of Management audit?
Answer:
The main advantages of Management audit are as follows.

  1. It helps to locate present and potential deficiencies in the performance of management functions.
  2. It helps to Improve the Control system to an Organisation by continuously monitoring the performance of Management.
  3. It Improves Coordination in the functioning of various departments so that they work together effectively towards the achievement of organizational objectives.
  4. It ensures updating of existing managerial .policies and strategies in the light of environmental changes.

Conducting a management audit may sometimes pose a problem as there are no standard techniques of management audit. Also, management audit is not compulsory under any law. Enlightened managers, however, understand its usefulness in improving the overall performance of the organization.

Controlling Important Extra Questions Long Answer Type

Question 1.
Explain the essential features of a sound control system. (Imp. Q)
Answer:
Essential features of a sound control system:
A good control system has the following essential features:
1. Clear out objectives: Before Planning a control system, it is essential to know clearly the objectives it will tend to achieve. The standards of performance should be based on these objectives. Thus, the control system should be directed towards the detection of early deviations from the standards to permit effective corrective action. If the standards are arbitrary or subjective in nature, they will not be able to measure deviations properly. Thus, the purpose of control will be lost.

2. Suitability: Control techniques should be appropriately designed to suit the nature of the activities being controlled. This means that different types of activities need different types of control techniques. The control technique should be tailored to reflect the performance of all types of operations, say, production or sales.

It is also essential that the overall control system should be appropriate for the organization. A control system that is good for a small organization may be inadequate for a big organization.

3. Simplicity: A good system of control should be simple and easy to understand. The employees must know what is expected of them and how their performance will be evaluated. If the employees don’t understand the standards of performance properly, the actual performance might turn out to be very poor.

4. Economy: The system of control must be worth its costs. It must justify the expenses involved. A control system is justifiable if the savings anticipated from it exceed the costs of its working. Small enterprises cannot afford elaborate control systems which are very costly. As far as possible, unnecessary paperwork and reports should be avoided.

5. Flexibility: A good control system must keep pace with the continuously changing environment. It must be responsive to changing conditions. It should be adaptable to new developments such as the introduction of new methods, materials, and changes in the requirements of the customers. If there is a change in any plan, the control system should be capable of absorbing such change.

6. Forward-looking: The system of control should be forward-looking in the sense that it should detect and report deviations promptly. Timeless is the essence of control. Ideally, deviations should be anticipated and preventive action should be taken. More realistically information on deviations should be monitored and feedback to the system immediately so that corrections are effected with minimum cost and inconvenience.

7. Suggesting Corrective Action: Merely pointing out deviations is not sufficient in a good control system. It must lead to taking corrective action to achieve the desired objectives. It may result in taking suitable action against the employees, giving them training, ensuring effective supervision, improving communication, revising standards, etc.

8. Concentration on exceptions: This is also known as ‘Control significant deviations from standards require management’s attention as they constitute exceptions. An attempt.to go through all deviation tends to increase unnecessary efforts and to decrease attention on important problems.

9. Strategic Point Control: Small deviations in certain cases may have greater significance than larger ones in other cases, for example, deviation of ten percent in budgeted labor cost may be more troublesome to a manager than a deviation of twenty-five percent in budgeted postal charges. The Principle of strategic point control states that effective control can be achieved if critical, key, or strategic points can be identified and close attention directed to adjustment at those points. In fact, strategic point control is an extension of control by exception.

Question 2.
Explain the relationship of the controlling the function of management with other functions of management.
Answer:
Relationship of control with other Function of Management: We know that control is the last function of management because its need arises only after performing the functioning of planning, organizing, and directing. But being the last function does not lower down its importance as compared to other functions. Control is very must linked to other functions of management. In fact, the success of all other functions depends upon effective control,

The relation of Control with other managerial functions is explained below:
1. Relationship of Control with Planning: There is a close relationship between the Control and Planning functions of Management. Planning is the basis of Control, The control process involves determining the standards, comparison of the deviations, and taking corrective action to remove such deviations. In this process, planning provides such standards against which the progress of the actual performance is compared.

Thus where there is no plan there can be no basis for control. HG Hick has rightly said that “Planning is clearly a prerequisite for controlling. It is utterly foolish to think that controlling could be accomplished without planning there is no predetermined understanding of the desired performance.

On the other hand, plans will also prove mere imaginations in the absence of control. Because in such a case there will be no one to check whether or not everything is being done according to plans. Also in the absence of control, there will be no corrective action to direct the actual progress towards the accomplishment of plans.

Thus planning is meaningless without control, and control is aimless without planning.

2. Relationship of control with the organization: Control can be meaningful only when the person responsible for deviations is identified and is held responsible. This is possible only because of the organization because it is only in an organization that the authority and responsibility of each one are defined. In the absence of organization, neither the person responsible for committing the mistakes will be identified nor any action can be taken against him. Hence, the same mistakes will be repeated time and again. As such, effective organization is essential for effective control.

Similarly, Organisation is also dependent upon control because organizing is to assign work to individuals and give them authority and fix responsibility in case of mistakes. Since fixing of responsibility’ is possible only through Control, therefore effective control is essential for effective organization.

3. Relationship of control with Direction: Direction removes the difficulties in the way of various functions even before they are reviewed in order to control their progress. Direction makes the employee more capable and enthusiastic towards their work and provides them with adequate guidance training and supervision. Thus direction and control are interlinked with each other.

4. Relationship of control with other functions of Management: Control has deep relations with other functions of management as well. Communication helps in comparison of actual results with the standards by providing timely information of actual results with the standards by providing timely information of actual progress. Decision-making helps in determining the way to remove the deviations and the type of corrective actions to be undertaken.

Motivation helps in encouraging people to work towards the achievement of organizational objectives. Under control performance of each person is evaluated and efficient persons are motivated by providing them financial and non-financial incentives whereas inefficient employees are provided training to remove their deficiencies. Thus, it is clear that all the functions of management are inter-linked.

The relationship of various managerial functions is depicted through the following diagram:
Class 12 Business Studies Important Questions Chapter 8 Controlling 1

From the above diagram, it is clear that the relationship between control and other managerial function is so deep that it is impossible to think of them separately. Every function affects the control and gets affected by it.

Directing Class 12 Important Extra Questions Business Studies Chapter 7

Here we are providing Class 12 Business Studies Important Extra Questions and Answers Chapter 7 Directing. Business Studies Class 12 Important Questions are the best resource for students which helps in class 12 board exams.

Class 12 Business Studies Chapter 7 Important Extra Questions Directing

Directing Important Extra Questions Short Answer Type

Question 1.
Explain the term Directing.
Answer:
Directing – Meaning: In the ordinary sense, directing means giving instructions and guiding people in doing work. In our daily life, we come across many situations-like a father directing his daughter to do work in organizing a function, a teacher directing his student to complete an assignment, a film director directing the artists about how they should act in the film, etc. In all these situations we can observe that directing is done to achieve some predetermined objective.

In the context of the management of an organization, directing refers to the process of instructing, guiding, selling, motivating, and leading people in the organization to achieve its objectives.

You can observe here that directing is not a mere issue of communication but encompasses many elements like supervision, motivation, and leadership. It is one of the key managerial functions performed by every manager. Directing is a managerial process that takes place throughout the life of an organization.

The main characteristics of directing are discussed below:

  1. Directing initiates action: Directing is a key managerial function. A manager has to perform this function along with planning, organizing, staffing, and controlling while discharging his duties in the organization. While other functions prepare a setting for action, directing initiates action in the organization.
  2. Directing takes place at every level of Management: Every manager, from the top executive to supervisor performs the function of directing. The directing takes place wherever superior-subordinate relations exist.
  3. Directing is a Continuous process: Directing is a continuous activity. It takes place throughout the life of the organization irrespective of people occupying managerial positions. We can observe that in organizations like Infosys, Tata, and BHEL, HLL – the manager may change but the directing process continues because without direction the organizational activities cannot continue further.
  4. Directing flows from top to bottom: Directing is first initiated at the top level and flows to the bottom through the organizational hierarchy. It means that every manager can direct his immediate subordinate and take instructions from his immediate boss.

Question 2.
Explain in brief the importance of supervision.
Answer:
Supervision: The term supervision can be understood in two ways. Firstly, it can be understood as an element of directing and secondly as a function performed by supervisors in the organizational hierarchy.

As an element of directing every manager in the organization supervises their subordinates. In this sense, supervision can be understood as the process of guiding the efforts of employees and other resources to accomplish the desired objectives. It means overseeing what is being done by subordinates and giving instructions to ensure optimum utilization of resources and achievement of work targets.

Secondly, supervision can be understood as functions to be performed by a supervisor, a managerial position in the organization hierarchy at the operative level i.e., immediately above the worker. The functions and performance of a supervisor are vital to any organization because he is directly related to workers whereas other managers have no direct touch with bottom-level workers.

Importance of Supervision:
The importance of supervision can be understood from multiple roles performed by a supervisor. These are explained below.
1. Supervisor maintains day to day contact and maintains friendly relations with workers. A good supervisor acts as a guide, friend, and philosopher to the workers.

2. Supervisor acts as link-between workers and management. He conveys management ideas to the workers on one hand and workers’ problems to the management on the other. This role played by the supervisor helps to avoid misunderstandings and conflict between management and workers/employees.

3. Supervisor plays a key role in maintaining group unity among workers placed under his control. He sorts out internal differences and maintains harmony among workers.

4. Supervisor ensures the performance of work according to the targets set. He takes responsibility for task achievement and motivates his workers effectively.

5. Supervisor provides good on-the-job training to the workers and employees. A skilled and knowledgeable supervisor can build an efficient team of workers.

6. Supervisory leadership plays a key role in influencing the workers in the organization. A supervisor with good leadership qualities can build up high morale among workers.

7. A good supervisor analyses the work performed and gives feedback to the workers. He suggests ways and means of developing work skills.

Question 3.
Explain in brief the elements of the communication process.
Answer:
Elements Of Communication P: Communication has been defined as a process. This process involves the elements like source, encoding, media/channel, receiver, decoding, noise, and feedback. The process is represented in the figure below.

Communication Process
Class 12 Business Studies Important Questions Chapter 7 Directing 1

The elements involved in the communication process are explained below –

  1. Sender: Sender means a person who conveys his thoughts or ideas to the ‘ receiver. The sender represents a source of communication.
  2. Message: It is the content of ideas, feelings, suggestions, order, etc. intended to be communicated.
  3. Encoding: It is the process of converting the message into communication symbols such as words, pictures, gestures, etc.
  4. Media: It is The path through which an encoded message is transmitted to the receiver. The channel may be in written form, face to face, phone call, internet, etc.
  5. Decoding: It is the process of converting encoded symbols of the sender.
  6. Receiver: The person who receives communication from the sender.
  7. Feedback: It includes all those actions of the receiver indicating that he has received and understood the message of the sender.
  8. Noise: Noise means some obstruction or hindrance to communication.

The hindrance may be caused to sender, message, or receiver. Some examples of noise are:
(a) Ambiguous symbols that lead to faulty encoding.
(b) A poor telephone connection.
(c) An inattentive receiver.
(d) Faulty decoding (attaching wrong meanings to message)
(e) Prejudices obstructing the poor understanding of the message.
(f) Gestures and postures that may distort the message.

Question 4.
Explain in brief the functions of a supervisor.
Answer:
Functions of a supervisor:
The function performed by a supervisor are briefly discussed below –
1. Preparation of Work Schedules: Scheduling involves laying down the time for starting and completing various activities. The supervisor determines the schedules of work for every individual in his unit or section. This is done to ensure a steady flow of work.

2. Improving Communication: The supervisor maintains direct contact with the subordinates which leads to effective communication. He also provides leadership to the workers of his department. He fixes production targets for them and provides them the necessary guidance for doing the work assigned to them.

3. Optimum Utilisation of Resources: The supervisor issues orders and instructions to the workers for achieving coordination in section. He tells them what to do and how to do it so that they may utilize machines, materials, money, and methods effectively.

4. Providing Motivation: The supervisor motivates his subordinates by providing financial and non-financial incentives. He inspires them for higher quality and productivity.

5. Control of Performance: The supervisor controls the performance of the workers by comparing their performance with the standards. He takes necessary action to ensure that goods are produced according to the predetermined standards. He also provides feedback to the subordinates about their performance and gives them counseling for improvement.

6. Reporting: The supervisor keeps records of output and other related aspects of each employee. On the basis of records, he sends performance reports and other necessary information to his superior.

7. Link Between Management and Workers: The supervisor is an important link between the management and the workers. He explains management policies to the workers and also passes on the management’s instructions. He has close contact with the workers and tries to understand their problems. He brings worker’s problems to the notice of the top management.

8. Human Relations: A supervisor tends to achieve good human relations in his unit. He can mix up with the workers and share their joys and sorrows. He also settles conflicts between workers or groups of workers.

9. Grievance Handling: A supervisor is in direct touch with the workers, so he can handle their grievances effectively. When a grievance is reported, he listens to the worker’s viewpoint and tries to remove the cause of grievance. But if he can’t redress the grievance he should report it to the upper-level management.

Question 5.
Differentiate between formal and informal communication.
Answer:
Comparison of Formal and Informal Communication:

Formal Communication Informal Communication
(i) It follows the official chain of command. (i) It is based on personal relationships and does not follow a fixed pattern.
(ii) It is slow as it has to follow the path laid down by the management. (ii) It ¡s very fast as it’s not supposed to follow a particular path.
(iii) t is rigid as deviations are not allowed. (iii) It is flexible as ¡t moves freely.
(iv) Formal communication ¡s generally accurate. (iv) Infonnal message may not be authentic.
(v) Chances of wrong information are very few. (v) Chances of distortion of information are very high.
(vi) In the case of formal communication, the status or position of the parties is very important. (vi) In case of informal communication, status or position. of the parties has no relevance.
(vii) It serves the needs of the organization. (vii) it serves the social needs of the members and also of the organization.

Question 6.
What is the importance of leadership? Explain.
Answer:
Importance of Leadership:
Leadership is considered the most important element of the directing function of management. It supports all other managerial functions by assisting in the formulation and execution of plans. Good leadership provides the following benefits.

1. Clarification of Goals: A leader interprets and explains the objectives of the group to his followers. As a result, the members of the group know the targets to be achieved and the contribution, each of them is to make towards common objectives. They are not likely to go astray and will continue in the right direction.

2. Motivation: A good leader creates- an urge for higher performance among people. He creates self-confidence and enthusiasm among his subordinates. He converts lukewarm desire into a burning passion for success. A sound leader can create an environment conducive to hard work. He directs the potential talent of employees towards the achievement of goals.

3. Moral Building: A leader builds up dedication and loyalty among a group of people. He develops mutual cooperation and self-discipline among people. The persons become ready to sacrifice even their lives for the good of the common goal. Under a good leader, people work willingly and enthusiastically. The leader encourages subordinates to take initiative and provides psychological support to them. He serves as a friend, philosopher, and guide for his group.

4. Teamwork: An organization can be successful only when all its members work together as a team rather than going in different directions. It is the leader who creates team spirit and coordination among different members of the group. He resolves internal conflicts and differences, of opinion. He serves as an arbitrator and mediator among the members. A leader harmonizes the personal goals and aspirations of subordinates with the goals of the organization as a whole.

5. Creates dynamic environment: In the dynamic environment of today, frequent changes are required in the structure and working of an organization. But change creates uncertainly and inconvenience. Therefore, people tend to resist change. A good leader persuades people to accept and carry out the desired changes. A leader is an important agent of organizational change and development. He provides psychological support to his followers.

6. Representation: A leader serves as the representative of his followers. He protects their interest and serves as their guardian. He acts as their spokesman and bargains with the outside forces for the welfare of the groups. A true leader upholds the interests of his followers and attempts to fulfill their hopes and aspirations. He is always ready to solve the problems of his followers. A leader manager represents his organization in business meetings, trade conferences, government committees, and so on.

Competent leadership is required at all levels of management. All managers must provide leadership so as to create an urge in the employees to cooperate and improve their performance towards the achievement of organizational objectives.

Directing Important Extra Questions Short Answer Type

Question 1.
Explain the nature and importance of communication in today’s changing business world.
Answer:
Nature Of Communication:
The foregoing definitions reveal the following characteristics of communication –
1. Pervasive Function: Communication is required at all levels of management and in all departments of the organization. It is an indispensable part of the management process. It is an activity of each and every manager. Therefore, communication is regarded as a pervasive function.

2. Continuous Process: Communication is an ongoing process that has to be in regular touch with their subordinates and superiors to maintain and improves performance.

3. Two-Way Process: Communication is a two-way process. It includes sending a message and the response to that message. It is not complete until the reaction or response to the message is available. The reaction or response is known feedback.

4. Circular Process: Communication becomes a circular process with feedback. The flow of communication is a circular one.

5. Two or more Parties: It requires at least two persons to complete the process of the communication-a sender with the message and a receiver who must understand the message and respond to it.

6. Understanding: The receiver may or may not agree with the point of view of the sender of the message. However, for communication to be complete, the receiver must understand the message in the same sense as intended by the sender.

Importance Of Communication:
Communication is an indispensable part of the process of management. Non an organization can survive and grow without an effective system of communication. Since the job of a manager is to get things done through others, he has to spend a major portion of his time on communication. The first executive function is to develop and maintain a system of communication. Most of the problems of management arise due to a lack of understanding. Therefore, communication’s the number one problem of management today:

Sound communication offers the following benefits:
1. Basis of Planning and Decision Making Communication is essential for decision-making and planning: The quality of managerial depends on the quality of communication (amount and quality of information available to the organization) Communication provides the necessary information with the help of which managers can diagnose problems, evaluate alternative courses of action and choose the right alternative.

Realistic Planning and sound decision making is not possible without accurate information through communication, for example, the entire sales plan of an enterprise may fail if the information about the latest market condition is not available to management. At the same time, the decisions and plans of management need to be communicated to the subordinates. Effective communication is also helpful for the proper implementation of plans and policies of the management.

2. Smooth and Efficient Operation: An effective communication system serves as a lubricant, fostering the smooth and efficient functioning of the enterprise. The achievement of goals of the enterprise is of paramount importance and communication is one of the important tools available to the manager to attain them. It is through communication that a manager issues orders and instructions and changes and regulates the behavior of subordinates in the desired direction. Effective communication promotes managerial efficiency and facilitates leadership.

3. Facilitates Coordination: In every organization, the work to be done is divided among several interrelated departments and sections. The activities and efforts of different individuals and groups must be coordinated. Communication is the most effective means of creating cooperation and coordination. The exchange of ideas and information helps in bringing about the unity of action in the pursuit of a common purpose.

Communication binds people together. Group meetings used in coordination involve the exchange of ideas and knowledge and the transfer of information and understanding. Communication is at the root of all group activity. It is through coordination that managers come closer to employees. Interaction and discussion between the two sides improve the superior-subordinate relationships.

4. Employee Motivation and Moral: Communication helps management to keep the employees fully informed about the plans, job changes, etc. The motivation and morale of employees tend to be high when they clearly understand what they are supposed to do. Sharing of information with employees and discussion with them on matters of common interest provide satisfaction to employees.

Communication is the means by which employees can bring their suggestions, difficulties, and grievances to the notice of the management. Upward communication ensures greater job satisfaction and stimulates worker’s enthusiasm and loyalty towards the enterprise. Effective communication satisfies the personal and social of employees.

5. Sound Industrial Relations: Effective communication helps to create mutual understanding and trust between the employer and the employees. It enables the management to come into close contact with workers. It serves as a bridge between management and labor and creates a spirit in the organization. Thus, an effective communication system is a prerequisite for good labor-management relations.

6. Industrial Democracy: Communication is essential for worker’s participation in management. It is helpful in the delegation and decentralization of authority. Effective communication is the basic training and development of managerial personnel. The process of leadership itself depends upon effective communication.

Public Relations: In the modern business world, every business enterprise must create and maintain a good corporate image in society. Communication is an indispensable means of developing a favorable public opinion. It is through communication that management can keep cordial relations with the government, trade unions, customers, and the community.

In modem business the role of communications has increased due to the following reasons:
(a) Increasing size of organizations.
(b) Growing complexities in decision making.
(c) Rapid changes in technology.
(d) Need for better industrial relations.
(e) Growth of trade union movements.
(f) Need to improve public relations,
(g) Increasing Competition.

Question 2.
Explain the nature and importance of Motivation.
Answer:
Importance Of Motivation:
The success of an organization to a great extent depends upon the motivation of its employees. By motivating employees, managers can obtain their best performance towards the achievement of organizational and individual goals. Motivation is the core of management due to the following reasons.

1. Higher Efficiency: The level of performance of employees in an organization depends on their abilities and willingness to work hard. Motivation bridges the gap between the ability to work and willingness to work. It induces employees to contribute their maximum efforts to achieve a higher level of performance. Therefore, motivation helps in increasing production and reducing the cost of operations.

2. Optimum utilization of Resources: Every organization has physical, financial, and human resources. Effective utilization of physical and financial resources requires competent and motivated people. Motivation creates the willingness to work among employees. It enables managers to achieve the best possible utilization of all resources.

3. Achievement of Organisational Goals: The objectives of an organization cannot be realized unless the people working in it are motivated to work hard contribute their best towards the fulfillment of the assigned tasks. Management can secure the willing cooperation of subordinates towards the accomplishment of organizational goals by satisfying their needs through appropriate rewards. Motivated employees fully cooperate with management in achieving the desired results.

4. Stability in the Workforce: Motivation creates confidence in employees. It helps to improve their loyalty and commitment towards the organization. As a result, the rates of labor absenteeism and labor turnover are reduced. This leads to the maintenance of a stable workforce. The knowledge skills and experience of employees continue to be available to the organization.

5. Cordial Relations: Motivation brings satisfaction among employees through monetary and non-monetary rewards. Therefore, it leads to a friendly and supportive relationship between the employer and employees. Industrial disputes are reduced and the morale of employees is increased.

6. Facilitates change: Effective motivation helps management in overcoming resistance to change motivated employees and to support all changes that are in the interest of the organization. This is because they identify their own progress with the prosperity of the enterprise.

7. Corporate linage: When the employees of an organization are motivated and satisfied, the organization makes rapid progress and its reputation improves. Thus, motivation helps in building a better image of the enterprise. As a result, the enterprise is able to attract qualified and competent people.

Consumer Protection Class 12 Important Extra Questions Business Studies Chapter 12

Here we are providing Class 12 Business Studies Important Extra Questions and Answers Chapter 12 Consumer Protection. Business Studies Class 12 Important Questions are the best resource for students which helps in class 12 board exams.

Class 12 Business Studies Chapter 12 Important Extra Questions Consumer Protection

Consumer Protection Important Extra Questions Short Answer Type

Question 1.
What is the provision regarding enforcement of the orders of District Forum, State Commission, or National Commission?
Answer:
Following are the provision regarding enforcement of orders of redressal agencies
1. Every order of (Redressal Forum, the Agency Commission, or National Commission) shall be a bind if no appeal has been preferred against such order under the provision of this Act.

2. No redressal agency shall admit a complaint unless it is filed within 2 years from the date on which the cause of action has arisen (sec. 24 A (1)).

3. A complaint may be entertained for the period specified above If the complaint satisfies that he had sufficient cause for not filing the complaint within such period [Sec. 24 A (2)].

It is worth mentioning that such delay can be condoned only when the reasons are recorded.

4. Every order made by such agency will be executed in the same way as the court, to which it is sent, shall execute as if it were decree or order sent to it for execution. (Sec. 25).

5. If the redressal agency fails to get its order executed, then it will send the order to the court in whose jurisdiction the dispute falls for its execution. Then the said court shall execute the order as if it were a decree or order sent to it for execution.

Jurisdiction is decided as under:

  • If the order is against a company, the jurisdiction will be divided on the basis of the place of the registered office of the company.
  • If the order is against any person, the jurisdiction will be decided according to the place where the person concerned voluntary resides or carries on business, or personally works for gain (Sec. 25)

6. When a complaint is put up before these agencies it is found to be frivolous or vexatious, it shall, for a reason to be recorded in writing, dismiss the complaint.

It can also be made an order that the complaint shall pay to the opposite party such cost, not exceeding ten thousand rupees, as may be specified in the order.

Question 2.
What are the Penal Provision under Consumer Protection Act?
Answer:
According to Sec. 27, where a trader or a person against whom a complaint is made, or the complainant, fails to comply with any order made by the District Forum, the State Commission, or National Commission, as the case may be, such traders or person or complainant shall be punishable

  • with imprisonment for not less than one month but which may extend to 3 yrs; or
  • with fine which shall be not less than Rs. 2000 but which may extend to Rs. 10,000; or
  • with both.

In case, the redressal agency is satisfied with the circumstances of the case, it can reduce the minimum limits of both imprisonment or fine, mentioned above.

Consumer Protection Important Extra Questions Long Answer Type

Question 1.
What are various prescribed authorities under the consumer protection Act, 1986? Describe their composition, object, and procedure for the meeting.
Answer:
Authorities under the Act as follows:
1. The Central Consumer Protecting Council (Sec. 4 (1)): This section provides provision for the establishment of the Central Consumer Protection Council (now Central Council) by the Central Government. The Central Government may issue a notification in this regard and may specify the date of establishment of such council in the notification.

Composition (Sec. 4(2)): The Central Council shall consist of the following members

  1. The Minister-in-Charge of consumer affairs in Central Government, who shall be its chairman, and
  2. Such member of other official or non-official members as may be prescribed.
  3. The Minister of State of Consumer Affairs in Central Government as Vice-Chairman of Council;
  4. The Minister of Food and Civil Supplies or Minister-in-Charge of Consumer Affairs in State;
  5. 5 members from Lok Sabha and 3 members from Rajya Sabha.
  6. The Secretary of National Commission for Scheduled Castes and Scheduled Tribes;
  7. Up to 20 representatives of the Central Government Department and autonomous organization concerned with consumer interest;
  8. At least 35 representatives of the Consumer Organisation concerned with consumer interest.
  9. not less than 10 representatives of women.
  10. Up to 20 representatives of farmers, trade, and industries.
  11. The Secretary in Department of Civil Supplies shall be the Member Secretary of Central Council.

The object of Central Council (Sec. 6)The Central Council shall work with the objective to promote and protect the rights of consumers.

Terms of CouncilTerm of Council shall be 3 years. A member may resign by submitting his written resignation to the chairman, the vacancies shall be filled from the same category by the Central govt, and such person shall hold office so long as the original member would have been entitled to hold office. The Central govt may constitute a standing working group from amongst the member of the council to monitor the implementation of the recommendation of the council.

Procedure for Meeting of Central Council (Sec. 5) The council shall meet as and when necessary, but at least one meeting of the council shall be held every year. The meeting shall be held at such place and at such time as the chairman may think fit.

2. The State Consumer Protection Council (Sec 7(1))This section authorizes the State Government to establish a Consumer Protection council by issuing a notification in this regard and with effect from such date as it may specify in the notification.

Composition Sec 7 (2): The State Council shall consist of the following members

  1. The Minister-in-Charge of Consumer Affairs in State Government as its chairman.
  2. Such member of other official or non-official members, representing various interest, as may be prescribed by State Government; and
  3. Up to 10 other official or non-official member nominated Objects of Council (Sec. 8) The objective of every state council shall be to promote and protect the rights of consumers as laid down in clauses (i) to (v) of section 6 within the state.

The procedure of Meeting (Sec 7 (4)): The State Council shall meet as and when necessary, but at least 2 meetings should be held every year. It shall meet at such time and place as the Chairman may think fit and observe such procedure which is prescribed by State Government for the transaction of its business.

3. The District Consumer Protection Council (Sec 8 A): This section was inserted in 2002 in Act by making amendment in it. ACU to the Section, the State Govt, shall establish District Consumer Protection Council for every district with effect from such date as is specified in the notification.

Composition: The District Council shall consist of the following members:

  • the Collector or Deputy Commissioner as its chairman; and
  • such member of other official or non-official members representing such interest as may be prescribed by State Government.

The object of Council (Sec. 8B): The District Council shall work with the objective of promoting and protecting the rights as specified in clauses (i) to (vi) of sec. 6 with the jurisdiction of the district.

Procedure for MeetingThe District Council shall meet as and when necessary, but at least 2 meetings should be held every yr. It shall meet at such time and place as the Chairman may think fit and observe such procedure which is prescribed by State Govt, for the transaction of its business.

Question 2.
Define the following terms
Answer:
(a) Consumer: Acc. to Consumer Protection Act, 1986, ‘Consumer’ means a person who:
1. Buys any goods for the consideration which has been paid or promised or partly paid and partly promised, or under any system of deferred payment and includes any user of such goods other than the person who buys goods for consideration paid or promised or under any system of deferred payment, when such use is made with the approval of such person, but does not include a person who obtains such goods for resale or for any commercial purpose or, (Sec 2(l) d 0).

2. Hires of avails of any services for a consideration which has been paid or promised or partly paid or promised, or under any system of deferred payment and includes any beneficiary of such services other than the person who hires or avails of services for consideration paid or promised, or under any system of deferred payment, when such services are availed of with the approval of the first-mentioned person.

Explanation Please note that the consumer also includes the user of goods or beneficiary of services when such goods or services are used or availed with the permission of the main buyer.

Also, the consumer does not include a person who uses the goods or services for the purpose of resale or any other commercial purpose.

But note that, ‘commercial purpose’ doesn’t include used by him exclusively for the purpose of earning his livelihood, by means of self-employment.

(b) Complaint ‘Complaint’ means any allegation in writing made by a complainant that:

  1. an unfair trade practice or a restrictive trade practice has been adopted by any trader;
  2. the goods, brought by him or agreed to be brought by him, suffer from one or more defects;
  3. the services, hired or availed of or agreed to be hired or availed of by him, suffer from a deficiency in any respect;
  4. A trader has charged for the goods, mentioned in the complaint, a price in excess of the price fixed by or under any law for time being in forces or displayed on the goods or any package containing such goods.
  5. Goods, which will be hazardous to life and safety when used, are being offered for sale to the public in contravention of permission of any law for time; being in forces requiring traders to display information in regards to contents, manner, and effect of the use of such goods (sec 2 (1) (c)).

(c) ComplainantAny person or institute mentioned below who files a complaint is called the complainant

  1. A consumer or
  2. Any voluntary consumer association registered under the Indian Companies Act, 1956 or any Voluntary Consumer Association registered under other Act in force in the country.
  3. The Central or State govt., who or which makes a complaint.
  4. In the case of numerous consumers having the same interest, one or more than one consumer.
  5. In case of death of a consumer, his legal heir or representative, who or which makes a complaint (Sec 2(1) (b)).

(d) ManufacturerIt means the person who:

  1. makes or manufacture any goods or parts thereof,
  2. does not makes or manufacture any goods but assemble parts thereof made or manufactured by others and claims the end-product to the goods manufactured by himself, or
  3. puts or causes to be put his own mark on any goods to be made or manufactured by, any other manufacturer and claims such goods to be made or manufactured by himself (sec 2(i)).

Explanation Where a manufacturer dispatches any goods or part thereof no any branch office maintained by him, such branch shall not be deemed* to be manufactured even though the parts so dispatched to it as assembled at such office and are sold or distributed from such branch office.

Question 3.
What is the need and importance of the consumer protection Act in India?
Answer:
Need and Importance of Consumer Protection Act can be explained as follows:
1. Unfair and Deceptive Trade Practices: In case of unfair and deceptive trade practices, such as selling of defective or sub-standard goods, ignoring safety standards, charging exorbitant prices, misrepresenting the efficiency or usefulness of goods, etc. Consumer Protection Act makes producers/traders more accountable to consumers. It also becomes inevitable for consumers to unite at a common platform to deal with issues concerning consumer protection.

2. Lengthy Legal Process: The violation of various Acts by traders/producers means an ordinary consumer has no other remedy but to initiate action by way of a civil suit which involves a lengthy legal process proving to be too expensive and time-consuming. In fact, very often the time, cost, and mental tension involved in the legal process is disproportionate to compensation claimed and actually granted to individual consumers. Therefore, it becomes necessary to involve laws directed at protecting the consumers providing for remedies that are simpler, more accessible, quicker, and less expensive.

3. Impact of other countries: the USA, European Union, Australia, etc. have taken effective and strict measures to protect the interest of consumers. Following these countries, India has also felt the necessity of consumer’s protection.

4. Welfare State India is a welfare state: One of the Directive Principles enshrined in the Indian Constitution is that state shall direct its policy towards securing that operation of economic system does not result in the concentration of wealth and means of production to determinantal to common man-keeping in view the consumer interest, Govt, passed Monopolies and Restrictive Trade, Practices Act, 1969. Later on, in 1984, provisions relating to unfair trade practices were also incorporated in Act. wide powers have been granted to the HRTP commission under the Act to control and prohibit monopolistic, restrictive, and unfair trade practices.

5. Economic DevelopmentDuring 55 years of planning in India, there is a spectacular change in the standards of living. The structural and institutional changes in the economy consequent upon Economic Reforms 1991 clearly indicate that there has been modernization and globalization of the economy wants of the consumers have increased manifold. Hence, the need for safeguarding consumer’s interests has also grown and has become more important.

6. Means of transport and communication: The rapid growth of means of transport and communication has brought the world consumers together. There is a strong ‘demonstration effect’ through Mass media of TV/ cable network that has made the consumers aware that they can no longer be exploited by the business community and kept isolated from other countries as far as their right to safety and health are concerned.

7. Role of Judicial System: Consumer Protection Act, 1986 has vested vast powers to t Supreme Court for the protection of consumer rights, their safety, and health. As a breakthrough, the remedies for consumer protection are now simpler, more accessible, quicker, and less; expensive.

8. LokAdalats: The concept of Lok Adalat in India is catching up fast. They have become part of a speedy, effective, and economical redressal system. Interesting to note, lakhs of cases relating to motor accidents, complaints diagrams Ltd. Delhi Electric Supply Undertaking have been settled involving crores of rupees. The concept of Lok Adalats has now been extended to other areas.

9. Concept of Public Interest Litigation (PIL): For consumer protection, a large number of petitions by way of PIL have been filed before High Courts and Supreme Court. The concept of PIL is catching fast. Under PIL, it is not only the aggrieved person, but any person can move to court in the interest of the weak or a group who or which may not be in a position to seek legal remedy on his own. ” Secondly, a complaint sent to Supreme Court even on postcard may be treated as a writ petition. PIL is virtually consumer interest litigation which has helped a lot in the cause of consumer protection.

10. Consumer Awareness: The spread of education especially higher education has made people aware of their rights as consumers. The relief granted to consumers and important judicial decisions regarding consumer protection or relief is often covered by, newspaper. Rising income has increased the purchasing power of people to spend more. The rise in prices of products has created in consumer an attitude to expect better quality or. at least to expect the product to be worth their money. Consumers expect better services for their durables. Legislation leading to consumer protection has created an awareness among consumers about their rights and remedies available to them.

11. Consumer organizations: There are more than 500 consumer organizations in India. These consumer organizations are performing a number of functions such as bringing out vouchers, journals, monographs, collecting data of different talks, seminars, workshops, and conferences for the purpose of focusing on the problems of consumers and finding solutions thereof.

Question 4.
Explain the problems of consumers under the Consumer Protection Act.
Answer:
Due to illiteracy, poverty, lack of information, etc. consumers has to face many problems every day. They tolerate silently all these because their outlook being traditional, They remain ignorant of their rights. Following are the problems under the Consumers Protection Act
1. Unfair Trade Practices Trade communities are engaged in various activities to increase their sale and change their economic use or to provide some services. They may devise any unfair method viz. false and misleading advertisement, free gifts, lucky draws. They falsely represent that the services are of particular quality or grade. Following are the unfair trade practice.

(a) False and misleading Advertisement: Trade community spends a lot of money on the advertisement of their goods and services but most of these are false and misleading. These are exaggerated and based on unprovable claims. They make advertisement of products of poor quality’s special standard product.

(b) Free gifts and Prizes: The offering of gifts, prizes, or other items with the intention of the net providing there as offered or creating an impression that something is being given or offered free of charge when it is fully or partly covered by the amount charged in the transaction as a whole is treated as unfair trade practice.

2. Spiralling price: The prices of the product are unduly hiked by the {froducers. The rising prices are the result of anti-social activities viz hoarding, black marketing, and creating of artificial scarcity of the product. It leads to consumer exploitation and victimization.

3. Adulteration It is a big consumer problem. Sometimes, it is very hazardous to health. The traders resort to many sources to earn high profits. Mixing animal fat with ghee, harmful seeds with grains and pulses, mustard oil with mineral oil are some of the adulterations.

4. Poor quality products Sale of poor quality products and sub-standard products is also a part of consumer exploitation. The manufacturer makes the declaration that the product is ‘Agmark’ is not sufficient. There is no matter missing to verify that the goods sold to consumers conform with a specification of safety. It results in a large number of the death by using sub-standard products and unsafe domestic products like a pressure-cooker, kerosene stoves, etc.

5. Deceptive packing Many times traders resort to practicing to deceive consumers. They put the smaller quantity of products in a packet or change the spelling of reputed product slightly like Tata Teas, the name ‘Tata Tea’ etc.

6. Underweight supplies Underweighfgoods by the trader to the consumer, For example, each LPG cylinder must contain 14.2 kgs. of gas but many times under-weight cylinders are supplied to customers.

7. Deficiency in Services Deficiency in Services is also a form of customer problem for instance

  • Under delay in courier service
  • wrong billing by electricity and telephone departments
  • under delay in setting insurance accident claims.

8. Negligence in services It is another cause of consumer exploitation, For instance, wrong operations by a surgeon. Many of these incidents are published in newspapers very often.

9. Monpoloistic trade practices Monopolistic is that market condition in which there is a single seller of a certain product in the market so he is in the position to exploit the consumer by charging high prices and low quality of product etc.

Question 5.
What is the redressal machinery for consumer disputes given in Consumer Protection Act, 1986? %
Answer:
The Act provides for three-tier Quasi-Judicial redressal machinery at District, State, and National Levels for redressal of consumer disputes and grievances. The District Forum has jurisdiction to entertain complaints where the value of goods and services complained against is less than Rs. 20 lakhs; the States Commission for claims between Rs. 20 Lakhs and Rs.100 Lakhs; and the National Commission for claims exceeding Rs. 100 Lakhs.
Class 12 Business Studies Important Questions Chapter 12 Consumer Protection 1

1. District Commission: Section 9 (a) of the Act provides for the establishment of a District Forum by the State Government for each district of the state by notification. The State Government may establish more than one District Forum if it deems fit to do so.

  • Composition (sec. 10(1)): (a) The Collector of the District will be its president.
  • Two or more members including at least one woman member, are the persons of ability, integrity, and standing.
  • They are appointed by the State Government on the recommendations of the selection committee, on the terms and conditions, salary, or honorarium, and allowances whatsoever.

The selection committee shall consist of the following, namely:

  1. The President of the State Commission – Chairman
  2. Secretary, Law Department of the State – Member
  3. Secretary-in-Charge of the department dealing with consumer affairs in the State member.

Every member of the District Forum will hold office for 5 years or till the completion of 65 years of age, whichever is earlier and shall be eligible for re-appointment. The members should have adequate knowledge and experience to solve the problems relating to economic, law, commerce, accountancy, industry, public affairs, and administration.

Jurisdiction (Sec. 11 ): District forum has:
(a) Pecuniary Limits Under the pecuniary limits of the district, the forum can entertain complaints up to the dispute of Rs. 20 Lakhs only.

(b) Territorial Limits Under the territorial limits, a complaint shall be constituted in the District forum within the territorial jurisdiction wherein the opposite party resides or carries business or has a branch office or any course of action arises wholly or in part.

The manner in which complaint shall be made (Sec 12): A complaint in relation to any goods sold or delivered or agreed, to be sold or delivered or any service provided or agreed to be provided may be filed with a district forum by

  1. The consumer to whom such goods are sold or delivered or, agreed to be delivered or such service provided or agreed to be provided;
  2. Any recognized consumer association whether the consumer to whom the goods sold or delivered.
  3. One or more consumers, where there are numerous consumers having the same interest, with the permission of the District Forum, on behalf of, or for the benefit of, all consumers so interested; or
  4. The Central or the State Government.

The procedure of Admission of Complaint (Sec 13):
(a) The district forum on receipt of a complaint shall refer a copy of the complaint within 21 days from the date of its admission to the opposite party mentioned in the complaint asking him to give his version within a period of 30 days which can be extended further for 15 days. If the opposite party denies the charges leveled against him, then the District Forum shall settle the consumer dispute in the mariner specified in clause (c) to (9) of .the Section 13.

(b) If the complaint received by it under section 12 relates to goods in respect of which the procedure specified in clauses (c) to (g) sub-section 1 of section 13 cannot be followed, or if the complaint relates to any services, then the district forum shall 1 proceed to settle the dispute on the basis of evidence brought to its notice by the complainant and the opposite party or on the evidence of complainant only if the opposite party omits or fails to take any action.

(c) No proceedings complying with the procedure laid down in sub-sections (1) and (2) shall be called in questions, in any court, on the ground that the principles of natural justice have not been complied with.

Power (sec 13):
(a) for the purpose of this section, the District Forum shall have the same powers as are vested in a Civil Court under the Code of Civil Procedure, 1908.

(b) Every proceeding before the District Forum shall be deemed to be a judicial proceeding within the meaning of sections 193 and 228 of the Indian Code (45 to 1860), and the District Forum shall be deemed to be a Civil Court for the purpose of section 195, and chapter XXVI of the Code of Criminal Procedure, 1973, (2, 1974).

Finding of the District Forums (Sec. 14 (1)): If, after the preceding, conducted under section 13, the District Forum is satisfied that the allegations about services are true or the loss is suffered by the complainant due to goods supplied to him, the forum shall issue an order to the opposite party directing him to do one or more of the following things, namely
(a) To remove the defects in goods;
(b) To replace the goods with new goods.
(c) To return the price to the complainant;
(d) To pay such amount as may be awarded by it as compensation;
(e) To remove the defects or deficiencies in the services;
(f) To discontinue the unfair trade or restrictive trade practice;
(g) Not to offer hazardous goods for sale;
(h) To withdraw the hazardous goods from being offered Or sale;
(i) To cease manufacture of hazardous goods and to desist from services.
(j) To pay such sum (minimum 5% of the value of goods sold) as may be determined by it, if it is of the opinion that loss or injury/ has been suffered by a large number of consumers who are not identifiable conveniently.
(k) To issue corrective advertisement to neutralize the effect of misleading advertisement at the cost of the opposite party;
(l) To Pay for adequate costs to parties,

Appeal (Sec. 15): The aggrieved party may appeal against the orders of District Forum to the State Commission within 30 days of the passing of the order. The State Commission has the power to entertain an appeal after the expiry of 30 days if it is satisfied that there was sufficient cause for delay. But for making an appeal to State Commission, the aggrieved party has to deposit 50% of that amount of Rs. 25,000, whichever is less, in a prescribed manner.

2. State Commission: The Act provides for the establishment of the State Consumer disputes Redressal Commission by the State Government in the State by notification:

Composition (Sec 16):
(a) A person who is or has been or has the qualification of a High Court Judge is appointed by the State Government in consultation with the Chief Justice of the High Court of the State, as its President [ (Sec. 16(a)).

(b) There will be two and other two members*including at least a woman member who are the persons of ability, integrity, and standing. Members should not be less than 35 years of age have a bachelor’s degree from a recognized university and should have at least 10 years of experience in dealing with problems relating to economics, law, commerce, accountancy, industry, public affairs as administration.

Every appointment under sub-section (1) of section 16 shall be made by the State Govt, on the recommendation of a selection committee consisting of the following members, namely:

  1. President of the State Commission – Chairman
  2. Secretary of the Law Department of the State – Member
  3. Secretary-in-Charge of the Department dealing with Consumer Affairs in the State – Member.

Every member of the State Commission shall hold office for a term of 5 years or up to the age of 67 years, whichever is earlier. He may be reappointed for 5 years if he fulfills other conditions (Sec. 16(3)).

Jurisdiction (Sec. 17): This section of the Act provides the jurisdiction of the commission as follows:
(a) The State Commission can entertain complaints where the value of goods or services and the compensation exceeds Rs. 20 lakhs but does not exceed Rs. 1 crore (Amended in 2002);

(b) The State Commission also has the jurisdiction to ‘entertain appeals’ against the orders of any District Forum within the state;

(c) The State Commission also has the power to call the record and pass appropriate orders.

Accordingly to Amendment, 2002, if the opposite party or parties (if these are more than one) actually and voluntarily resides or carries on business or has a branch office or personally works for gain at the t time of instituting the complaint, comes under the jurisdiction of State – Commission. If the opposite party or parties do not reside or carry on business or have a branch office or personally work for gain, but acquiesce in such institution, also comes under the jurisdiction of the commission.

Procedure Applicable for State Commission (Sec. 18): The Provisions of Sections 12, 13, and 14 and the rules made thereunder for the disposal of complaints by the Districts Forum shall, with such modification as may be necessary, be applicable to the disposal of disputes by the State Commission.

Vacancy in the office of the President (Sec. 18 A): This section has been omitted from the Act as per the amendment of 2002.

Appeals (Sec. 19): Any aggrieved person by an order made by the State Commission in the exercise of its power conferred by sub-clause
1. of clause (a) of Section 17 may prefer an appeal against such order to the National Commission within a period of 30 days from the date of the passing of the order in such form and manner as may be prescribed.

The National Commission may entertain an appeal after the expiry of the said period if it is satisfied that there was sufficient cause for not filing it within that period. As per Amendment Act, 2002 the National Commission shall not entertain the ‘Appeal’ unless the appellant has deposited in the prescribed manner 50% of the amount of Rs. 25,000 whichever is less.

Sec 19A(inserted in Amended Act, 2002) provides that an appeal filed before the State Commission or National Commission shall be f heard as expeditiously as possible and an endeavor shall be made to finally dispose of the appeal within a period of 90 days from the date of its admission.

3. National Commission (Sec. 20): Clause (c) of Sec. 9 provides for the establishment of the National Consumer Dispute Redressal Commission by the Central Government by giving notification in the Official Gazette.

Composition (Sec. 20):
(a) National Commission shall consist of a person who is or has been the judge of the Supreme Court, to be appointed by the Central Government (in consultation with the Chief Justice of India) who shall be its president.

(b) Not less than four iid, not more than such number of members as may be prescribed, one of whom shall be a woman, who shall have the following qualifications, namely;

  • be not less than 35 years of age;
  • possess a bachelor’s degree from a recognized university; and
  • be persons of ability, integrity, and standing and have adequate knowledge and experience of at least 10 years in dealing with problems relating to Economics, Law, Commerce, Accountancy, Industry, Public affairs, or Administration.

Every appointment under this clause shall be made by the Central Government on the recommendation of a selection committee consisting of the following namely;
(a) a person who is a Judge of the Supreme Court, to be nominated by the Chief Justice of India- Chairman
(b) The Secretary of the Department of legal affairs in the Government of India – Member.
(c) Secretary of the Department dealing with consumer affairs in the Government of India – Member.

Every member of the National Commission shall hold office for a term of 5 years or up to the age, of 70 years, whichever is earlier. A member shall be eligible for reappointment for another term of 5 years if he satisfies the qualifications and other conditions for the appointment mentioned above.

Jurisdiction Sec 21 provides that the National Commission shall have jurisdiction
(a) To entertain complaints about the value exceeding Rs. 1 crore (as per Amended Act 2002);
(b) To entertain appeals against the orders of any State Commission;
(c) To call for the records and pass appropriate orders.

It is important to know that each hierarchy in the Act, is empowered to entertain a complaint by the consumer for the value of the goods or services and compensation. The word ‘compensation’ in the legal sense means suffering, insult, injury, or loss.
(a) Powers and Procedure Applicable to the National Commission (Sec. 22): Sec. 22 (I) provides that the provisions of sections 12, 13, and 14 and the rules made for district forum shall be available to the National Commission for the settlement of disputes.

(b) Review of orders Passed (Sec. 22 (2)): It provides that without prejudice to the provisions of sub-section (1), prove the National Commission shall have the power to review any order made by it when there is an error apparent on the face of the record.

(c) Power to set aside Orders (Sec. 22 (A)): It provides that where an order is passed by the National Commission ex-party against the opposite party, the aggrieved party may apply to the commission to set aside the said order in the interest of justice.

(d) Transfer of Cases (Sec. 22 B): The National Commission has the power of transferring the pending case from the District Forum of one state to a district forum of another state or before one state Commission to another state commission.

(e) Circuit Benches (Sec. 22C): The National Commission shall ordinarily function at New Delhi and perform its functions at such other place as the Central Government in consultation with the National Commission may notify in the Official Gazette from time to time.

(f) Vacancy in the office of the President (Sec. 22D): If the office of a resident of a District Forum, State Commission, or National Commission is vacant or the person occupying such post is absent or is unable to perform the duties, then the senior-most member will preside over the national commission.

Appeal (Sec. 23): This section provides that any person who is aggrieved by an order of the National Commission may appeal to the Supreme Court within a period of 30 days from the date of order. The Supreme Court may entertain an appeal after the expiry of the said period if it is satisfied that there was sufficient cause for delay, No appeal will be entertained by the Supreme Court unless that person has deposited 50% of that amount of Rs. 50000 whichever is less.

District Forum, the state commission, or the National Commission shall entertain the complaint if it is made within 2 years from the date on which the cause of. action arises. The District Forum, the State Commission, or the National Commission may entertain such complaints. if they are satisfied with the cause of the delay.

Staffing Class 12 Important Extra Questions Business Studies Chapter 6

Here we are providing Class 12 Business Studies Important Extra Questions and Answers Chapter 6 Staffing. Business Studies Class 12 Important Questions are the best resource for students which helps in class 12 board exams.

Class 12 Business Studies Chapter 6 Important Extra Questions Staffing

Staffing Important Extra Questions Short Answer Type

Question 1.
State whether the following statements are true or false.
(i) “Personal Manager is both a line manager as well as a staff manager.”
Answer:
True.

(ii) Staffing and employing refer to the same activity.
Answer:
True.

(iii) Personnel problems exist only in a large organisation.
Answer:
False.

(iv) Recruitment and selection are one and the same thing.
Answer:
False.

(v) Training and development are synonymous.
Answer:
False.

Question 2.
What is meant by staffing? How staffing is a line as well as staff function?
Answer:
In simple words, staffing is the processing of obtaining and maintaining capable and competent people to fill all positions from top management to the operative level. This includes securing, recruiting, selecting, training, appraising and maintaining the individuals in organizations. Let us pull the views of management scholars on the definition of staffing.

  • Staffing is the function by which managers build an organization through the recruitment, selection, development of individuals as capable employees.
  • Staffing is the executive function which involves the recruitment, selection, compensating, training, promotion and retirement of subordinate managers.
  • Staffing is concerned with the placement, growth development of all those members of the organization whose function is to get things done through the efforts of another individual.
  • Staffing is the whole personnel function of bringing in and training the staff and maintaining favourable conditions of work.

Almost all the scholars unanimously agree that –
1. Staffing is a function involving the recruitment, selection and training and development of people.

2. Staffing is broad enough to cover both rank and file employees and managers. Staffing provides the managers with tremendous opportunity to surround themselves with subordinates of their own choosing. More often than not, staffing is equated to hiring the employees in work organization. Actually, staffing is more than hiring, for managers cannot build excellent, effective and efficient organization solely by hiring. The concept -of staffing is so broad to include several activities frequently assigned to the personnel departments such as transfers, discharge, retirements, training, development and orientation.

Truly, staffing concept widens and magnifies the decision zone of managers but at the same time, staffing entails a balanced sharing of the staffing function with the personnel department.

Question 3.
Define personnel Management. How is it different from Human Resource Management?
Answer:
Personnel management is a set of activities and personnel management focusses on the effective use of human resource in an organization. Hence it is also Labelled as Human Resource Management or Human Engineering. The other names for the term Personnel Management are PMIR, PHRM. The Lexicon of personnel management was traditionally dominated by Plippo, Julius, Pigors and Myres, Strauss and Sales. The new scholars in the field include Andrew Dubrin, Dennis Middlemist, Lloyd Byars, Leslie W. Rue, Deris Torrington, Michael Hitt and Charles Carrer etc. Let us briefly see their views on human resource management.

PHRM is that organizational function, which provides specialized concepts, methods, techniques and professional judgement, geared toward effective and efficient utilization of human resources.

Human Resource Management encompasses those activities designed to provide for coordination the human resources of an organisation.

Personnel management is a series of activities enabling working man and his employing organization to reach an agreement about the nature and objectives of the employment, the relationship between them and then to fulfil those agreements.

Personnel management is that function of all enterprises which provides for effective utilization of human resources to achieve both the objectives of the enterprise and the satisfaction and development of employees.

Personnel management is the integration and coordination of human resources in order to move effectively toward the desired objectives.

These definitions need little explanation. What we study in Personnel/HRM:
The organisation is definitely in the people business. Human resource management is that function performed in the organization to facilitate the most effective use of people to achieve organizational goals. Human resource management, in this process, is concerned with –

  1. Manpower planning.
  2. Employee recruitment, selection, training and placement.
  3. Performance evaluation (appraisal), counselling, career development, and training.
  4. Employee’s development.
  5. Employee’s welfare, safety and health.
  6. Maintenance of harmonious Labour relations.
  7. Compensation and fringe benefits.
  8. Job analysis and employment opportunities programmes.

The Objectives of Personnel/HRM:
As things stand now, the management scholars, pedagogues, and researchers – all unanimously agree that human resource management may be conceived as the process of developing, applying and evaluating policies, procedures methods and programmes relating to the individual members and groups in the organization. The conception applied whether the specific employee is a file clerk, a maintenance mechanic, a research and development scientist a marketing executive, a finance view-president or a production superintendent.

Essentially as noted in the previous paragraphs, personnel/human resources management is basically concerned with the management of human resources in sharp contrast to the management of material and other financial resources.

Another sophisticated way of defining human, resource management is in terms of goals/objectives. The goals of HRM, frankly speaking, are the same as the goals of management of an organization in general. Actually, HRM starts with objectives – what the organization is aiming to do about the people it employs.

The overall aim of the personnel/human resource management is to make an effective contribution to the achievement of overall organizational objectives and to the fulfilment of corporate social responsibility. But unfortunately, there are no universal objectives just as there are no universal (absolute) principles governing personnel policies and practices. Personnel objectives and the means for achieving them depend on their context. At the most, they are only certain basic heading and guidelines which provide a conceptual and analytical framework within which organization does what it needs to do in the \yay which best suits itself.

In every organisation, although the human resource managers tend to carry out a unique set of activities having to do with the utilization of human resources, this work is performed with the Primary objective of accomplishing the exact same objective as is the work of other managers – finance, marketing, production and research etc.

Human resource management is basically aimed at –

  1. Helping the organization to reach its goals.
  2. Employing the skilful, intelligent workforce in an organization.
  3. Providing the organization with well-trained and well-motivated employees.
  4. Increasing to the fullest the employee’s job satisfaction and self-actualization.
  5. Developing and maintaining the quality of work life.
  6. Helping the members to maintain ethical policies and., behaviour.
  7. Managing change to the mutual advantage of individuals groups the enterprise and the general public.

Personnel management thus aims at the attainment of maximum individual development maintaining desirable working relationships (between employee and organization) handling human problems in the organization and acquiring, developing, utilizing and maintaining an effective workforce. In more sophisticated terms, the personnel objectives are concerned with the organization, manpower, relationship and responsibility.

Question 4.
Name the components of staffing.
Answer:
The modern concept of staffing comprises of three important components.

  1. Recruitment: Recruitment is a positive step which aims as attracting a number of candidates to apply for the given job. The higher the number of people who apply for a job, the higher will be the possibility of getting a suitable employee one of them.
  2. Selection: Selection, on the other hand, is a negative process. It aims at selecting the most reliable person out of the candidates who have applied for the job.
  3. Training: Training is concerned with up-gradation the knowledge and skills of the employees so that their abolition to perform can be enhanced.

Components of Staffing
Class 12 Business Studies Important Questions Chapter 6 Staffing 1

Question 5.
Explain Employment Interview and its importance.
Answer:
Employment Interview: According to Julius Michael “An interview is a face to face, oral, observational and personal appraisal method.” Usually, it is used as a means of getting information from the candidate. It also involves giving information that will help the applicant make up his mind about the company.

Interviewing the candidates is an important aspect of the selection procedure. The final selection is partly based on the performance of the candidate in different tests and partly on his performance in his final interview. In the interview, the candidate has to appear before the interview or group interviewers. The candidate’s overall personality is judged in the interview. The interview may last for 10 to 20 minutes or even more. Various questions are asked from the candidates and so on. interviewing technique is used in all companies and in the case of all categories of Staff to be recruited.

Importance of Interview: For the selection of the right type of people, employment interview is very important. The advantages of employment interview are as follows:

  1. There is face-to-face contact between the employer and the candidate, the employer can assess the personality traits of the candidate.
  2. The candidate can seek more information about the employer and the job. This creates better the employer and the job. This creates a better understanding of the mind of the candidate.
  3. The communication skill of the candidate can be judged in the interview. His way of thinking can also be known.
  4. the interview is very important where the candidate has not to go through employment tests. The information contained in the application form can be checked during the interview.
  5. Many companies do not follow the elaborate selection procedure as it is costly and time-consuming. They can relay on an interview if it is properly planned and administered.

Question 6.
Explain in brief the types of training.
Answer:
Depending upon the purpose of training of the following kinds of training programmes are used in industry:-

  1. Induction or Orientation Training
  2. Apprenticeship Training
  3. Internship Training

Now we shall discuss these kinds of training:
1. Induction or Orientation Training: Induction is concerned with introducing or orienting a new employee to the organisation and its procedures, rules and regulation.

When a new employee reports for work, he must be helped to get acquainted with the work environment and fellow employees. It is better to give him a friendly welcome when he joins the organisation, get him introduced to the organisation and help him to get a general idea about the rules and regulations working conditions etc. of the organisation.

The benefits of induction or orientation and socialisation of new employees are as follows

  1. It builds up the new employee’s confidence in the organisation and in himself so that he may become an efficient employee.
  2. It gives the new entrant the information he needs such as the location of locker rooms, cafeteria and other facilities, time to break off, leave rules etc.
  3. It promotes a feeling of belonging and loyalty to the organisation among newcomers.
  4. It ensures that new employee does not form false impressions regarding the place of work because the first impression is the last impression.

2. Apprenticeship Training: Apprenticeship training involves imparting knowledge and skills, in a particular craft or trade such as printing tool making etc. The government of various countries have passed laws which make it obligatory on certain employers to provide apprenticeship training to young people. Apprenticeship training is desirable in industries which require a constant flow of new employees expected to become all-round craftsmen. It is very much prevalent in printing trade building and construction and vocations like mechanics, electricians, welders etc. it is similar to on-the-job training.

Under apprenticeship training, the trainee is placed under the supervision of an experienced person who imparts him the necessary skills and regulates his performance. The advantages of apprenticeship training to the trainees are that they receive a stipend while learning and acquire valuable skills which command a high wage in the labour market. In India, there are so many ‘earn when you learn’ schemes, both in the private as well as public sector undertakings. This is also advantageous to employers. Some employers look upon apprentices as a source of cheap labour.

3. Internship Training: Under this method, an educational institute enters into an arrangement with industrial enterprises for providing practical knowledge to its students. Internship training is usually meant for such vocations where advanced theoretical knowledge is to be backed up by practical on the job experience. For instance, engineering students are sent to big industrial enterprises for gaining practical work experience and medical students are sent to big hospitals to get practical knowledge.

The period of such training varies from six months to two years. The trainees do not belong to the business enterprises but they come from the vocational or professional institutions. It is quite usual that enterprises giving them training absorb them by offering suitable jobs.

Question 7.
Explain in brief the importance of training.
Answer:
Training is beneficial to both, employers and employees. A well-trained employee is an asset to the enterprise because his efficiency and productivity are high. Training enables the employees to obtain job security, high earnings and promotion. In fact, management has no choice. Whether or not to train employees. The only choice left is whether training will be imported through a formal and systematic programme or not. In the absence of formal training employees learn by ‘trial and error’. They pick the wrong ways of doing things and the time involved in learning is very long. Formal training helps to minimise time, cost and wastage involved in training. The main advantages of training are as follows

1. Higher Productivity: Training helps to improve the job knowledge, skills and job performance of employees. Well trained employees are more efficient and as a result the quantity and quality of performance increases.

2. Reduced Supervision: Well-trained employees are self-reliant. Trained employees tend to be more professional and disciplined. They take more interest in their jobs. They do not require continuous and intensive supervision. Therefore the supervisors can save their time and energy.

3. Better Safety: Human error or negligence is the major cause of accidents in industries. Employees who lack knowledge and skills regarding their job often commit mistakes. Training makes employees proficient and reduces accidents. Training makes employees safety conscious and enables them to make better use of safety devices.

4. Economy: Trained employees make better and economical use of the materials and machinery. Proper handling of facilities reduces wastage, spoilage and breakage. The loss to damage is minimised and the cost of production is reduced.

5. Higher Morale: Effective training improves job attitudes and self-confidence of employees. They feel that management cares for them. Trained employees can work better and thereby earn rewards. As a result, their motivation and morale are boosted. Higher morale helps to reduce absenteeism and labour turnover. Relations between management and labour can be improved.

6. Promotion and Career Growth: Training enables employees to acquire knowledge and skills for more responsible jobs. It prepares employees for higher positions in the organisation. They can earn promotions more quickly. Thus training facilitates career growth of an employee.

7. Stability and Growth: Through training, an organisation can develop its future executives and thereby ensure its stability. It becomes flexible as well-trained employees can handle a great variety of jobs. Training makes employees more dynamic and adaptive to changes. With the help of well-trained staff, an organisation can smoothly expand and diversity. It can face adverse conditions more effectively.

Question 8.
Explain the term Job Analysis, Job Description and Job Specification.
Answer:
Job Analysis: Job analysis is the process of determining the tasks which comprise the job methods and equipment used in the job and the skill and knowledge required for the successful performance of the job. It involves a systematic and detailed study of a job so as to determine its contents and requirements.

Job analysis serves the following purposes:

  1. Job analysis provides a scientific basis for proper recruitment and selection of personnel.
  2. It helps in placing the right person on the right job.
  3. Job analysis facilitates the training and development of employees by identifying the abilities required for a job.
  4. It helps in the proper evaluation of a job.
  5. Job analysis helps in improving the design and methods of jobs. The information generated by job analysis is used to prepare two statements
    (a) job description and
    (b) job specification.

Job Description: Job description is a written, organised and factual statement of the nature and contents of a job.

It consists of the following information:
(a) Title or name of the job
(b) Location, code no. of the job
(c) Department concerned
(d) Duties involved in the job
(e) Working conditions
(f) Equipment used
(g) Relationship with other jobs.

Job Specification: Job specification is a formal and written statement of the minimum human qualities required for the successful performance of a job. It specifies the knowledge, skills, experience and aptitude which the job holder should possess. Job specification helps in selecting and training the right person for a job.

Question 9.
Explain in brief the merits and demerits of internal sources recruitments.
Answer:
Internal Sources: Such sources of labour supply exist within the organisation. There are two internal sources of recruitment, namely, transfer and Promotion. These are discussed below.

1. Transfer: It involves shifting of an employee from one job to another, one department to another or from one shift to another, Transfer is a good source of filling vacancies with employees from over-staffed departments or shifts. It may also be used as a tool for training.

At the time of transfer, it is ensured that the employee to be transferred to the new job is capable of performing it. In fact, the transfer does not involve any drastic change in the responsibilities, pay and status of the employee.

2. Promotion: It leads to shifting on the employee to a higher position, carrying higher responsibilities, facilities, status and pay. Many companies follow the practice of filling higher jobs by promoting employees who are considered fit for such positions. Filling vacancies in higher jobs from within the organisation has the benefit of motivating the existing employees. It has a great psychological impact on employees. A promotion at the higher level may also lead to a chain of promotions at lower levels in the organisation.

3. Recalling of Laid Off Employees: The term lay off means temporary separation of the employees from the employer because of lack of work or shortage of raw materials, or other reasons. When the situation gets normal, the demand for labour F will increase. The management can recall laid-off employees to fill the vacant positions.

Merits of Internal Sources:
Filling vacancies in higher jobs from within the organisation has the following merits

  1. Employees are motivated to improve their performance.
  2. The moral of the employees is increased.
  3. Industrial peace prevails in the enterprise because of promotional avenues.
  4. Filling of jobs internally is cheaper as compared to getting candidates from external sources.
  5. A promotion at a higher level may lead to a chain of promotions at lower levels in the organisation. Thus many employees are satisfied.
  6. Transfer or job rotation is a tool of training employees for higher jobs.
  7. The transfer has the benefit of shifting the workforce from the surplus departments to those where there is a shortage of- staff.

Demerits of Internal Sources:

  1. When vacancies are filled through internal promotions, the scope for fresh talent is reduced.
  2. The employees may become lethargic if they are sure of time-bound promotions.
  3. The spirit of competition among the employees may be hampered.
  4. Frequent transfer of employees may reduce the overall productivity of the organisation.

Question 10.
Explain in brief on the job methods of training.
Answer:
Under this method, the worker is given training at the workplace by his immediate supervisor. In other words, the worker learns in the actual work environment. It is based on the principle of learning by doing’. On-the-job training is considered to be the most effective method of training the operative personnel.

On the job training is suitable for imparting skills that can be learnt in a relatively short time. It has the chief advantage of strongly motivating the trainee to learn. It is not located in an artificial situation. It permits the trainee to learn on the equipment and in the work- environment, On-the-job training methods are relatively cheaper and less time-consuming. Another important factor in on-the-job training is that supervisor playing an important part in training subordinates.

There are four methods of on-the-job training described below –

1. Coaching: Under this method, the supervisor imparts job knowledge and skills to his subordinate. The emphasis in coaching or instructing the subordinate is on ‘learning by doing’ This method is very’ effective if the superior has sufficient time to provide coaching to his subordinate.

2. Understudy: The superior gives training to his subordinate as his understudy or assistant. The subordinate learns through experience and observation. This technique prepares the subordinate to assume the responsibilities of the superior’s job in case the superior is absent or . he leaves the organisation.

3. Job Rotation: The trainee is systematically transferred from one job to another so that he may get the experience of different jobs. This will broaden his horizon and capacity to do a variety of jobs. Rotation of an employee from one job to another should not be done frequently. He should be allowed to stay on a job for a sufficient period so that he ’ may acquire the full knowledge of the job.

Job Rotation is used by many organisations to develop all-round workers. The employees learn new skills and gain experience in handling, different kinds of jobs. They also come to know the interrelationship between different jobs. Job rotation is also used to place workers on the right jobs and prepare them to handle other jobs in case of need.

4. Vestibule Traning: Vestibule training is adapted to the same work environment as prevails at the actual work-place in the factory. Vestibule training is suitable where a number of persons are to be trained at the same time for the same kind of work. A vestibule training workshop may be set up by an industrial organisation when it is not possible to give training to the employees at the work-place. The training job is entrusted to – the qualified instructors. The main emphasis is on learning rather than on production.

Vestibule training is an attempt to duplicate as nearly as possible the actual conditions of the work-place. The learning conditions are carefully controlled. The trainees can concentrate on training because they are not under any pressure of work. Their activities do not interfere with the regular process of production. Thus vestibule training is very must suitable where a large number of persons are to be trained arid where mistakes are likely to occur which will disturb the production schedules.

Staffing Important Extra Questions Lomg Answer Type

Question 1.
Today staffing is the activity of personnel Department/ Human Resource Management. Explain the functions of Human Resource Management?
Answer:
Creation of Human Resource or Personnel Department: Staffing’ is the responsibility of every manager. However, in not organisation, personnel or Human Resource Department is set up under the charge of Personnel or Human Resource Manager. The personnel department serves as a service department. It performs various personnel functions assigned to it by the other departments. The Personnel Manager enjoys the status of a specialist in personnel matters. Normally, persons with post-graduate qualifications in Human Resource Management, Personnel Management and Industrial Relations are preferred for this post.

The establishment of the Personnel Department does not relieve the line managers of the staffing responsibilities. In fact, the staffing function is an inherent part of the job of every manager. The Personnel Manager is appointed to provide expert assistance to them in the performance of their staffing functions of manpower planning, employment, placement, induction, training and performance appraisal. Besides these functions, the personnel department is also responsible for motivation. Working conditions, human relations and personnel records. We shall study these functions under the heading of operative Functions or Responsibilities.

Functions of Human Resources Management.
There are two sets of functions of human resources management. These include

  1. Managerial functions
  2. Operative functions

1. Managerial Functions: The Human resources or Personnel Manager is a part of the management. So he performs the basic managerial functions of planning, organising, directing and controlling in relation to his department Like any other manager, the Personnel Manager performs all the managerial functions.

2. Operative Functions or Responsibilities: The operative functions are the specific responsibilities which are entrusted to the personnel department under the supervision of the Human Resource Manager. There are concerned with, employment, training, development, compensation, integration and maintenance of personnel of the organisation.

A brief description of the basic responsibilities or functions of the Personnel Manager is given below –
1. Employment of Personnel: The first major responsibility of the Personnel Manager is the employment for proper kinds and a number of persons necessary to do various jobs in the Organisation. It involves .manpower, planning, recruitment, selection, placement etc. of the personnel.

Manpower planning helps to determine the manpower requirements for various departments. Recruitment is concerned with the sources of supply of work force, whereas selection involves a number of steps to employ the right type of people for various jobs. The selected employees are placed in the jobs for which they are better suited.

2. Training and Development: After placing the people on various jobs, personnel management is concerned with imparting them training to do their work efficiently. Proper development of Personnel is essential to increase their skills in the performance of their jobs. The personnel department designs and runs the appropriate training programmes for developing the necessary skills among the personnel.

3. Compensation: This function is concerned with the determination of adequate .and fair remuneration of the people for their work. The employees can be compensated both in terms of monetary as well as non-monetary rewards. Factors which must be borne in mind while fixing the compensation or remuneration of personnel are their basic need, requirements of jobs, legal provisions regarding minimum wages, the capacity of the organisation to pay, wage level afforded by the competitors etc. For fixing the wage levels, the Personnel Manager can also make use of techniques like job evaluation, performance rating etc.

4. Motivation of Workforce: Employees work in the Organisation for the satisfaction of their needs. In many cases, it is found that they do not contribute towards the organisational goals as much as they can. This happens because employees are not adequately motivated. The personnel Manager helps the various department managers to devise a system of financial and non-financial rewards to motivate the employees.

5. Maintainance of Good Working Conditions: The employees must be provided with good working conditions so that they like their work and work-place and maintain their efficiency. Working conditions influence the motivation and morale of the employees. These include the measures taken for the health, safety and comfort of the working force. The personnel department also provides for various welfare services which relate to the physical and social well-being of the employees. These may include the provision of the cafeteria, restrooms, counselling, group insurance, education of children of employees, recreational facilities etc.

6. Achieving Good Human Relations: The personnel Manager must provide an efficient system of communication to ensure the two-way exchange of information. Many time industrial disputes occur because of poor communication. The personnel manager should always keep himself in contact with, the trade union leaders to understand their grievances and attempt to remove them so that harmony is maintained in the organisation,

7. Personnel Records: It is the duty of the personnel department to maintain records of the employees working in the enterprise. It keeps full records about their training, achievement, transfer, promotion etc. It preserves many other records relating to the behaviour of personnel like absenteeism and labour turnover and personnel programmes and policies of the organisation. It also maintains various records and registers as required by the Factories Act, the employees state Insurance Act and other Labour Laws.

Question 2.
What is manpower planning? Explain the different steps to be taken while preparing Manpower Planning?
Answer:
Manpower planning or human resource planning is the process of determining scientifically the number and type of employees that an enterprise will need in a specified period of time in future. Its purpose is to ensure that the organisation will have an adequate number c of qualified persons at the proper time to perform various jobs efficiently and with personal satisfaction. Manpower planning consists of the following steps

1. Forecasting Manpower Needs: First of all number and type of personnel required are anticipated. The number of employees required in a future period can be estimated by keeping in mind the expected workload. Workload depends upon the production and sales budgets, expansion plans etc. of the company. The type of employees required is estimated by keeping in view the requirements of job vacancies to be filled. Job requirements can be determined by analysing jobs. Job analysis is a thorough analysis of the job to identify the knowledge, skills and experience required for effective performance.

2. Preparing Manpower Inventory: A detailed list of existing manpower is prepared. Then the number and quantity of existing staff are assessed to determine the extent to which manpower forecast can be met from within the organisation. The qualifications, experience, aptitude etc. Of every employee are analysed. Such an inventory of existing manpower is called manpower inventory or manpower audit.

Manpower inventory will give an idea as to how far the future requirements of manpower can be met from within the organisation. It will reveal the adequacy of manpower in terms of number and skills. Absenteeism and labour turnover and such other manpower problems are also anticipated. A comparison between manpower for cast and manpower inventory will reveal gaps in manpower to be filled in form outside.

3. Formulating Manpower Programmes: Detailed programmes are prepared for recruitment, selection, training, transfer and promotion of employees so as to meet future manpower needs, The first step in the staffing process is the estimation of manpower requirements. It is known as human resource planning or manpower planning. Under it the number and kind of personnel required by the organisation during a specified future period (e.g. one year) are determined. Then the number and type of existing personnel are assessed.

This indicates the extent to which the future manpower needs can be met from within the organisation. It also gives an idea as to how far it is necessary to recruit people from outside. Finally, programmes are formulated to recruit, select and train the required staff over the planning period.

The objectives of estimating staff requirements are to ensure that the organisation has adequate number and quality of employees to fill in the various positions. It is useful in many ways. It continuously provides the personnel required at various levels in the organisation. It enables the organisation to make full use of its resources. The organisation can meet its changing manpower needs without any problem. It is also in a position to fill in vacancies arising from the retirement of its senior managers.

While estimating man-power requirements, the managers should consider several factors which is as follows:
(a) Plans of the organisation concerning products services, expansion of operations etc:
(b) Nature and size of the organisation including the degree of decentralisation, a span of control staff units, departmentation etc.
(c) Type of technology to be adopted i.e. a degree of mechanisation and automation.
(d) Retirement schedule of the existing staff.
(e) Number of employees who may leave the organisation.
(f) the Average number of personnel absent from the job.

Systematic manpower planning necessary due to the following reasons:
(a) Future man-power needs: Future manpower needs cannot be determined without systematic manpower planning. With the help of manpower planning, an organisation can secure the services of the right type of people at the right time.

(b) Scarce talent: Modem organisations require highly specialised technicians and professionals. There is a scarcity of such talent. Manpower planning helps in ensuring an adequate supply of skilled personnel for an organisation. ,

(c) Coping with changes: Changes in technology, products, marketing conditions etc. require
changes in job content, skill requirements, kind of people etc. Manpower planning helps in avoiding a shortage of manpower in some areas and surplus in other areas.

(d) Growth and expansion: Manpower planning is necessary for ensuring replacements from time to time due to retirement and death of existing employees. Moreover, an organisation can properly meet its manpower requirements arising out of expansion and growth schemes. Manpower planning helps in optimum utilisation of manpower and in minimising the cost of labour. Workers who become redundant due to automation can be absorbed in new jobs after some training. This helps to improve industrial.

Question 3.
Describe three off-the-job methods of training.
Answer:
Off-the-job Training: Off-the-job training as the name itself indicates, refers to training conducted away from the actual work setting. There may be a special site in the organization itself or in a non-organizational location elsewhere (for example, vocational school or university). Off-the-job training is particularly useful and appropriate for certain managerial skills such as interpersonal abilities and also for certain production jobs where machinery is employed to control the pace of work-an example may be the assembly-line operation and is also useful for some technical jobs where teaching expertise is found elsewhere.

Some of the common methods of off-the-job training include lectures, conferences, group discussions, role-playing, case studies, programmed instruction, and T. group training.

(a) Lectures and classroom instruction: Classroom training is conducted off the job and is probably the most familiar method. It is an effective means of imparting the information and knowledge quickly to a large chunk of members „ with limited knowledge or no knowledge of the subjects being taught. Lecturing is particularly useful for teaching the factual material, concepts, principles, theories and their application to job situations.

In general, classroom instructions are more frequently used for technical, professional and managerial employees. These ‘ lectures are formally organized talks by the training specialists themselves. Lecturing is an effective method and is interesting especially when able lecturers are employed to impact the knowledge – technical or otherwise. But the disadvantages of lecturing include:

  1. the learners may be passive instead of active.
  2. there is no feedback from the audience regarding their lecturers.
  3. a clear and vigorous presentation on the part of the lecturer requires a great deal of preparation;
  4. the untrained and inexperienced lecturer may deliver an unpalatable lecture, he might rumble of pack too much redundant „ information in a single lecture leaving the important technical details.
  5. lecturing emphasizes the routine memorization of facts rather than the practical aspects of a job. However, the lecture method in training is useful to introduce the subject matter its overview, its principles, laws, classification, and summaries etc. to the listeners. Because of its simplicity and efficiency in imparting knowledge, the lecture method is still alive in work organizations.

(b) The conference method: Instead of indulging in straight lecturing, some organizations prefer to hold conferences where participants are required to pool their ideas, viewpoints, suggestions and discuss them at conferences. Conferences provide a common plate form for intensive and through group discussion and result in suggesting the improved methods of performing work in the organization.

The conference allows the trainee to look at the problem from a broad angle allow him to analyse it more carefully and arrive at conclusion. Conference method is ideal for analysing problems and issues concerning organizations and their members’ conferences reduce the dogmatism and promote understanding between members. Upon close and intensive discussions, members will be willing to accept change, if any for the betterment of the organization. Conferences method has several limitations such as

  1. it is limited to a small group of people ranging from fifteen to twenty-five.
  2. progress of learning is slow because all the members have ‘ full freedom to speak and in the curiosity of participation some. members may come out with totally irrelevant issues, and
  3. some members may feel that the whole conference is useless unless they are made aware of the objectives of holding the conference.

(c) Group discussion: Also known as team discussion, or seminar in the group discussion the members are requested to present papers and discuss the papers in a common platform. The trainees are allowed to read their respective papers and this is followed by a thorough critical discussion. While preparing the paper, the trainee has free access to files concerning the subject and compile the information.

After consulting the necessary files the trainees may discuss the ramifications and complexities of a particular job or work and suggest solutions for the probable problems the trainees are likely to encounter in near future.

(d) Roleplaying: The role-playing goes by a variety of names, such as psychodrama. role-reversal, social-drama, and soon. Role-playing involves the spontaneous acting out of a situation by two or more people under the specific direction of a trainer. The notable characteristic of role-playing is that dialogue usually ensues and the trainees are enthusiastic, playing out their roles.

In role-playing, trainees act out a given role as they would be performing in a stage play. The role players are informed only about the situation and of the role they are expected to play. Role-playing primarily involves hiring, firing, discussions about the grievance procedures employed, employer-employee relationships. The primary advantages of role-playing include

  1. development of leadership skills and decision-making skills of the entire group.
  2. trainees learn the importance of participation in bringing about the acceptance of resource allocation decisions.
  3. human interaction and sensitivity are emphasized in role-playing and
  4. it brings desired changes in employee attitudes and behaviour. However, role-playing can be very time consuming and without competent leadership, it could be a waste of time.

(e) Case studies: Another sophisticated off-the-job- training is through case studies. The case study is based on the firm belief that managerial competence can best be attained through the study contemplation and discussion of concrete cases. The trainees are given the cases and are asked to identify the basic problem and suggest solutions. The case study is primarily useful for supervisory personnel and serves as a useful technique for developing decision-making and problem-analysing skills to the middle managers.

(f) Programmed instruction: Programmed instruction involves breaking, information into meaningful units and rearranging them in a proper sequence to form a learning package. Programmed learning consists of three functions:

  1. presenting questions, facts or problems to the learner.
  2. allowing the trainee to respond and
  3. providing the necessary feedback on the accuracy of his answers. Programmed instruction makes use of books or manuals but normally it is supported by electronic teaching machines, computer systems. In practice, the trainee reads a particular set of materials and then responds to questions usually multiple-choice questions or true-false type questions. If the answer is correct the trainee proceeds to answer the next question. However, if the answer is incorrect the trainee is furnished additional information and is required to respond to questions on that material. This procedure is repeated until the trainee has answered correctly.

Programmed instruction method is appreciated because it incorporates several learning techniques including movement from simple to complex material and provision of feedback. Research reveals that programmed instruction is one of the more effective methods for building knowledge and retention of that knowledge.

(g) T-group training: Also frequently known as sensitivity training T-group training is a process in which several individuals work together for several days for the purpose of buildings self-awareness, understanding of group processes and a greater understanding of interpersonal relationships. The trainees are encouraged to portray their feelings, abilities and needs in building interpersonal relationships.

The basic purpose of sensitivity training to increase the participant’s insight into his own behaviour and the behaviour of others by encouraging an open expression of feelings in the trainer-guided T group laboratory.

Question 4.
Write short notes one
1. Employment Tests
2. Evolution of HRM
Answer:
1. Employment Tests: An employment test is a mechanism (either a paper and pencil test or an exercise) that attempts to measure certain characteristics of individuals. These characteristics range from aptitudes, such as manual dexterity, to intelligence to personality.

Important tests used for selection of employees:
(a) Intelligence Tests: This is one of the important psychological tests used to measure the level of intelligence quotient of an individual. It is an indicator of a person’s learning ability or the ability to make decisions and judgements.

(b) Aptitude Test: It is a measure of individuals potential for learning new skills. If indicates the person’s capacity to develop. Such tests are good indices of a person’s future success score.

(c) Personality Tests: Personality tests provide clues to a person’s emotions, his reactions, maturity and value system etc. These tests provide an overall personality. Hence these are difficult to design and implement.

(d) Trade Test: These tests measure the existing skills of the individual. They measure the level of knowledge and proficiency in the area of professions or technical training. The difference between aptitude test and trade test is that the former measures the potential to acquire skills and the later the actual skills possessed.

2. Evolution of HRM: Human Resource Management has replaced the traditional concept of labour welfare and personnel management. HRM in its – present form has evolved from a number of significant inter-related developments, which date back to the era of industrial revolution emergence of trade union movement lead to the need of a person who could act as an effective line between the owners and workers.

Thus the concept of labour welfare officer came into the being.

His role was limited to the bare minimum welfare activities of employees. In fact, he was looked down by both the workers and the owners.

With the introduction of the factory system, thousands of persons began to be employed under one roof. The job of hiring people for the organisation was given to one man, who later on was assigned the responsibility of recruitment, selection and placement of personnel.

This led to the emergence of a personnel officer in the first place and personnel manager, later on.

Human relations approach recognizes the human factor as the most important instrument of success in the organisation. Fast-changing technological developments, how our, necessitated new skills development and training of employees. People came to be recognized as a valuable resource, which can be further developed. Increase in the scope of the work led to the replacement of personnel manager to the human resource manager. Hence HRM came to mainstream activity due to the failure of the earlier concepts to promote the potential benefit of effective management of the people.

Question 5.
What is the importance of staffing function in today’s environment?
Answer:
Importance of staffing:
Human resources are the foundation of any business. The right people can help you take your business to the top: the wrong people can break your business. Hence staffing function has assured greater ” importance their days because of the rapid advancement of technology, increasing the size of the organization and complicated behaviour of human beings. Human resources are the most important asset of an organisation.

The ability of an organisation to achieve its goals depends upon the quality of its human resources. Therefore, staffing is a very important managerial function. No organisation can be successful unless it can fill and keep filled the various positions provided for in the structure with the right kind of people.

Proper staffing ensures the following benefits to the organisation:

  1. helps in discovering and obtaining competent personnel for various jobs;
  2. makes for higher performance, by putting the right person on the right job;
  3. ensures the continued survival and growth of the enterprise through the succession planning for managers;
  4. helps to ensure optimum utilization of human resources.

By avoiding overmanning, it prevents updo utilisation of personnel and high labour costs. At the same time, it avoids disruption of work by indicating in advance the shortage of personnel; and improves job satisfaction and morale of employees through objective assessment and fair rewarding of their contribution. Staffing function must be performed efficiently by all organisation. If the right kind of employees is not available, it will lead to wastages of materials, time, efforts, resulting in lower productivity and poor quality of products.

The enterprise will not be able to sell its products profitably. It is therefore essential that the right kind of people must be available in the right number at the right time. They should be given adequate training so that wastage is minimum: They must also be induced to show higher productivity and quality by offering them. proper incentives.

Staffing and Human Resource Management (HRM): The affiances and effectiveness of an organization in achieving its goals are determined to a great extent on the competence, motivation and general effectiveness of its human resources. Managing the human component or an organisation is the most important task because the performance of the organisation depends upon how well this function ” is performed. Human resource management is that part of management process which develops and managers the human element of the enterprise considering their resourcefulness in terms of total knowledge, skills, creative abilities, talents, aptitudes and potential for effectively contributing to the organisational objectives.

Human Resources Management is concerned with all aspects of managing the human resources of an organisation. More specifically, human resource management involves determining the organisation’s need of human resources, recruiting and selecting the best available employees, developing counselling and rewarding employees, acting as a liaison with unions and government organisations and handling matters related to the well being of employees. Each of these functions is necessary to some degree irrespective of type and size of the organisation.